MedEd/ Sofia Based SBAs Flashcards

SBAs based on past questions and SOFIA objectives, with a lot of contributions from MedEd. This is the most useful deck.

1
Q

A gentleman presents with acute breathlessness and stabbing chest pain. O/E his respiratory rate is 27bpm with good air entry in all fields. His ECG shows sinus tachycardia and right axis deviation.

What is the most likely diagnosis?
A. Pneumothorax
B. Pneumonia
C. COPD
D. Pulmonary Embolism
E. Sarcoidosis
A

D. Pulmonary embolism

Right axis deviation can sometimes be seen on the ECG of a patient with a PE: this is because the pressure in the pulmonary artery is increased by the clot, which dilates the right atrium and shifts the heart to the right.
The most common ECG finding with a PE is sinus tachycardia. RBBB may also be found, and is associated with increased mortality.

The ‘classic’ ECG finding for a PE is S1Q3T3, though this isn’t actually all that common (~10% cases), which consists of prominent S-waves in lead I, and Q-waves and inverted T-waves in lead III

How well did you know this?
1
Not at all
2
3
4
5
Perfectly
2
Q

A 35 year old lady presents with acute onset SOB, chest pain and one episode of haemoptysis. She has recently noticed a swelling in the left leg. O/E her RR is 28 and HR is 105. You suspect a pulmonary embolism.

What is the first investigation you perform?
A. Chest X-Ray
B. CTPA
C. D-Dimer
D. ECG
E. Peak flow
A

B. CTPA

Although a CTPA is the first investigation you would order, in a situation where you strongly suspect a PE, you begin anticoagulation first with LMWH and give the patient oxygen. The exception to this is when the patient is haemodynamically unstable, whereupon thrombolysis or emboletomy is indicated. Where CTPA is contra-indicated (pregnancy, contrast allergy) a V/Q scan would be used.

How well did you know this?
1
Not at all
2
3
4
5
Perfectly
3
Q

A 23 year old student presents to A&E with SOB. He says it came on suddenly. O/E his trachea is undisplaced with reduced breath sounds on the left. A chest x-ray confirms a 1cm pneumothorax.

What is the most appropriate management?
A. Immediate chest decompression
B. Intercostal drain
C. Aspiration
D. Reassure and discharge
E. Low molecular weight heparin
A

C. Aspiration

Although the patient’s pneumothorax is <2cm, they are experiencing SOB, so aspiration is indicated. If this fails, a chest drain is indicated.

Primary (no underlying cause):
<2cm: discharge and repeat CXR later
>2cm/ SOB: aspiration (if unsuccessful - chest drain)

Secondary (known cause e.g. COPD)
>2cm: aspiration (if unsuccessful - chest drain)
<2cm: chest drain

How well did you know this?
1
Not at all
2
3
4
5
Perfectly
4
Q

What signs would you expect on physical examination of someone with pneumonia?

A. Deviated Trachea, ↓ expansion, Dull to percussion
B. Bronchial Breathing, ↓ expansion, ↓ vocal resonance
C. Central Trachea, ↓ expansion, ↑ vocal resonance
D. Dull to Percussion, ↑ expansion, Pyrexia
E. Tachycardia, ↑ expansion, Cyanosis

A

C. Central Trachea, ↓ expansion, ↑ vocal resonance

Decreased expansion, increased vocal resonance, bronchial breathing in the peripheral lungs, dullness to percussion, pyrexia, and tachycardia are all signs of pneumonia.

How well did you know this?
1
Not at all
2
3
4
5
Perfectly
5
Q

A 71 year old Gentleman is brought in by his carer with a 4 day history of a fever and a cough. As you go to examine him he shouts and asks that you leave his bedroom. His RR is 30, BP 103/68. The lab phones you a hour later and let’s you know his urea is 7.8.

Where would you manage this patient?
A. Admit and treat
B. Treat at home
C. Consider ITU
D. Refer for palliative care
E. Refer to primary care
A

C. Consider ITU

The decision whether to admit is based on the patient’s CURB65 score.
CURB65 is calculated as such:
C - Confusion (AMTS of 8 or below)
U - Uraemia (7.1mmol or higher)
R - Respiration rate (30 or higher)
B - Blood pressure (systolic <100 or diastolic <60)
65 - Patient age of 65 or over

Each element is allocated 1 point:

Low severity: 1 or less indicates treatment at home and treatment with oral amoxicillin

Moderate: 2 or more indicates hospital admission and treatment with I.V. amoxicillin and clarithromycin

High severity: 3 or more indicates ITU admission and treatment with I.V. co-amoxiclav and clarithromycin

How well did you know this?
1
Not at all
2
3
4
5
Perfectly
6
Q

25M presents to A&E with a fever and a cough. He says he has been generally unwell over the last year . O/E he is acutely SOB with a RR of 28. You also note an incidental finding of purple patches on his nose.

What is the most likely causative organism?
A. Pseudomonas Aeruginosa
B. Strep Pneumoniae
C. Pneumocystis Jiroveci
D. Mycoplasma Pneumoniae
E. Haemophilus Influenza
A

C. Pneumocystis Jiroveci

The purple patches mentioned are Kaposi’s sarcomas, which are caused by HHV-8 and are classed as AIDS-defining illness. Given that this person has AIDS, they are more susceptible to unusual infections, in this case the cause is Pneumocystis Jiroveci.

How well did you know this?
1
Not at all
2
3
4
5
Perfectly
7
Q

A 55 year old man presents with a cough and fever. He usually lives in the USA where he is a plumber, but recently travelled to the UK to visit family. He has an extensive smoking history. His wife has brought him to hospital and comments that he has been coughing a lot recently (though bringing nothing up) and seems confused.

Which test will confirm the most likely diagnosis?

A. Chest x-ray
B. Sputum culture
C. Bronchoscopy
D. Urinary antigen test
E. Lung function tests
A

D. Urinary antigen test

This is a history of Legionella which is endemic to the USA. There is an increased chance of infection for those who are frequently exposed to stagnant water, as the bacteria grow in water, and smoking is also a risk factor.
Hyponatraemia is a feature of certain infections (e.g. Legionella, typhoid) and is helpful in narrowing down infective causes of disease. The test for Legionella is a urinary antigen test.

How well did you know this?
1
Not at all
2
3
4
5
Perfectly
8
Q

10F presents to A&E with a fever and a cough and O2 sats: 92%. Her parents don’t seem worried as they are used to bringing her into hospital for treatment for her respiratory condition.

What is the most likely causative organism?
A. Pseudomonas Aeruginosa
B. Haemophilus Influenzae
C. Staph Aureus
D. Coronavirus
E. Legionella Pneumophilia
A

A. Pseudomonas Aeruginosa

The history is hinting at a PMHx of cystic fibrosis. Patients with cystic fibrosis are more susceptible to lung infections, and one organism in particular that affects them is Pseudomonas Aeruginosa.

How well did you know this?
1
Not at all
2
3
4
5
Perfectly
9
Q

A known IVDU is brought into A&E, he was found unconscious by two friends who were worried he might have overdosed. You notice an abscess in his groin. Temp: 39, HR 120, BP 90/50. You immediately admit him.

What is the most likely causative organism?
A. Haemophilus Influenzae
B. Staph Aureus
C. Coronavirus
D. Legionella Pneumophilia 
E. Pseudomonas Aeruginosa

What would be the best treatment?

A

B. Staph Aureus

Staphylococcus Aureus is a common pathogen on people’s skin and is harmles sunless the skin is broken. In this case, IVDU has given Staph Aureus a route past the skin and so it has caused an abscess
Flucloxacillin is the standard treatment for a staph infection
Vancomycin is used if MRSA is detected

How well did you know this?
1
Not at all
2
3
4
5
Perfectly
10
Q

A 35 year-old male soldier presents with cough, malaise, and low grade fever for 3 months. Mycoplasma pneumoniae infection is diagnosed.

What is the most appropriate treatment?
A. Amoxicillin
B. Erythromicin
C. Rifampicin
D. Clalvulanic acid
E. Ciprofloxacin
A

B. Erythromicin

A macrolide (e.g. erythromycin, clarithromycin) is indicated for Mycoplasma pneumoniae infection.
Amoxicillin is commonly used to treat more standard pneumonia.
Rifampicin is part of the therapy for TB.
Clavulanic acid is a beta-lactamase inhibitor that is given to prevent beta-lactam resistance. It is given along with amoxicillin (co-amoxiclav) as part of the treatment for high severity pneumonia.

How well did you know this?
1
Not at all
2
3
4
5
Perfectly
11
Q

A 30 year-old female presents with recurrent throbbing headaches. They last several hours, are severe and left-sided, and are preceded by a tingling sensation in her arms.

What is the most likely diagnosis?
A. Cluster headache
B. Intracranial space-occupying lesion
C. Medication overuse
D. Migraine
E. Tension headache
A

D. Migraine

The tingling sensation described is an aura, and may appear in histories for both migraine and seizures.
The stereotypical description of a migraine is a unilateral, severe, throbbing headache that forces the patient to go and lie down

Acute treatment is with Sumitriptan, an anti-emetic (e.g. Metoclopromide), and NSAIDs

Long-term the patient will be asked to keep a headache diary, to help discover what triggers migraines. They may also be placed on propranolol or topiramate as first line treatment, or amityrptilline as second line. However medication can cause headaches when overused.

How well did you know this?
1
Not at all
2
3
4
5
Perfectly
12
Q

A 45 year-old male has been experiencing very painful headaches behind the eye that make his eyes water for a month. They occur 4 times a week, and he remembers he had a similar string of episodes 4 months ago.

What is the most likely diagnosis?
A. Cluster headache
B. Intracranial space-occupying lesion
C. Migraine
D. Subarachnoid haemorrhage
E. Meningitis
A

A. Cluster headache

Cluster headaches are very painful, generally unilateral headaches which will stereotypically occur behind the eye and make it water. They occur in clusters (hence the name) which is hinted at in this history - the string of episodes 4 months ago. They will also usually occur at the same time of day consistently.
Treatment acutely is 12-15L (up to 100%) oxygen through a non-rebreathable mask and subcutaneous Sumitriptan. Verapamil is used to prevent episodes.

How well did you know this?
1
Not at all
2
3
4
5
Perfectly
13
Q

A 27 year-old woman presents with nausea and headache for a week. Both are worse in the morning and improve throughout the day. She has noticed fatigue and irritability over the last few weeks, and her period is late.

What is the most likely diagnosis?
A. Excessive exercise.
B. Migraine
C. Pituitary tumour
D. Pregnancy associated tension headache
E. Trigeminal neuralgia
A

C. Pituitary tumour

In this case the tumour has grown large enough to cause raised intra-cranial pressure (ICP). Raised ICP causes headache and nausea that are worse when lying down (hence worse after a night of lying down), bending over, or coughing.
Raised ICP can also cause personality and mood changes, and focal neurology.
Papilloedema (blurred edge of the optic disc) can be seen on fundoscopy in those with raised ICP

How well did you know this?
1
Not at all
2
3
4
5
Perfectly
14
Q

A 19 year-old woman presents to A&E with severe headache. She has never felt pain like it, and is sensitive to light. She also has a stiff neck. She has no significant medical history, and her only family medical history is that her mother had polycystic kidney disease.

What is the most likely diagnosis?
A. Acute glaucoma
B. Meningitis 
C. Migraine
D. Subarachnoid haemorrhage
E. Trigeminal neuralgia
A

D. Subarachnoid haemorrhage

Very severe, spontaneous headaches should prompt thoughts of a SAH, especially if the onset is sudden - it will often be described as ‘thunderclap’ or ‘like being hit in the head with a baseball bat’.
The sensitivity to light (photophobia) and stiff neck are signs of meningism - irritation of the meninges - which may occur in SAH.
Polycystic kidney disease is an autosomal dominant condition that increases the risk of SAH.

How well did you know this?
1
Not at all
2
3
4
5
Perfectly
15
Q

A 25 year-old is brought into A&E with loss of consciousness. He was playing rugby and sustained a hit to the temple. After a minute he was able to continue the game and seemed fine, but collapsed an hour later after complaining of worsening headache. His GCS was 12 in the ambulance, and is now 10.

What is the most likely diagnosis?
A. Extradural haemorrhage
B. Intraventricular haemorrhage
C. Meningitis
D. Subarachnoid haemorrhage
E. Subdural haemorrhage
A

A. Extradural haemorrhage

The temple is the site of the pterion - a weak point where the frontal, parietal, temporal, and greater wing of the sphenoid bones meet. The pterion is vulnerable to fracture, and when it fractures there is a risk that the middle meningeal artery (which runs underneath the pterion) will rupture.
Classically with an extradural haemorrhage there will be a ‘lucid interval’ where the patient exhibits no symptoms, followed by headache and rapid deterioration of GCS.

How well did you know this?
1
Not at all
2
3
4
5
Perfectly
16
Q

A 20 year-old student is brought into A&E by her friend with nausea, vomiting, confusion, and a rash. Upon questioning the friend, you discover the patient felt ill for the past few days, spent all their time in their room, and complained of a stiff neck. When their friend found them, they were twitching and jerking on the ground. Blood cultures have been taken, and the patient’s GCS is calculated to be 10.

What is the most appropriate next step in their management?
A. CT Head
B. Lumbar puncture
C. I.V. Benzylpenicillin
D. 500ml I.V. saline
E. Metoclopromide
F. Fundoscopy
A

C. I.V. Benzylpenicillin

A CT head is used to check for raised ICP in meningitis, and so in this case would be of no use, as the decreased GCS and seizure indicate raised ICP. This makes an LP dangerous as it could cause a tonsilar herniation which would compress the medulla oblongata and cause respiratory distress.
There are 4 signs that it is unsafe to perform an LP without prior neurological imaging: papilloedema, GCS 12 or less, continuous or uncontrollable seizures, focal neurology.
Other LP contraindications include: rapidly evolving sepsis, anticoagulant therapy/ thrombocytopenia, infection at the LP site, and cardiorespiratory compromise.
When meningitis is suspected, an LP and blood should be taken, and I.V. or I.M. antibiotics should be started before the results are back. NICE guidelines recommend an immediate parenteral dose of benzylpenicillin, though it’s worth noting that a UK Joint Specialist Societies group recommended ceftriaxone or cefotaxime as first line (follow the below link to see, but use NICE for exams).

https://www.britishinfection.org/files/5614/5674/2938/McGill_meningitis_guidelines_Final_published_proof.pdf

How well did you know this?
1
Not at all
2
3
4
5
Perfectly
17
Q

A 78 year-old male presents with right-sided headache that is worse when he eats. The pain radiates into his shoulders, and has been worsening over the past week. His scalp is tender O/E especially at the temple.

What is the most appropriate next step in his management?
A. Measure ESR
B. Start high dose oral prednisolone
C. Temporal artery biopsy
D. Give opiates
E. Measure CRP
F. Give paracetamol
A

B. Start high dose oral prednisolone

In this case the history suggests temporal arteritis. Temporal arteritis can cause blindness if not treated, so the best course of action is to start high dose steroids straight away. ESR and CRP can be measured to assess how well the disease is being managed by steroids, and temporal artery biopsy is the gold standard to confirm diagnosis.
Temporal arteritis can be accompanied by pain and stiffness in the pelvic and shoulder girdles

How well did you know this?
1
Not at all
2
3
4
5
Perfectly
18
Q

A 65 year-old man with a PMHx of MS presents with sudden bursts of pain. The pain is often triggered by shaving, is severe, and disappears quickly afterwards.

What is the most likely diagnosis?
A. Meningitis
B. Migraine 
C. Temporal arteritis
D. Tension headache
E. Trigeminal neuralgia
A

E. Trigeminal neuralgia

Trigeminal neuralgia is characterised by sudden and intense shooting pain within the sensory distribution of the trigeminal nerve. The pain is typically triggered by touch (e.g. shaving) and can idiopathic, or can be caused by trigeminal nerve compression, shingles (post-herpetic neuralgia), or MS.

How well did you know this?
1
Not at all
2
3
4
5
Perfectly
19
Q

A 70 year old man is brought in by his daughter to the GP. Over the last week he has developed a headache which lasts most of the day and rarely goes. He lives with his daughter and son-in-law as he is prone to falls due to his recent left hip replacement. The daughter also mentions that his father’s behaviour has changed lately and tends to exaggerate some of his stories.

What is the most appropriate next step in his management?
A. MRI scan
B. Routine CT scan
C. Sumitriptan + NSAIDs
D. Urgent CT scan
E. Watchful waiting
A

D. Urgent CT scan

The behavioural changes and implied memory issues suggest dementia. Cerebral atrophy is a feature of dementia, and causes stretching of the vessels in the subdural space as the brain shrinks, increasing the chance of venous rupture and subsequent subdural haemorrhage. Combined with the history of falls, this patient is at high risk of subdural haemorrhage.

How well did you know this?
1
Not at all
2
3
4
5
Perfectly
20
Q

A 26 year old man presents with weakness and paraesthesia that started in his hands and feet a week ago. Since then his arms and legs have become increasingly weak. Reflexes are diminished bilaterally and equally, as is power. He can recall no significant PMHx bar some diarrhoea two weeks ago.

Given the diagnosis, what is it most important to monitor in this patient?
A. CRP
B. Forced vital capacity
C. ESR
D. RBC
E. Peak flow
F. Nerve conduction speed
A

B. Forced vital capacity

This history is indicative of Guillain-Barre syndrome, which is characterised by an ascending neuropathy after gastrointestinal infection (typically, but not exclusively, with Campylobacter jejuni). This can affect the respiratory muscles causing respiratory failure in roughly 30% of patients. To monitor respiratory function, regular FVC tests are conducted.

How well did you know this?
1
Not at all
2
3
4
5
Perfectly
21
Q

A 40 year-old homeless man is admitted to A&E after being found in a state of confusion. You cannot smell alcohol on his breath, but his records show a previous admission for delirium tremens and treatment for alcohol withdrawal. He seems unsteady on his feet and has trouble following your finger with his eyes. He also has diminished sensation to soft touch in his peripheries, some motor weakness in his legs, and up-going plantar responses. He seems disorientated, though as far as you can tell his memory is intact.

What is the most likely cause of his symptoms?
A. Korsakoff's syndrome
B. Vitamin K + D deficiency
C. Vitamin B deficiency
D. Stroke
E. Anton-Babinski syndrome
F. Parkinson's disease
G. Supranuclear palsy
A

C. Vitamin B deficiency

The alcoholism in this patient’s history should prompt thoughts about both alcoholic damage and malnutrition, as alcohol impairs liver function and GI absorption. In addition, alcoholics often have poor diet. In this case the patient has presented with symptoms of two vitamin deficiencies. The triad of opthalmoplegia (paralysis of the eye muscles), gait ataxia (unsteady on his feet), and confusion suggest Wernicke’s encephalopathy (in reality it is rare for all thee symptoms to be present together) which is caused by a B1 (thiamine) deficiency.

The diminished sensation to soft touch, and up-going plantar responses are features of subacute spinal cord degeneration, which is a consequence of B12 deficiency. Demyelination of the dorsal columns causes the impairment of soft touch sensation, and demyelination of the lateral corticospinal tracts causes the weakness and up-going plantar response.

How well did you know this?
1
Not at all
2
3
4
5
Perfectly
22
Q

A 50 year old man visits his GP complaining of weakness in his right arm. He reports the weakness has gradually developed over the last 2 months. On inspection, the GP notices wasting of his tongue and hyperreflexia. His right arm is spastic, and you can see fasciculations.

What is the most likely diagnosis?
A. Stroke
B. Multiple Sclerosis
C. Parkinson’s disease
D. Motor Neuron Disease
E. Carpel Tunnel Syndrome
F. Herniated disc
A

D. Motor Neuron Disease

The combination of lower (fasciculations, wasting) and upper (rigidity, hyperreflexia) indicates MND, as this is the only option with the potential to affect upper and lower motor neurons.

Spasticity vs. rigidity

Spasticity is characteristic of upper motor neuron lesions (stroke, spinal cord damage, space-occupying lesion, MND) and is often described as clasp-knife rigidity. This is because tone at the start of a movement will be greatest (as when unfolding a penknife) and will then suddenly decrease through the rest of the movement. Spasticity is also velocity-dependent, meaning the faster you attempt to move a joint, the greater the resistance will be.

Rigidity is an extrapyramidal sign generally seen more in Parkinson’s disease, and is divided into lead-pipe rigidity, and cog-wheel rigidity. Lead-pipe rigidity is consistent rigidity throughout the whole movement of a joint. Cog-wheel rigidity is rigidity superimposed onto a tremor, which causes intermittent rigidity throughout a movement, like the cogs of a gear catching as it turns.

How well did you know this?
1
Not at all
2
3
4
5
Perfectly
23
Q

A 70 year-old man is brought in to see the GP by his daughter because he has fallen three times in the last week. She is also worried about his memory and says his mood has been low. Throughout the consultation the man seems expressionless, and his arm moves slowly when he goes to shake your hand.

What is the most likely diagnosis?
A. Lewy body dementia
B. Stroke
C. Korsakoff's syndrome
D. Parkinson's disease
E. Depression
F. Hypothyroidism
G. Space-occupying cerebral lesion
H. Myasthenia gravis
A

D. Parkinson’s disease

Parkinson’s disease is defined by a triad of: resting pill-rolling tremor (4-6Hz), bradykinesia, and rigidity. Postural instability is also considered to be a key feature, and causes the patient to adopt a stooped posture with a shuffling gait (arm swing will also be absent).

Bradykinesia will generally first be noticed as difficulty with fine motor tasks - combing hair, doing up shirt buttons, tying shoe laces

The postural instability may worsen into gait freezing (short periods of time where the patients are unable to initiate movement while walking) or festination (where a combination of stooped posture and shuffling gait leads to increasingly rapid short steps and ends in a fall)

Parkinson’s is caused by loss of dopaminergic neurons in the substantia nigra, which is part of the basal ganglia. The basal ganglia has a key role in modifying motor output, and this dysfunction is responsible for Parkinson’s symptoms.

There are 5M's of Parkinson's disease to remember (apart from the four classic signs):
Micrographia
Monotonous speech
Mask-like face
Misery - depression
Memory - dementia

It is also worth mentioning that one of the earliest signs of Parkinson’s is loss of smell sensation.

How well did you know this?
1
Not at all
2
3
4
5
Perfectly
24
Q

A 55 year old gentleman is accompanied to the GP by his daughter. It transpires that he has started swearing at people in the street and flirting with all the women he meets. He is able to chat to you about current events and his favourite sport team’s latest match. What is the most likely diagnosis?

A. Pick’s disease
B. Lewy body dementia
C. Vascular dementia
D. Alzheimer’s dementia
E. Wernicke-Korsakoff syndrome
F. Stroke
A

A. Pick’s disease

Pick’s disease is the most common form of fronto-temporal dementia. The frontal lobe is responsible for inhibition, so antisocial and unrestrained behaviour is indicative of frontal lobe disease.

Also contained within the frontal lobe is Broca’s area (normally on the left) which is responsible for speech production, hence Pick’s disease can cause non-fluent aphasia (though it has not done in this case).

Semantic dementia may also occur - this is loss of semantic memory (memory of facts, rather than memory of specific episodes of time) that will often manifest as loss of memory of word meanings.

Loss of planning ability and deficiencies in working memory may also present with Pick’s disease, as they are functions of the frontal lobe.

How well did you know this?
1
Not at all
2
3
4
5
Perfectly
25
Q

A 70 year-old man is brought in by his wife who has noticed a recent sharp decline in his mental status and memory. He seems confused, and on examination has weakness in his left leg, and appears to have some trouble swallowing. He has previously has three TIAs, and thinks the weakness in his leg may have started after the last one.

What is the most likely diagnosis?
A. Malignancy
B. Alzheimer's disease
C. Stroke
D. Polymyalgia Rheumatica
E. Vascular dementia
F. Myasthenia Gravis
G. Multiple sclerosis
A

E. Vascular dementia

Vascular dementia is cognitive decline due to infarction. Typically a patient will present with cognitive decline, and some focal neurology (leg weakness, dysphagia) on a background of ischaemic disease (CVA, TIA, MI). Vascular dementia causes deterioration in a step-wise fashion, as the trigger for each deterioration is a new infarct.

How well did you know this?
1
Not at all
2
3
4
5
Perfectly
26
Q

A 68 year-old man presents with a decline in his cognitive abilities. He is brought in by his son because the previous day he began talking to someone who wasn’t there. The son states that his father has seemed “mentally slower” than usual, and that his memory is worse than it was. On further questioning the son states that his father has been struggling to care for himself for the past 6 months, that he has fallen a few times, and that he has trouble dressing himself as he finds his shirt buttons to be fiddly. The patient’s wife has also mentioned to the son that the father has become very restless during the night and moves about a lot in his sleep.

What is the most likely diagnosis?
A. Parkinson's disease
B. Alzheimer's disease
C. Lewy body dementia
D. Supranuclear palsy
E. Subdural bleed
F. Multiple sclerosis
G. Cerebellar stroke
A

C. Lewy body dementia

The essential feature of LBD is dementia: the characteristic memory issues that often accompany dementia may well not appear till later on, but visuoperceptual defects, attention deficits, and executive dysfunction may all be early and prominent signs.

There are three other core features:
Visual hallucinations (typically vivid and detailed)
REM sleep behaviour disorder (loss of muscle paralysis during REM sleep - the patient can be seen to act out their dreams)
Parkinsonism (bradykinesia, tremor, rigidity)

How well did you know this?
1
Not at all
2
3
4
5
Perfectly
27
Q

A 55 year-old known alcoholic is brought into A&E after his wife noticed irregularities in his memory. As you are interviewing the wife, the patient begins to talk about how they were married in the snow, but the wife then states they had a summer wedding on a nice day. You notice that it is snowing outside as this conversation takes place. O/E the patient seems a little unsteady on his feet, and has a nystagmus in his left eye.

What is the most likely diagnosis?
A. Cerebellar stroke
B. Lateral medullary syndrome
C. Subdural bleed
D. 6th nerve palsy
E. Toxoplasmosis
F. Wernicke's-Korsakoff's syndrome
G. Hepatic encephalopathy
A

F. Wernicke’s-Korsakoff’s syndrome

The ataxia and nystagmus are both suggestive of Wernicke’s encephalopathy. The memory problems are indicative of Korsakoff’s syndrome. Both syndromes are caused by thiamine deficiency. It has been previously sugested that Korsakoff’s syndrome is due to direct alcoholic brain damage, but this is probably incorrect as it has been observed in malnutritioned patients with anorexia nervosa, or who are post-bariatric surgery.
That said, both Wernicke’s and Korsakoff’s are usually caused by alcoholism, as alcohol impairs the absorption, liver storage, and conversion of thiamine to its active form.

Wernicke’s tends to occur before Korsakoff’s, and is reversible with thiamine supplements. By the time Korsakoff’s manifests, the damage to the thalamus and mammillary bodies is generally irreversible.

The memory issue described here is confabulation: when the damage to the thalamus and mammillary bodies becomes extreme, the patient will start to become incapable of recalling old memories or making new ones. Instead the patient will confabulate: the brain will create false memories based on their environment (hence the snowy wedding), but they will believe these memories to be true.

How well did you know this?
1
Not at all
2
3
4
5
Perfectly
28
Q

A 40 year old man starts to make random jerky movements at points throughout the day. Worried about this, he visits his GP. Upon questioning, he informs the GP that his father died in his 40s, but he was too young to remember why, although he did have similar symptoms.

What test should be arranged?
A. FBC
B. Karyotyping 
C. Whole genome sequencing 
D. CAG repeat testing
E. MRI head
F. CT head
A

D. CAG repeat testing

The family history of early death, together with the choreiform (dance-like) movements points to Huntingdon’s disease. Huntingdon’s disease is an autosomal dominant condition in which the number of CAG repeats in the Huntingtin gene (4p16.3) increases, causing a malformed protein.
The number of CAG repeats in an individuals Huntingdin gene determines whether they will develop the disease:
35-39 repeats, and they might develop the disease
40+ repeats, and they definitely will
If they have between 29 and 34 repeats, they will not develop symptoms, but their children would be at risk
28 or below is a normal result

How well did you know this?
1
Not at all
2
3
4
5
Perfectly
29
Q

A 28 year old Norwegian woman presents to A&E after she was unable to fell the hot water on her left leg whilst taking a bath. CSF analysis demonstrated oligoclonal bands that were unmatched with the serum.

Which of the following would most likely confirm a diagnosis of Multiple Sclerosis?
A. Multiple lesions on MRI that all enhanced with gadolinium
B. The patient’s symptoms reoccur 1 year later
C. The patient develops blurry vision in one eye a year later
D. The patient reports blurry vision currently
E. 1 year follow up finds oligoclonal bands matched with the serum

A

C. The patient develops blurry vision in one eye a year later

The most common MS symptoms are:
Optic neuritis (hence fundoscopy is a useful exam in MS)
Sensory disturbance
Motor weakness
Fatigue

The key points when diagnosing MS are:
To have two episodes of CNS dysfunction separated by time
To have two CNS lesions separated by space
There must be no alternative diagnosis

A gadolinium MRI is useful to determine how old brain lesions are, because gadolinium can only enter parts of the brain that are inflamed - as this makes the blood-brain barrier leaky. The inflammation after an acute attack of MS lasts between 2-6 weeks, hence any lesion that lights up with gadolinium contrast is at most 6 weeks old.

Oligoclonal bands are formed when IgG is produced against myelin (as MS is an autoimmune disease). If they are found only in the CSF and don’t match any in the serum, then they have a CNS origin. If they match the serum oligoclonal bands, they could be from infection outside the CNS.

How well did you know this?
1
Not at all
2
3
4
5
Perfectly
30
Q

A 26 y/o male returns from holiday in India. He has had diarrhoea after eating at a seafood restaurant on his last night. He is feverish and nauseous. You notice that the whites of his eyes are yellow.

Which infection is he most likely to have?
A. 	Hepatitis A
B. 	Hepatitis B
C. 	Hepatitis C
D. 	Hepatitis D
E. 	Hepatitis E
A

A. Hepatitis A

Hepatitis A (along with E) is a virus transmitted via the faecal-oral route (think of fAEcal) which causes an acute infection with signs of liver disease, but is ultimately self-resolving. It is particularly associated with eating seafood abroad where there is a risk of the seafood coming from water contaminated with sewage.

How well did you know this?
1
Not at all
2
3
4
5
Perfectly
31
Q

A 64 y/o male with thalassaemia is investigated under the two-week wait for jaundice and weight loss. His blood tests show a raised αFP.

Which chronic infection is he most likely to have?
A. 	Hepatitis A
B. 	Hepatitis B
C. 	Hepatitis C
D. 	Hepatitis D
E. 	Hepatitis E
A

C. Hepatitis C

Hepatitis C is mostly spread through blood, and causes a chronic infection (acute symptomatic infections only occur in 15% of cases). Though hepatitis B can definitely also cause a chronic infection, this only occurs in 5% of infected adults. Though more likely to cause chronic infection, hepatitis C is very reliably curable, whereas hepatitis B is not.

C for cancer, C for chronic, C for curable
Alpha fetoprotein (αFP) is a marker of liver and testicular cancer
How well did you know this?
1
Not at all
2
3
4
5
Perfectly
32
Q

A 32 y/o male returns from holiday in Thailand, feeling ‘under the weather’ with RUQ pain, fevers and nausea. He is jaundiced. He reveals he has used IV drugs and had unprotected sex with a stranger while on holiday.

Which test is most likely to give the correct diagnosis?
A. 	Liver function tests
B. 	HIV serology
C. 	Hepatitis B serology
D. 	Hepatitis C PCR
E. 	CXR
A

C. Hepatitis B serology

B is also spread through blood and sex, but is more likely to cause acute symptomatic infection, such as that described here.

How well did you know this?
1
Not at all
2
3
4
5
Perfectly
33
Q

Which hepatitis virus requires another virus to be present for successful infection?

A. 	Hepatitis A
B. 	Hepatitis B
C. 	Hepatitis C
D. 	Hepatitis D
E. 	Hepatitis E
A

D. Hepatitis D

Hepatitis D needs Hepatitis B surface antigens to be expressed on liver cells in order to enter the cell.
To remember this dependence: two D’s on top of each other look like a B.

How well did you know this?
1
Not at all
2
3
4
5
Perfectly
34
Q

A 16 y/o confused boy is brought to A&E by police after being found wandering the streets. He is disorientated and unable to give a clear history. You notice golden-brown rings round his irises.

What is he likely to have?
A. 	Alcohol intoxication
B. 	Wilson’s disease
C. 	Opiate overdose
D. 	Haemochromatosis
E. 	Hypoglycaemia
A

B. Wilson’s disease

Wilson’s disease is an autosomal recessive fault in copper transporters in the biliary tree (copper is usually excreted in the bile) which leads to accumulation in the liver and deposition in the basal ganglia (neuro symptoms) and the eyes (Kaiser-Fleischer rings).

How well did you know this?
1
Not at all
2
3
4
5
Perfectly
35
Q

A 43 y/o man with T1DM visits the GP for his regular HbA1c reading. You comment on his tanned complexion but he insists he hasn’t been aboard recently. Iron studies are requested.

Which of the following results would fit with your suspected diagnosis?
A. High serum iron, high ferritin, high transferrin, high TIBC
B. High serum iron, low ferritin, low transferrin, high TIBC
C. Low serum iron, low ferritin, high transferrin, high TIBC
D. High serum iron, high ferritin, low transferrin, low TIBC
E. High serum iron, high ferritin, high transferrin, low TIBC

A

D. High serum iron, high ferritin, low transferrin, low TIBC

This is a description of hereditary haemochromatosis - an autosomal recessive condition resulting in an inability to control iron intake. Since the body has no mechanism for disposing of iron once it has been absorbed through the gut, iron accumulates and deposits causing multiple organ dysfunction and failure.

This patient requires a HbA1c reading because they are diabetic, possibly from iron deposition in the pancreas. The tanned complexion is a result of iron deposition in the skin.

Since iron uptake is unregulated, serum iron will be high.
Ferritin will be high, as it increases to deal with increased serum iron
Transferrin will be low because transferrin corresponds to iron demand, and demand is low
TIBC is the measure of the blood’s ability to bind iron with transferrin - since iron is in excess transferrin will be saturated, and the body’s ability to bind any more iron will be low

How well did you know this?
1
Not at all
2
3
4
5
Perfectly
36
Q

A 41 y/o obese female presents with a history of colicky, right sided abdominal pain. She states the pain is worse after eating fish and chips and Indian takeaways. On examination her abdomen is soft and non-tender.

Which is the best investigation to confirm her diagnosis?
A. 	Abdominal X-ray
B. 	ERCP
C. 	Liver biopsy
D. 	USS of biliary tree
E. 	CT-KUB
A

D. USS of biliary tree

This history is suggestive of gallstones given that the pain is colicky, and worse after fatty meals. In addition, the patient has three of the five f’s of gallstones:
Fair (caucasian)
Fertile (children)
Female
Fat (BMI >30)
Forty/ familial (generally risk increases at age 40 or above, but can occur in younger patients with a family history - hence familial)

How well did you know this?
1
Not at all
2
3
4
5
Perfectly
37
Q

A 41 y/o female presents to A&E with a history of severe, continuous, RUQ pain. She feels feverish and complains of an occasional pain in her right shoulder. On examination she displays RUQ tenderness and a positive Murphy’s sign.

What is the most likely diagnosis? 
A. 	Biliary colic
B. 	Ascending cholangitis
C. 	Acute cholecystitis
D. 	Primary biliary cirrhosis
E. 	Cholangiocarcinoma
A

C. Acute cholecystitis

The shoulder pain is called Boas’ sign and, along with Murphy’s sign, is indicative of cholecystitis. The presentation is very similar to ascending cholangitis, but lacks the jaundice and rigors (though rigors are not strictly necessary for diagnosis, they will often be in SBAs to nudge you in the right direction).
Murphy’s sign is pain on inspiration when an examiner places a hand in the RUQ just below the costal margin.

How well did you know this?
1
Not at all
2
3
4
5
Perfectly
38
Q

A 35 y/o man presents with a two week history of jaundice and RUQ pain. He is taking mesalazine for a “bowel condition”.

What is the most likely cause of his jaundice?
A. 	Autoimmune hepatitis
B. 	Haemochromatosis
C. 	Primary sclerosing cholangitis
D. 	Primary biliary cirrhosis
E. 	Drug side effect
A

C. Primary sclerosing cholangitis

The bowel condition being hinted at is most likely ulcerative colitis, which is heavily associated with primary sclerosing cholangitis (75% patients with PSC also have IBD, usually UC). Primary sclerosing cholangitis is most common in men between 20-40.

PSC is strongly associated with perinuclear anti-neutrophil cytoplasmic antibodies (pANCA)

How well did you know this?
1
Not at all
2
3
4
5
Perfectly
39
Q

A 41 y/o female presents to A&E with a history of severe, continuous, RUQ pain. She feels feverish and complains of alternating hot and cold sensation causing her to shiver. On examination she displays RUQ tenderness and appears jaundiced.

What is the most likely diagnosis? 
A. 	Biliary colic
B. 	Ascending cholangitis
C. 	Acute cholecystitis
D. 	Primary biliary cirrhosis
E. 	Cholangiocarcinoma
A

B. Ascending cholangitis

It is caused by obstruction of the bile duct (tumour, gallstone, stricture etc.) which allows bacteria from the duodenum to travel up through the ampulla of Vater into the common bile duct.

Ascending cholangitis is defined by Charcot’s triad:
Jaundice
Fever (with or without rigors)
RUQ pain

As it progresses, Reynold’s pentad may be seen:
Charcot’s triad, but with hypotension and confusion (sepsis)

Management centers initially around treatment of the sepsis:
Administer oxygen
Administer I.V. antibiotics
Administer I.V. saline
Take blood cultures
Take serial lactate measurements
Monitor urine output

Afterwards the original cause is addressed with either cholecystectomy or ERCP

How well did you know this?
1
Not at all
2
3
4
5
Perfectly
40
Q

A 60 year-old man presents with jaundice. He claims to be generally well and has recently started a new exercise program to lose weight, which has worked. He has been feeling a little tired but he attributes this to his strenuous new routine. He previously smoked and drank heavily, ever since he was a young man, but has successfully gone cold turkey on both.

What is the most likely diagnosis?
A. Gilbert's syndrome
B. Carcinoma of the head of the pancreas
C. Hepatic cirrhosis
D. Hepatocellular carcinoma
E. Gallstones
F. Small cell lung carcinoma with metastases
A

B. Carcinoma of the head of the pancreas

Painless jaundice combined with weight loss and a prominent alcohol history should make you consider pancreatic cancer. The jaundice is caused by the tumour enlarging to the point where it obstructs the common bile duct, causing post-hepatic jaundice.
The other cancer options would probably cause pain, as would gallstones, and cirrhosis would cause a whole host of signs not reported in this patient.
Gilbert’s syndrome is an isolated rise in bilirubin with what is otherwise a perfectly functional liver, and so is not concerning.

How well did you know this?
1
Not at all
2
3
4
5
Perfectly
41
Q

A 60 year old obese woman presents to her GP with a
lesion on her breast. The lesion is red, crusted, around her right nipple.

Which is the most likely diagnosis?
A. Breast cyst
B. Breast abscess
C. Paget’s disease of the breast
D. Fibroadenoma
E. Fibrocystic disease
A

C. Paget’s disease of the breast

Paget’s disease of the breast is a stage 1 in situ carcinoma that is often mistaken for eczema or some other skin complaint.

How well did you know this?
1
Not at all
2
3
4
5
Perfectly
42
Q

22 year old woman presents to clinic with a three-month history of a single lump in her right breast. Examination reveals a mobile, firm, smooth and non-tender lump of 2cm in the lower outer quadrant that is not attached to the overlying skin. No axillary lymph nodes are palpable.

Which is the most likely diagnosis?
A. Acute mastits
B. Breast abscess
C. Breast cyst
D. Fibroadenoma
E. Periductal mastitis
A

D. Fibroadenoma

Fibroadenomas are usually found incidentally and are completely harmless. They are common, accounting for ~70% of all breast masses. They can be differentiated from fibrocystic disease because they don’t tend to fluctuate so much with the menstrual cycle.

How well did you know this?
1
Not at all
2
3
4
5
Perfectly
43
Q

A 21 year old woman presents to her GP with pain in her
right breast. She has just given birth to her first child. On
examination, right breast appears swollen, red and there is tenderness on palpation. You do not suspect there are any abscesses present.

Which is the most appropriate management
plan?
A. Antibiotics
B. Refer to surgeon for incision and drainage
C. Refer to breast feeding support group and prescribe
analgesia
D. Watch and wait
E. Confirm diagnosis with USS
A

C. Refer to breast feeding support group and prescribe
analgesia

Mastitis is common when women begin to breast feed, and the best option is to give analgesia and encourage the patient to continue breast feeding. Antibiotics can be given in more serious cases.
Breast abscesses are similar, but gnerally require fine needle aspiration and surgical drainage.

How well did you know this?
1
Not at all
2
3
4
5
Perfectly
44
Q

Risk factors for developing breast cancer include:

A. Having only one child
B. Early menarche
C. Early menopause
D. Anaemia
E. HIV/AIDS
A

B. Early menarche

Many of the risk factors for breast cancer center around increased exposure to oestrogen. Someone who begins menstruating earlier than normal will have a longer than average exposure to oestrogen.
Early menopause decreases risk of breast cancer, as it decreases the length of time a woman is exposed to oestrogen for.
Nullparity is a risk factor because pregnancy induces changes in breast tissue that persist after childbirth, and make the breast tissue less susceptible to carcinoma
Anaemia and HIV both have no particular association with risk of developing breast cancer

How well did you know this?
1
Not at all
2
3
4
5
Perfectly
45
Q

Describe the A FROGMAN risk factors for breast cancer

A

These just seemed quite useful to learn

A - age
F - family history
R - radiation
O - OCP and obesity
G - genetics (e.g. known BRCA mutation)
M - menarche (early) or menopause (late)
A - alcohol
N - nullparity
How well did you know this?
1
Not at all
2
3
4
5
Perfectly
46
Q

A 56 year old woman presents to her GP after noticing abreast lump 3 weeks ago. On examination, the lump is 3 cm in diameter, firm and tethered to the skin and immobile. There are no overlying skin changes.

What is the next most
appropriate investigation?
A. Total mastectomy
B. Urgent mammogram
C. Urgent USS
D. FNAC
E. Core biopsy
A

B. Urgent mammogram

The patient has presented with a lump that is suggestive of breast cancer (firm and immobile) so the next step (after taking a history) is imaging. Since the patient is over 35, a mammogram is required

How well did you know this?
1
Not at all
2
3
4
5
Perfectly
47
Q

A 22 year old medical student presents to her GP
complaining of ‘lumpiness’ in her breasts and nipple
discharge for 2 weeks. She is concerned as her mother,
who had breast cancer, had the same symptoms before she was diagnosed. What is the next most appropriate
investigation?

A. Refer for urgent mammogram
B. Refer for urgent USS
C. Blood hCG levels
D. Reassure and discharge
E. CT Head
A

C. Blood hCG levels

This is a bit of an irritating trick question, but illustrates a useful point: a beta-hCG can be done in the space of a GP appointment, and so there is no reason not to do one before referring this patient for imaging, as pregnancy could well be causing the breast changes.
That said, the second best option would be refer for urgent USS, as ultrasound is better for imaging younger, denser breast tissue.

How well did you know this?
1
Not at all
2
3
4
5
Perfectly
48
Q

BRCA gene mutations are associated with breast and
which other type of cancer?

A. Biliary
B. Uterine
C. Gastric
D. Ovarian
E. Colon
A

D. Ovarian

Other relevant cancer markers are:
Ca19-9: pancreatic cancer
Ca15-3:  breast cancer
Ca125: ovarian cancer
aFP: liver and testicular cancer
b-hCG: testicular cancer
CEA: colorectal cancer
How well did you know this?
1
Not at all
2
3
4
5
Perfectly
49
Q

A 35 year-old woman presents with breast pain and nipple discharge. The discharge is thick and green and the nipple appears retracted, she has recently started breast-feeding.

What is the most likely diagnosis?
A. Mastitis
B. Duct estasia
C. Abscess
D. Breast cancer
E. Paget's disease of the breast
A

B. Duct ectasia

Duct ectasia can mimic cancer (nipple retraction, pain, bloody discharge) but is a benign, self-limiting condition. You would probably advise the patient to use over the counter analgesia and wait.
Thick green discharge is most commonly caused by duct ectasia, and nipple retraction or inversion is also typical, allowing duct ectasia to be distinguished from an abscess or mastitis.

How well did you know this?
1
Not at all
2
3
4
5
Perfectly
50
Q

A 64 year old man presents with a lesion on his upper ear that has been present for months but has now begun to ulcerate. On examination: non-pigmented,
hyperkeratotic, crusty lesion with raised everted edges on the pinna.

What is the most likely diagnosis?
A. Basal call carcinoma
B. Malignant melanoma – superficial spreading type
C. Malignant melanoma – nodular type
D. Non-healing scab
E. Squamous cell carcinoma
A

E. Squamous cell carcinoma

SCC is causes by mutation of keratinocytes, and so are hyperkeratotic and non-pigmented. The greatest risk factor is UV light exposure but chronic inflammation (wound), a family history, immunosuppression, and acitinic keratosis (pre-cancerous crusty growths) are all risk factors.

SCC tends to present in older patients, is stereotypically situated on the pinna of the ear (but can occur anywhere - usually on the head), may ulcerate without healing, and has everted edges

SCC needs to be referred urgently to dermatology as it is invasive and can metastasise.

How well did you know this?
1
Not at all
2
3
4
5
Perfectly
51
Q

A 76-year-old woman has recently attended her GP because of a ‘spot that
won’t go away’. The lesion is on her nose and has rolled edges. The GP suspects a basal cell carcinoma.

A. What investigations are likely to be needed?
B. Dermatology referral/assessment - 2-week wait
C. Dermatology referral/assessment – routine
D. None – it is a non-dangerous chronic condition
E. None – GP to start treatment
F. Re-assessment in primary care periodically

A

C. Dermatology referral/assessment – routine

A basal cell carcinoma is slow-spreading and rarely metastasises. From the description there is no immediate risk (e.g. to the eye) so a routine referral will suffice.

How well did you know this?
1
Not at all
2
3
4
5
Perfectly
52
Q

A 32-year old caucasian scuba diver who lives in the Maldives had a seizure three days ago. He has no history of epilepsy but he’s had headaches for the past 5 months. The headaches are worse when he goes to bed. On examination, a dark irregular skin lesion is found on the back of his neck. An MRI scan shows multiple lesions across both cerebral hemispheres.

What is the most likely diagnosis?
A. Acoustic neuroma
B. Glioblastoma multiforme
C. Meningioma
D. Metastases
E. Neurofibromatosis type I
A

D. Metastases

The hints here are:
Lesion that should arouse suspicion of melanoma
History of significant sun exposure (Maldives, caucasian)
Headache characteristic of raised ICP (worse when lying down)

How well did you know this?
1
Not at all
2
3
4
5
Perfectly
53
Q

A 4-year old girl presents to the GP with multiple lesions on her face. The lesions are
raised and shiny, non-tender, non-erythematous, and 3 mm in diameter. They have an
umbilicated centre. The patient is known to be HIV positive.

What is the most likely diagnosis?
A. Chicken pox
B. Molluscum contagiosum
C. Atopic eczema
D. Eczema herpeticum
E. Herpes simplex virus
A

B. Molluscum contagiosum

Molluscum contagiosum is a viral skin infection generally seen in schoolchildren. Small pearly papules with a dip in the middle (umbilicated centre) are classic of molluscum contagiosum. The incidence is higher in HIV patients, and HIV patients will likley have more papules, buthealthy children can contract the virus too.
No treatment is necessary

How well did you know this?
1
Not at all
2
3
4
5
Perfectly
54
Q

A 52-year-old woman presents to the GP with redness and swelling of her right

cheek. On examination the area of erythema is well-demarcated and warm to the
touch. Her temperature is 37.9oC and she feels unwell.

i) Most likely diagnosis?
A. Cellulitis
B. Skin abscess
C. Erysipelas
D. Necrotising fasciitis
E. Gum infection
A

C. Erysipelas

The history of well-demarcated redness and swelling, particularly in the face, is suggestive of erysipelas - a form of bacterial (usually beta-heamolytic group A Strep) infection similar to cellulitis, but more superficial.
The location and clear boundaries are what separate erysipelas from cellulitis. In addition, erysipelas generally causes more systemic symptoms of illness (malaise, fever) which is unexpected as it is more superficial than cellulitis.

How well did you know this?
1
Not at all
2
3
4
5
Perfectly
55
Q

A 52-year-old woman presents to the GP with redness and swelling of her right

cheek. On examination the area of erythema is well-demarcated and warm to the
touch. Her temperature is 37.9oC and she feels unwell.

ii) Next steps?
A. Cold compress, reassure, home
B. Admit to intensive care unit
C. Take skin swabs, blood cultures, and
give paracetamol
D. Draw around the lesion, give pain
relief, oral fluids and antibiotics
A

D. Draw around the lesion, give pain
relief, oral fluids and antibiotics

The history of well-demarcated redness and swelling, particularly in the face, is suggestive of erysipelas - a form of Staphylococcus Aureus infection similar to cellulitis, but more superficial.
The location and clear boundaries are what separate erysipelas from cellulitis, though the treatment is actually the same: antibiotics for the infection, fluids for support, and draw around the lesion so you can monitor whether it is spreading easily.

How well did you know this?
1
Not at all
2
3
4
5
Perfectly
56
Q

A 12-year-old girl presents with dry, itchy skin that involves the flexures in front of her elbows and behind her knees. She has symptoms of hay fever and was diagnosed with egg and milk allergy at 6 months old. Her mother has asthma.

What is the most likely diagnosis?
A. Seborrheic dermatitis
B. Atopic dermatitis
C. Psoriasis (chronic plaque)
D. Psoriasis (guttate)
E. Urticaria
A

B. Atopic dermatitis

Atopis dermatitis (also called eczema) typically affects the flexor surfaces of the arms and legs (in contrast to psoriasis which affects the extensor surfaces) causing dry, itchy, red skin and is common in children.
The patient's PMHx of food allergies and hayfever increase the liklehood of her skin complaint having an allergic cause, as all these complaints are part of atopic march.
A family history of atopic disease (asthma) also increases the chance of this being atopic dermatitis.
How well did you know this?
1
Not at all
2
3
4
5
Perfectly
57
Q

An otherwise healthy 23-year-old man complains of sore red lesions on his extremities which have a central clearance (targetoid). These appeared after a recurrence of his ‘coldsores’.

What is the diagnosis?
A. Erythema multiforme
B. Chicken pox
C. Herpes simplex virus
D. Stevens-Johnson’s syndrome
E. Toxic epidermal necrolysis
A

A. Erythema multiforme

The description of red lesions with a central clearance is typical of erythema multiforme, which can occur after herpes simplex infections (cold-sores). It can also be triggered by sulphonamide drugs and penicillin, and is associated with blood cancers (leukaemia, lymphoma, myeloma), and autoimmune conditions (RhA, SLE, sarcoidosis)

How well did you know this?
1
Not at all
2
3
4
5
Perfectly
58
Q

A mother brings her 3 year-old baby boy into the GP practice because he has developed a facial rash. On inspection the rash consists of yellow greasy scales that are particularly prevalent around the mouth, nasolabial folds, and eyebrows.

What is the most likely diagnosis?
A. Psoriasis
B. Contact dermatitis
C. Eczema herpeticum
D. Atopic eczema
E. Sebhorreic eczema
A

E. Sebhorreic eczema

Sebhorreic eczema is fairly easy to identify because of the description of yellow greasy scales that particularly affect the nasolabial folds and eyebrows.
Sebhorreic eczema can cause dandruff and nappy rash, and is associated with malassaezia yeasts.

How well did you know this?
1
Not at all
2
3
4
5
Perfectly
59
Q

A 45 year-old man presents to A&E with severe leg pain. The skin of his left leg is dark and blistered, and he is feverish and tachycardic. He admits to i.v. drug use. A group A haemolytic streptococcus is cultured from a swab.

What is the most likely diagnosis?
A. Erysipelas
B. Necrotising fascitis
C. Erythema multiforme
D. Eczema herpeticum
E. Compartment syndrome
A

B. Necrotising fascitis

Necrotising fascitis is caused by a streptococcal infection of the deep fascia, just above the muscle. Risk factors include diabetes, malignancy, trauma, surgical wounds, and i.v. drug use, although around 50% of cases occur in previously healthy people.
Necrositing fascitis warrants an emergency surgical referral for debridement. Gas may be sene underneath the soft tissue on an x-ray or CT.

How well did you know this?
1
Not at all
2
3
4
5
Perfectly
60
Q

A 65 year-old man presents with blurred vision. He is taking ramipril, amlodipine, and bendroflumathaizide. Upon fundoscopy you see AV nipping, silver wiring, and cotton wool spots.

What grade of this condition does the patient have?
A. I
B. II
C. III
D. IV
A

C. III

The patient has hypertensive retinopathy, which is separated into 4 grades:
I - Twisting of the retinal arteries and silver wiring
II - AV nipping (crossing over of arteries and veins)
III - Flame haemorrhages and cotton wool spots
IV - Papilloedema

How well did you know this?
1
Not at all
2
3
4
5
Perfectly
61
Q

A middle-aged man presents to his GP with recent weight gain. On examination, his blood pressure is mildly raised and you notice several bruises on both his arms. U&Es reveal hypokalaemia.

What is the most appropriate investigation?
A. LFTs
B. HbA1c
C. Low dose dexamethasone suppression test
D. aFP
E. High dose dexamethasone suppression test
F. Synacthen test

A

C. Low dose dexamethasone suppression test

This history is suggestive of Cushing’s, which is confirmed with a low dose dexamethasone test. A high dose dexamethasone test is to determine whether the cause is Cushing’s disease or syndrome.
LFTs might also be done, in light of the bruising, but are not the most appropriate investigation.
aFP is a marker of liver and testicular cancer.
A synacthen test is used to diagnose Addison’s
HbA1c is used to monitor long-term glycaemic control

How well did you know this?
1
Not at all
2
3
4
5
Perfectly
62
Q

A 40 year-old woman presents to her GP with a 1 month history of episodes of sweating, headache, sudden mood changes, and palpitations. In between episodes she feels completely normal. She is noted to be unusually tall with long limbs. She mentions that her mother had part of her thyroid removed when she was young but doesn’t remember why.

What is the most appropriate investigation?
A. Thyroid function tests
B. 24 hour urinary VMA
C. ECG
D. Echocardiogram
E. Oestrogen levels
F. U&amp;Es
A

B. 24 hour urinary VMA

This is a description of a phaeochromocytoma. They are very rare, but Imperial love them so it’s important to know this.
Episodes of agitation, headache, palpitations, and increased sweating are a common presentation of phaeochromocytoma. Blood pressure is very elevated during these episodes, but may be relatively normal inbetween.
VMA is the end metabolite of catecholamines, and an increased urinary level indicates episodes of high catecholamine release.
Phaeochromocytomas can feature as part of MEN2 (multiple endocrine neoplasia) along with medullary thyroid cancer, and mucosal gastrointestinal neuromas. The clues in the history are the description of the patient as tall with long limbs - this is hinting at Marfan’s syndrome which is associated with MEN2B - and the mother’s thyroid removal - hinting at thyroid medullary cancer which is a feature of MEN2B.

How well did you know this?
1
Not at all
2
3
4
5
Perfectly
63
Q

A 20 year-old woman presents to her GP with low mood. on further questioning she admits to recent weight gain, and says that her periods have been infrequent and irregular. On inspection she has more facial hair than you would expect, patches of dark velvety skin, and acne. She states that all of these change started around the same time, 1 month ago.

What is the most likely diagnosis?
A. Polycystic kidney disease
B. Cushing's syndrome
C. Polycystic ovary syndrome
D. Hypothyroidism
E. Obesity
A

C. Polycystic overy syndrome

Polycystic ovary syndrome is caused by excess release of androgens, in turn caused by either:
High levels of LH release from the adenohypophysis
Chronically elevated insulin levels.
The androgens cause the symptoms of hirsutism, acne, acanthosis nigricans (dark velvety skin) and centripetal weight gain. PCOS is inherited in an autosomal dominant way, but the expression of the disease in women is variable.

Whilst some of these symptoms are associated with obesity (hirsutism, oligomenorrhea, acne), the onset is too quick, and it is unusual to experience all of these symptoms together.

The ‘cysts’ seen on ultrasound are actually immature egg follicles that have started to develop, but then stopped because of altered ovarian function.

How well did you know this?
1
Not at all
2
3
4
5
Perfectly
64
Q

A 70 year-old man presents with back pain. Upon further questioning he reveals he has been constipated, but has been urinating more frequently than usual, and drinking more water. He has also felt tired and occasionally nauseous.

What is the most likely diagnosis?
A. Cerebral space-occupying lesion
B. Colon carcinoma
C. Type II diabetes mellitus
D. Multiple myeloma
E. Hypothyroidism
A

D. Multiple myeloma

Multiple myeloma is a cancer of plasma cells (mature B-cells) that reside in the bone marrow. The effects of MM can be remembered using CRAB:

C - calcium: calcium is released from bone as it is broken down by the cancer, increasing serum calcium. This leads to polyuria, polydipsia, constipation, confusion, kidney stones, and altered mental status (stones, abdominal moans, and psychic groans).

R - renal: malignant plasma cells secrete malformed proteins which damage the kidneys causing kidney failure. This in turn may lead to uraemia, causing nausea, and electrolyte imbalances e.g. hyperkalaemia, which can cause cardiac arrhythmia. In addition to the effect of proteins, the elevated serum calcium leads to calcium deposition in the kidneys, worsening the renal failure. These proteins will also cause rouleaux formation on a blood film.

A - anaemia: infiltration of healthy bone marrow by malignant plasma cells causes a decrease in RBC output. Other cell lineages will also be affected, including white cells, meaning that people with MM are more vulnerable to infection.

B - bone: bone pain is a very common complaint in MM and helps to differentiate the other symptoms from other causes. In MM the malignancy can spread to the extent where it compresses the spinal cord, causing radicular pain, bladder and bowel incontinence, loss of sensation, and paraplegia.

How well did you know this?
1
Not at all
2
3
4
5
Perfectly
65
Q

A 40 year-old man is brought to A&E by ambulance after a car accident. He is conscious but in considerable pain and short of breath. There is extensive trauma to the chest. His JVP is raised and increases on inspiration, but you are unsure if heart sounds are normal as they are hard to hear. HR is 130, BP is 80/50.

What is the most likely diagnosis?
A. Ruptured aorta
B. Tension pneumothorax
C. Pulmonary haemorrhage
D. Hepatic haemorrhage
E. Cardiac tamponade
A

E. Cardiac tamponade

The clue here is the presence of Beck’s triad: decreased arterial pressure, muffled heart sounds, and a raised JVP. The JVP will also exhibit a paradoxical rise (Kussmaul sign) which is seen in any cause of right ventricular dysfunction.

How well did you know this?
1
Not at all
2
3
4
5
Perfectly
66
Q

A 70 year-old man presents to A&E with central crushing chest pain radiating to the right arm and jaw. ST-elevation is seen on an ECG in leads V1-V4. The patient states his symptoms began 5 hours ago, and since then he has been given oxygen, diamorphine, aspirin, clopidogrel, and nitrates.

What is the most appropriate next step in his management?
A. Thrombolysis with alteplase
B. Propranolol, ramipril, and epleronone
C. Percutaneous coronary intervention
D. Fondaparinux and a LMWH
E. I.V. amiodarone
A

C. Percutaneous coronary intervention

This is a description of a STEMI
Thrombolysis with alteplase is avoided if PCI is possible, though if the wait for PCI is more than 2 hours, and the patient is still within the 12 hour window from when they first experienced symptoms, thrombolysis may be attempted.
B describes the long-term treatment for a STEMI
D describes two anti-coagulants used in various conditions
E is used for certain arrhythmias and in cardiac arrest

How well did you know this?
1
Not at all
2
3
4
5
Perfectly
67
Q

An 80 year-old man is brought to see the GP from a nursing home with worsening mental status and ataxia. Upon questioning of his nurse, you find out he has been experiencing incontinence.

What is the most likely diagnosis
A. Alzheimer's disease
B. UTI
C. Glioblastoma
D. Normal pressure hydrocephalus
E. Spinal cord compression
A

D. Normal pressure hydrocephalus

Normal pressure hydrocephalus is an abnormal accumulation of CSF within the ventricles, it is idiopathic.

When an elderly patient presents with a triad of ataxia, dementia, and urinary incontinence, normal pressure hydrocephalus should be a prominent differential.

How well did you know this?
1
Not at all
2
3
4
5
Perfectly
68
Q

A 30 year-old afro-carribean woman with diagnosed SLE presents with pain and swelling of her left calf. This has happened twice before, and both were confirmed to be DVTs. Upon further questioning she was pregnant 5 months ago, but developed pre-eclampsia and ultimately miscarried. Blood tests show a low platelet count. She states that her symptoms have been well controlled for the past few years.

What is the most likely diagnosis?
A. Acute lymphoblastic leukaemia
B. Effects of SLE
C. Disseminated intravascular coagulation
D. Antiphospholipid syndrome
E. Acute myeloid leukaemia
A

D. Antiphospholipid syndrome

Antiphospholipid syndrome is an autoimmune hypercoaguble state caused by antibodies directed at certain proteins (lupus anticoagulant and cardiolipin) causing dysfunction of Protein C (coagulation cascade regulator) and increased cleavage of prothrombin to thrombin, increasing coagulation.

The incidence of antiphospholipid syndrome is increased in people with SLE.

This syndrome is particularly prominent during pregnancy, where it may cause miscarriage, stillbirth, intra-uterine growth restriction, pre-eclampsia, and pre-term delivery.

How well did you know this?
1
Not at all
2
3
4
5
Perfectly
69
Q

A 12 year-old boy presents with unusual bruising, a persistent chest infection, and fatigue over the course of a month. He was recently given a course of anti-viral drugs for hepatitis B infection.

What is the most likely diagnosis?
A. Hepatocellular carcinoma
B. Adverse drug reaction
C. Aplastic anaemia
D. Acute lymphoblastic leukaemia
E. Acute myeloid leukaemia
A

C. Aplastic anaemia

Aplastic anaemia results from damage to the haematopoeitic stem cells in the bone marrow, depleting the marrow’s ability to produce cells of all lineages. It can be idiopathic, but in this case was triggered by hepatitis B infection. Certain drugs can also trigger aplastic anaemia but it is very rare.

Thrombocytopenia - easy bruising
Leukopenia - persistent infections
Anaemia - malaise, fatigue, pallor, breathlessness, palpitations

Though pancytopenia could be caused by ALL, and it does affect children, this boy is a little older than most children are when they develop the disease, and he has reported no FLAWS symptoms.

How well did you know this?
1
Not at all
2
3
4
5
Perfectly
70
Q

A 6 year-old girl is brought into A&E with bloody diarrhoea and blood in her urine. Her father says it started 6 days ago but the GP told them not to worry and to keep her hydrated, but she has now become confused and nauseous. She is admitted and monitored closely. Her urine output is noted to be low and it is noticed that she has developed several large bruises on her body. Blood tests reveal raised LDH.

What is the most likely causative organism?
A. Streptococcus pneumoniae
B. Neisseria meningitidis
C. Salmonella enterica
D. Escherichia coli
E. Parvovirus B19
A

D. Escherichia coli

This is a case of hemolytic-uraemic syndrome which occurs most often in children. It’s caused by Shiga-like toxin-producing bacteria, most commonly EHEC (enterohaemhorragic Escherichia coli). HUS is usually preceded by bloody diarrhoea by roughly a week.

HUS is defined by a triad of uraemia, haemolytic anaemia, and thrombocytopenia.

The Shiga toxin activates the complement pathway, causing inflammation and thrombosis which tends to affect the kidneys most commonly, and most seriously. The thrombi also cause the haemolytic anaemia through microangiopathic haemolysis - they damage the endothelium of blood vessels and narrow the lumen, causing RBCs to be sheared and destroyed as they pass.

How well did you know this?
1
Not at all
2
3
4
5
Perfectly
71
Q

A 6 year-old girl is brought to the GP by her mother because she has noticed bruises on her arms that she can’t explain. The mother also thinks her child has seemed more tired recently. O/E the child has an enlarged spleen.

What is the most likely diagnosis?
A. Hodgkin's lymphoma
B. Acute myeloid leukaemia
C. Acute lymphoblastic leukaemia
D. Non-Hodgkin's lymphoma
E. Haemophilia A
F. Haemophilia C
A

C. Acute lymphoblastic leukaemia

ALL is usually occurs in young children, and is the most common leukaemia in children.
AML, though it does occur in children, is more common in the elderly and reaches it peak incidence between 75 and 80.
Haemophilia would explain the bruising, but not the fatigue. Splenomegaly may have some association with haemophilia but it is not particularly well defined.

How well did you know this?
1
Not at all
2
3
4
5
Perfectly
72
Q

An 18 year-old visits his GP asking for antibiotics. He has been feeling unwell for the past two weeks. O/E he has raised cervical lymph nodes. He reports an itchy feeling recently, but puts this down to the large amounts of alcohol he has been consuming, but he says he has now stopped.

What is the most likely diagnosis?
A. Hodgkin's lymphoma
B. Acute myeloid leukaemia
C. Acute lymphoblastic leukaemia
D. Non-Hodgkin's lymphoma
E. Haemophilia A
F. Haemophilia C
A

A. Hodgkin’s lymphoma

In theory, Hodgkin’s lymphoma more commonly causes B-symptoms (systems of systemic illness e.g. FLAWS) whereas non-Hodgkin’s lymphoma more usually features with infiltratve symptoms.

In reality, a better way to differentiate the two is that Hodgkin’s lymphoma is rarer than non-Hodgkin’s and can occur in younger people, whereas non-Hodgkin’s lymphoma rarely occurs in younger people (peak incidence is 70).

Reed-Sternberg cells will be seen on the blood film of somebody with Hodgkin’s lymphoma: if the question mentions these as being present, it is definitely Hodgkin’s lymphoma.

The risk of both Hodgkin’s and non-Hodgkin’s lymphoma is increased in those who have previously had glandular fever.

How well did you know this?
1
Not at all
2
3
4
5
Perfectly
73
Q

A 13 year-old girl is brought in by her father to see the GP because of a sore throat that began 2 days ago. O/E she is feverish, and a grey exudate can be seen on her tonsils. She denies any cough.

What score would you use to assess the cause of her sore throat?
A. CHA2DS2-VASc
B. CENTOR
C. CURB65
D. Wells score
E. Duke criteria

What is the cause?

A

B. CENTOR

This is bacterial pharyngitis (most likely Strep), as 3 or more of the CENTOR score elements are present, making a viral cause unlikely.

C - cough absent
E - exudate on, or swelling of, the tonsils
N - nodes, anterior cervical lymph nodes swollen or tender
T - temperature of over 38 degrees
OR - young or old modifying factors (under 15 add 1 point, over 44 subtract 1 point)

CHA2DS2-VASc score is used to estimate the risk of stroke in atrial fibrillation.
CURB65 score estimates mortality and guides treatment in pneumonia
Wells score estimates the chance of a patient having had a PE based on their symptoms
The duke criteria is used to diagnose infective endocarditis (the Duke staging system is used for colon cancer)

How well did you know this?
1
Not at all
2
3
4
5
Perfectly
74
Q

A 28 year-old man presents with a scrotal lump. Upon palpation you cannot get above the lump, and it is reducible. You reduce the lump and hold your thumb over the inguinal mid-point and ask the patient to cough. The lump does not reappear.

What is the most likely diagnosis?
A. Direct inguinal hernia
B. Indirect inguinal hernia
C. Variocele
D. Hydrocele
E. Testicular torsion
A

B. Indirect inguinal hernia

Indirect inguinal hernias occur as a segment of bowel herniates through the deep inguinal ring. This is a result of an embryonic failure of the process vaginalis to close.

A direct hernia occurs when bowel herniates through a weakpoint in the transversalis fascia. The weakpoing is within Hesslebach’s triangle which is bordered laterally by the inferior epigastric artery, medially by the linea semilunaris (edge of the rectus abdominis), and inferiorly by the inguinal ligament.

To distinguish between the two, the hernia is reduced (pushed back into the abdomen) and the doctor places their thumb over the inguinal midpoint and asks the patient to cough. If the hernia reapperas, it is a direct hernia, and if not it is an indirect hernia.

NB: the key feature of a scrotal lump that indicates it is a hernia, is that the dictor is unable to ‘get above it’

How well did you know this?
1
Not at all
2
3
4
5
Perfectly
75
Q

A 35 year old woman presents saying that sex with her husband has now become painful and difficult. She also finds it difficult to swallow and cannot taste well anymore. With further investigations, antinuclear antibodies are present.

Please select the best test to confirm your diagnosis:
A. Schirmer’s Test
B. Erythrocyte Sedimentation Rate
C. Anticentromere antibodies
D. Chest X-Ray
E. Antinuclear Antibodies
A

A. Schirmer’s Test

Schirmer’s test is used to monitor tear production, and is an investigation for Sjogren’s syndrome. Sjogren’s is an autoimmune condition affecting the exocrine glands of the body, and is most common in middle aged women. Patients may report dry eyes, dry mouth (with associated dysphagia and loss of taste), painful sex (decreased lubrication in women), dry cough, hair loss, and dry skin.
The main antibodies seen in Sjogren’s are anti-Rho (SS-A) and anti-La (SS-B).
Sjogren’s is associated with other autoimmune conditions (primary biliary cirrhosis, RhA, SLE) and increases the risk of non-Hodgkin’s lymphoma. Extra-glandular manifestations include interstitial nephritis, Raynaud’s phenomenon, vasculitis, lymphadenopathy, and hepatobiliary disease.

Antibodies to know:
pANCA: primary sclerosing cholangitis, Churg-Strauss syndrome, microscopic polyangitis
cANCA: Polyangitis with granulomatosis
Anti-mitochondrial: primary biliary cirrhosis
Anti-Scl70: diffuse scleroderma
Anti-centromere: CREST syndrome
Anti-Rho and Anti-La: Sjogren’s syndrome
Anti-dsDNA: SLE
Anti-Smith: SLE
Anti-CCP: Rheumatoid arthritis
Anti-glomeruler basement membrane: Goodpasture’s syndrome
Anti-smooth muscle: autoimmune hepatitis
Lupus anticoagulant: antiphospholipid syndrome
Anti-cardiolipin: antiphospholipid syndrome

How well did you know this?
1
Not at all
2
3
4
5
Perfectly
76
Q

A 40 year old African man who has a history of interstital lung fibrosis, uveitis, parotitis, and erythema nodosum.

Please select the best test to confirm your diagnosis:
A. Schirmer’s Test
B. Erythrocyte Sedimentation Rate
C. Anticentromere antibodies
D. Chest X-ray
E. Antinuclear Antibodies
A

D. Chest X-ray

This is a description of sarcoidosis, a condition causing non-caseating granulomas which most typically affect the lungs, but can affect any organ. Sarcoidosis causes lung fibrosis which can be seen on an x-ray, along with bilateral hilar lymphadenopathy. Sarcoidosis also causes raised ACE and calcium serum levels.
Signs of sarcoidosis include erythema nodosum, uveitis, lupus pernio, a history of dry cough and weight loss, parotid gland swelling, and diabetes insipidus.

How well did you know this?
1
Not at all
2
3
4
5
Perfectly
77
Q

A 60 year old man presents to his GP having since Christmas had telangiectasia, calcified spots on his skin, dysphagia and extremely white fingertips in cold weather.

Please select the best test to confirm your diagnosis:
A. Schirmer’s Test
B. Erythrocyte Sedimentation Rate
C. Anticentromere antibodies
D. Chest X-Ray
E. Antinuclear Antibodies
A

C. Anti-centromere antibodies

ACA are an antibody frequently seen in CREST syndrome (a type of scleroderma), but less frequently in the diffuse variant of the disease.
Anti-scl70 is seen more in diffuse scleroderma than CRESt, but is not very sensitive.
CREST is the limited form of the disease, which typically affects the hands, face, and fingers. Diffuse scleroderma may affect the heart, lungs, kidneys, and GI tract in addition.
CREST is an acronym which refers to the features that define the disease:
Calcification
Raynaud’s phenomenon
Esophageal dysmotility
Sclerodactyl (thickening and ulceration of the skin of the fingers)
Telangiectasia

Antibodies to know:
pANCA: primary sclerosing cholangitis, Churg-Strauss syndrome, microscopic polyangitis
cANCA: Polyangitis with granulomatosis
Anti-mitochondrial: primary biliary cirrhosis
Anti-Scl70: diffuse scleroderma
Anti-centromere: CREST syndrome
Anti-Rho and Anti-La: Sjogren’s syndrome
Anti-dsDNA: SLE
Anti-Smith: SLE
Anti-CCP: Rheumatoid arthritis
Anti-glomeruler basement membrane: Goodpasture’s syndrome
Anti-smooth muscle: autoimmune hepatitis
Lupus anticoagulant: antiphospholipid syndrome
Anti-cardiolipin: antiphospholipid syndrome

How well did you know this?
1
Not at all
2
3
4
5
Perfectly
78
Q

A 19 year old woman presents to your clinic with with discoid rashes, a malar rash and mouth ulcers.

Please select the best test to confirm your diagnosis:
A. Schirmer’s Test
B. Erythrocyte Sedimentation Rate
C. Anticentromere antibodies
D. Chest X-Ray
E. Antinuclear Antibodies
A

E. Antinuclear Antibodies

These skin symptoms are indicative of systemic lupus erythematosus.

Anti-nuclear antibodies are present in multiple autoimmune diseases: the two important ones in SLE are anti-dsDNA (highly specific for SLE) and anti-Smith.

Blood tests sometimes reveal anaemia of chronic disease, thrombocytopenia, and leukopenia, and often reveal increased ESR.

Antiphospholipid syndrome is strongly associated with SLE, and so associated antibodies are often seen in SLE.

SLE, like other autoimmune conditions, can affect most organs, but typically affect the skin, muscles, and joints causing arthritis, oral ulcers, rashes, and serositis.

Antibodies to know:
pANCA: primary sclerosing cholangitis, Churg-Strauss syndrome, microscopic polyangitis
cANCA: Polyangitis with granulomatosis
Anti-mitochondrial: primary biliary cirrhosis
Anti-Scl70: diffuse scleroderma
Anti-centromere: CREST syndrome
Anti-Rho and Anti-La: Sjogren’s syndrome
Anti-dsDNA: SLE
Anti-Smith: SLE
Anti-CCP: Rheumatoid arthritis
Anti-glomeruler basement membrane: Goodpasture’s syndrome
Anti-smooth muscle: autoimmune hepatitis
Lupus anticoagulant: antiphospholipid syndrome
Anti-cardiolipin: antiphospholipid syndrome

How well did you know this?
1
Not at all
2
3
4
5
Perfectly
79
Q

A 8 year-old boy is brought in by his mother because she has noticed large bruises on him which she cannot figure the cause of. O/E the boy’s left knee is swollen and painful, but is not hot or red. There are no signs of systemic illness.

Which blood test result would confirm the most likely diagnosis?
A. Prolonged PT
B. Increased INR
C. Prolonged aPTT
D. Decreased INR
E. Vitamin K deficiency
A

C. Prolonged aPTT

Haemarthrosis (bleeding into the joint) is indicative of haemophilia. You can’t tell from this history which type of haemophilia the patient has (though A is 6 times more common than B) but the blood test result would be the same for any type of haemophilia because all the different clotting factors that can deficient in haemophilia are involved in the intrinsic pathway, which is tested with an aPTT test.
PT tests the extrinsic pathway, and INR is just a ratio of patient’s PT over normal PT.
Vitamin K is needed to activate factors II, VII, IX, and X to allow them to bind to endothelium, and so vitamin K deficiency would cause prolonged aPTT and PT as it affects factors in both the intrinsic and extrinsic pathways.

How well did you know this?
1
Not at all
2
3
4
5
Perfectly
80
Q

A 50 year-old man presents with excruciating pain in his big toe that woke him from sleep this morning and has become progressively worse. He is obese and admits to drinking a relatively large amount of beer, though he says his diet is good because he frequently eats meat. He has several hard swellings on his ear. Synovial fluid is aspirated and examined, and he is started on allopurinol.

What would most likely be seen on examination of his synovial fluid?
A. Crystals that turn deep blue with an H&E stain
B. Raised WCC in the fluid
C. Rhomboid crystals with a positive birefrigence seen under polarised microscopy
D. Needle-shaped crystals with a negative birefringence seen under polarised microscopy
E. Needle-shaped crystals with a positive birefringence seen under polarised microscopy

A

D. Needle-shaped crystals with a negative birefringence

This history describes gout: elevated uric acid levels in the blood leading to uric crystals being deposited in the joints. Typically the metacarpo-phalangeal joint of the big toe is affected (this is called podagra). The swellings on the ear are tophi - deposits of uric crystals that are pathognomic of gout.
Risk factors include obesity, alcohol intake, and high red meat intake, along with genetic susceptibility.
A and C are both features of pseudo-gout, which involves calcium rather than uric acid.

How well did you know this?
1
Not at all
2
3
4
5
Perfectly
81
Q

A 47-year-old man visits his GP having developed a skin rash. He has been feeling ‘generally unwell’ for the past 3 months, and has suffered from abdominal pain accompanied by some rectal bleeding. An angiogram reveals ‘Rosary sign’. He regularly attends hospital for check-ups since he was diagnosed with chronic hepatitis B, 2 years ago.

What is the most likely diagnosis?
A. Giant cell arteritis 
B. Dermatomyositis 
C. Polyarteritis nodosa 
D. Granulomatosis with polyangiitis 
E. Behcet’s disease
A

C. Polyarteritis nodosa

Polyarteritis nodosa is an inflammatory disease of small and medium-sized blood vessels that often affects the kidneys, but generally spares the lungs. It causes multiple aneurysms to form along the same blood vessel causing a ‘rosary bead’ appearance on an angiogram. PN is associates with hepatitis B.

How well did you know this?
1
Not at all
2
3
4
5
Perfectly
82
Q

An 18 year-old boy presented with breathlessness that came on while he was playing football with his friends on a chilly winter’s day. He stated that this breathlessness had occurred before and had woken him up at night once. The boy is diagnosed with asthma and given salbutamol and beclamethasone. He returns three months later and says that he is having to use the salbutamol three times a week.

What is the most appropriate next treatment?
A. Add another inhaled corticosteroid
B. Add a long-acting beta agonist
C. Increase the inhaled corticosteroid dose
D. Leukotriene receptor antagonist
E. Add a long-acting muscarinic antagonist

A

D. Leukotriene receptor antagonist

Guidelines:

1) For patients who have infrequent short-lived wheezes with normal lung function, consider a short-acting beta agonist with no additional therapy.
2) For patients with frequent episodes or asthma uncontrolled by SABA, prescribe a low-dose inhaled corticosteroid
3) If the low-dose ICS does not control symptoms, add a leukotriene receptor antagonist
4) If combined maintenance therapy with ICS and LRA does not manage symptoms, add a long-acting beta agonist and consider discontinuing LRA.
5) If ICS and LABA (with or without LRA) fails to control symptoms, switch the patient to MART (combined ICS and LABA inhaler)
6) If MART does not control symptoms, increase the ICS dose to a moderate dose, either as part of MART, or as a fixed dose regimen with a LABA, and a SABA to use when required.
7) If moderate dose ICS and LABA does not control symptoms, increase the dose to high dose ICS on a fixed schedule with a SABA when needed. Alternatively carry on as in step 6, but add a long-acting muscarinic antagonist or theophylline

How well did you know this?
1
Not at all
2
3
4
5
Perfectly
83
Q

A 74 year-old man is brought to A&E with left-sided abdominal pain and vomiting. His abdomen is distended and upon questioning he has not opened his bowels for two days. Bowel sounds are absent. He has no significant PMHx apart from a laparotomy two years ago. An abdominal x-ray without contrast is performed.

What is the AXR most likely to show?
A. Coffee bean sign
B. Thickened haustral folds
C. Transverse colon dilated >6cm
D. A colon tumour
E. Bird's beak sign
A

A. Coffee bean sign

This history is suggestive of a sigmoid volvulus - where a loop of bowel twists around on the axis of its mesentery. The classic radiological sign of a sigmoid volvulus is the coffee bean sign on AXR. A bird’s beak sign may also be seen, but only with a contrast enema.

How well did you know this?
1
Not at all
2
3
4
5
Perfectly
84
Q

A 37 year-old woman presents to her GP as she has been feeling very tired over the past couple of days and has noticed unusual bruises on her body, along with bleeding form her nose and gums. PMHx includes a diagnosis of SLE. O/E she seems feverish and jaundiced. U&Es show elevated creatinine.

What is the most likely diagnosis?
A. Microangiopathic haemolytic anaemia
B. Haemolytic-uraemic syndrome
C. Thrombotic thrombocytopenic purpura
D. Hodgkin's lymphoma
E. Aplastic crisis
A

C. Thrombotic thrombocytopenic purpura

TTP is typically associated with antibodies against an enzyme that cleaves Von-Willebrand factor, leading to an abnormally high amount of VWf in the blood. This causes clotting in small blood vessels, and a consequent thrombocytopenia leading to bleeding.

Microangiopathic haemolytic anaemia is a feature of TTP which is caused by damage to RBCs as they pass through vessels that have been occluded by thrombi. This causes the RBCs to shear, resulting in a haemolytic anaemia, which may also cause s pre-hepatic jaundice.

Haemolytic-uraemic syndrome may present similarly to TTP, but usually occurs in children and will feature a history of gastrointestinal infection.

The classic pentad of TTP rarely occurs together but is as follows:
Fever
Microangiopathic haemolytic anaemia
Mental state changes
Renal impairment
Thrombocytopenia
How well did you know this?
1
Not at all
2
3
4
5
Perfectly
85
Q

A 29 year-old man presents to his GP with pain in his right knee. He has also been experiencing some hip stiffness and pain. You notice that his eyes seem bloodshot. The patient admits to a prolific sexual history and some recent discharge from his penis which has also developed a lesion on the glans.

What is the most likely diagnosis?
A. Behcet's syndrome
B. Rheumatoid arthritis
C. Reactive arthritis
D. Sarcoidosis
E. Sjogren's syndrome
A

C. Reactive arthritis

Defined as a triad of urethritis, conjunctivitis, and arthritis following infection, reactive arthritis is a non-septic, autoimmune reaction causing asymmetrical oligoarthritis of the lower limbs. It is associated with HLA B27 and is triggered by genitourinary (chlamydia) or gastrointestinal infection (campylobacter, salmonella, shigella).

Arthritis usually follows between 1-4 weeks after infection and includes extra-articular manifestations: enthesitis (plantar fascitis, dactylitis, achilles tendonitis), conjunctivitis, and skin inflammation (circinate balantitis - lesions on the glans, and keratoderma blenorrhagicum - a psoriasis-like rash on the palms and soles of the feet))

How well did you know this?
1
Not at all
2
3
4
5
Perfectly
86
Q

A mother brings in her 5 year old child into your clinic, saying they have been tired, irritable, and feverish over the past few weeks. On examination they have hepatomegaly, splenomegaly, and enlarged anterior cervical lymph nodes. You also notice testicular swelling, and petechiae on the limbs on examination.

What is the most likely diagnosis?
A. Acute Lymphoblastic Leukaemia
B. Acute Myeloid Leukaemia
C. Chronic Lymphoid Leukaemia
D. Chronic Myelogenous Leukaemia
E. Hodgkin’s Lymphoma
F. Non-Hodgkin’s Lymphoma
G. Multiple Myeloma
A

A. Acute Lymphoblastic Leukaemia

ALL is the most common cancer from the options in young children, and is an aggressive cancer, though it does respond well to treatment.
ALL causes anaemia (tiredness) and thrombocytopenia (petechiae) through ‘crowding out’ of healthy cell production in the bone marrow.
Testicular and lymph node swelling are both seen in ALL, and testicular infiltration can be problematic, as chemotherapy does not target cancerous cells in the testes particularly well.

How well did you know this?
1
Not at all
2
3
4
5
Perfectly
87
Q

Which of these fulfils the diagnostic criteria of Type 2 Diabetes?

A. Polyuria and polydipsia
B. Asymptomatic, single fasting plasma glucose >6.9 mmol/L
C. Asymptomatic, single reading of elevated HbA1c
D. Symptomatic, single fasting plasma glucose> 6.9 mmol/L
E. Symptomatic, single random plasma glucose <11.1 mmol/L

A

D. Symptomatic, single fasting plasma glucose> 6.9 mmol/L

To diagnose diabetes you need a random blood glucose >11.1 or a fasting blood glucose >6.9. If the patient is symptomatic, only one measurement is needed, but if the patient is asymptomatic, two separate readings on different days are needed.

How well did you know this?
1
Not at all
2
3
4
5
Perfectly
88
Q

A 19 year old male presented to A&E with abdominal pain, nausea and vomiting. He reports nocturia and examination reveals Kussmaul breathing. Capillary glucose demonstrates hyperglycaemia.

Which of these would be the first step in this patient’s managing?
A. IV Insulin
B. Oral Glucose
C. Diet and Exercise 
D. Fluid Replacement 
E. Glibenclamide
A

D. Fluid Replacement

This patient is in diabetic ketoacidosis. Ketoacidosis is usually accompanied by dehydration, so the most important first step is to administer i.v. fluids. This is initially saline and potassium chloride (hypokalaemia is a dangerous complication of DKA as potassium is lost in the urine). Insulin is also given (glucose is added when the blood sugar drops below 14mmol).

How well did you know this?
1
Not at all
2
3
4
5
Perfectly
89
Q

A 26 year old lady visits her GP complaining she is going to the toilet constantly, getting up several times in the night. Urinalysis is negative for glucose and ketones. A fluid deprivation test demonstrates urine osmolality of 200 mOsmol/kg, which rises slightly to 250 mOsmol/kg following administration of desmopressin. Her medical history includes a diagnosis of bipolar disorder.

What should be the GP’s next step?
A. Regular desmopressin
B. Fluid restriction
C. Review medication
D. Prescribe an NSAID
E. Insulin injections
A

C. Review medication

Lithium is used to treat diabetes and can cause nephrogenic diabetes insipidus.
Treatment for nephrogenic is medication review, thiazides, NSAIDs, and sometimes high dose desmopressin

How well did you know this?
1
Not at all
2
3
4
5
Perfectly
90
Q

A patient presents with chest pain that is triggered by exercise, but does not occur at rest. A CT coronary angiogram shows stenosed coronary arteries. The patient is diagnosed with stable angina.

What is the first line treatment for symptomatic control of angina?
A. ACEi or ARB
B. CCB or thiazide-like diuretics
C. Alpha blocker and a beta blocker
D. CCB or beta blocker
E. CCB and a statin
A

D. CCB or beta blocker

Guidelines:
Step 1: CCB or beta blocker
Step 2: if one is ineffective, either switch to the other one, or use both

If CCB and beta blockers are both contraindicated, or if the other drug is contraindicated at step 2, use one of the following instead:
Nicorandil
Ivabradine
Long-acting nitrates
Ranolzaine

Aspirin and statins are also used for prevention of further cardiovascular disease

How well did you know this?
1
Not at all
2
3
4
5
Perfectly
91
Q

A patient comes to the GP for a review of her blood pressure. You perform fundoscopy and notice cotton wool spots that were not present at her last appointment. Blood pressure is 145/95. The patient is currently on ramipril, amlodipine, and indapamide.

Which drug should be added to control this patient's hypertension?
A. Propranolol
B. Bendroflumathiazide
C. Spironolactone
D. GTN
E. Furosemide
A

C. Spironolactone

Guidelines:
Step 1: ACEi or ARB
Step 2: ACEi with CCB
Step 3: ACEi with CCB and thiazide-like diuretic (e.g. indapamide)
Step 4: ACEi with CCB and thiazide-like diuretic (e.g. indapamide) and spironolactone

If step 4 is ineffective or spironolactone is contraindicated, alpha or beta blockers may be used.

The exception to these guidelines is patients who are Afro-Carribean or over the age of 55, in which case a thiazide-like diuretic or CCB is indicated for step 1.

If ACEi can’t be tolerated (e.g. due to cough) an ARB is an acceptable alternative, but the two are not used together.

How well did you know this?
1
Not at all
2
3
4
5
Perfectly
92
Q

A woman comes in with a 2 month history of weight loss, diarrhoea, and abdominal pain. A colonoscopy with biopsy is ordered, and from the results she is diagnosed with Crohn’s disease.

What would the results show?
A. Cobblestoning of the terminal ileum with rose thorn ulcers
B. Mucosal ulcers, depletion of goblet cells, and crypt abscesses
C. Transmural non-caseating granulamtous inflammation
D. Lead pipe mucosa
E. Contiguous inflammation from the rectum to the distal segment of the transverse colon

A

C. Transmural non-caseating granulamtous inflammation

Cobblestoning of the terminal ileum may be seen on endoscopy, but rose thorn ulcers are seen on a barium follow through.
Mucosal ulcers, depletion of goblet cells, and crypt abscesses are signs of UC seen on a biopsy.
Lead pipe mucosa is a sign of UC seen in a contrast barium enema, where chronic UC destroys the normal architecture of the colon.
Continuous inflammation from the rectum to the distal segment of the transverse colon is more indicative of UC, which particularly affects the rectum and spreads in a continuous fashion proximally. By contrast, Crohn’s can affect any part of the GIT and often causes skip lesions.

How well did you know this?
1
Not at all
2
3
4
5
Perfectly
93
Q

A 40 year-old woman presents with a 1 month history of abdominal pain, bloating, weight loss, and foul-smelling pale diarrhoea. You notice a vesicular rash on her elbows. Blood tests show a microcytic anaemia.

What would be the most appropriate next set of investigations?
A. Colonoscopy
B. TTG ab
C. OGD
D. Duodenal biopsy
E. Stool culture
A

B. TTG ab

Antibodies against TTG are a sensitive and specific test for coeliac disease, which is described here. Biopsy is the gold standard and so would ultimately be needed to confirm the diagnosis, but TTGab is a less invasive initial test.
Coeliac disease is an autoimmune reaction triggered bu ingestion of gluten. The autoantibodies produced attack the villi of the small intestine, causing villous atrophy that can be seen on a duodenal biopsy.

The vesicular rash described is dermatitis herpetiformis - a rash resembling herpes that is associated with coeliac disease.

How well did you know this?
1
Not at all
2
3
4
5
Perfectly
94
Q

A blood sample taken from a patient shows the following:
HBsAg+, HBeAg+, and HBcAb IgG+

What is the Hepatitis B status of this patient?
A. Acute infection
B. Cleared infection
C. Chronic infection
D. Vaccinated
E. Not exposed
A

C. Chronic infection

IgG is the most useful marker here:
If IgG is present with IgM, it is an acute infection
If IgG is present with HBsAb, the infection has been cleared
If IgG is present without either IgM or HBsAb, it is a chronic infection

If HBsAb is present alone, the patient has been vaccinated
If IgM is present, it is an acute infection

How well did you know this?
1
Not at all
2
3
4
5
Perfectly
95
Q

A 57 year-old diabetic woman visits her GP for a review of her condition. Her HbA1c is 8.8% and she has been taking metformin for 3 months.

What is the most appropriate next step in her management?
A. Diet and more exercise
B. Add acarbose
C. Add a DPP-4 inhibitor
D. Continue with current medication
E. Add insulin
A

C. Add a DPP-4 inhibitor

Metformin is first line for type II diabetes, but if a patient’s HbA1c rises above 7.5% on metformin, addition of a DPP-4i, pioglitazone, sulphonylurea, or a SGLT-2 inhibitor is indicated.
Insulin is used for control when multiple therapy with other drugs has failed.
Acarbose is not very widely used n the UK, paritally due to the unpleasant side-effects of flatulence and diarrhoea.

How well did you know this?
1
Not at all
2
3
4
5
Perfectly
96
Q

A patient visits his GP for a general checkup. His blood pressure is recorded as 165/95.

What grade of hypertension is this?
A. Normal
B. High normal
C. Grade I
D. Grade II
E. Grade III
A

D. Grade II

120/80 = optimal
120-129 and/or 80-84 = normal
130-139 and/or 85-89 = high normal
140-159 and/or 90-99 = grade I hypertension
160-179 and/or 100-109 = grade II hypertension
180+ and/or 110+ = grade III hypertension (malignant)
140+ and <90 = isolated systolic hypertension

How well did you know this?
1
Not at all
2
3
4
5
Perfectly
97
Q

A 28 year-old man presents to A&E with bloody diarrhoea. He recently returned from South America where he thinks he may have eaten some suspect food.

What is the most likely causative organism?
A. Hepatitis A
B. Trypanasoma brucei
C. Entamoeba histolytica
D. Staphylococcus aureus
E. Klebsiella
A

C. Entamoeba histolytica

The other organisms do not generally cause bloody diarrhoea
The acronym to remmeber organisms that cause bloody diarrhoea is CHESS:
C - Campylobacter jejuni
H - Haemorrhagic E.coli (especially 0157:H7)
E - Entamoeba histolytica
S - Shigella
S - Salmonella

98
Q

A 16 year-old boy presents to A&E with an asthma attack.

Which of the following is indicative of an acute severe asthma exacerbation?
A. A decrease in GCS from 15 to 13
B. A pulse rate of 115 and a resp rate of 28
C. PEFR that is 55% of the previous best measurement
D. A silent chest upon auscultation
E. Oxygen saturation of 90%

A

B. A pulse rate of 115 and a resp rate of 28

Exacerbations of asthma are divided into 3 categories:

Moderate: PEFR 50-75% predicted/ previous best, patient is still able to speak normally.

Acute severe: PEFR 33-50% predicted/ previous best with oxygen saturation of at least 92%, OR patient can’t complete sentences in one breath, OR heart rate is above 110bpm, OR respiratory rate is above 25 bpm, OR the patient is having to use accessory muscles to breathe

Life-threatening: PEFR <33% predicted/ previous best, OR oxygen saturation is below 92%, OR a silent chest is heard, OR respiratory effort becomes poor, OR cyanosis, OR hypotension, OR cardiac arrhythmia, OR altered consciousness/ confusion

99
Q

A 34 year-old woman presents with jaundice and RUQ pain. She has a low grade fever, and has been feeling tired and achy for some time. Her period has not come for 7 weeks. A diagnosis of autoimmune hepatitis is made.

Which antibodies are associated with autoimmune hepatitis?
A. Anti-Rho
B. Lupus anticoagulant
C. cANCA
D. pANCA
E. Anti-smooth muscle
A

E. Anti-smooth muscle

Antibodies to know:
pANCA: primary sclerosing cholangitis, Churg-Strauss syndrome, microscopic polyangitis
cANCA: Polyangitis with granulomatosis
Anti-mitochondrial: primary biliary cirrhosis
Anti-Scl70: diffuse scleroderma
Anti-centromere: CREST syndrome
Anti-Rho and Anti-La: Sjogren’s syndrome
Anti-dsDNA: SLE
Anti-Smith: SLE
Anti-CCP: Rheumatoid arthritis
Anti-glomeruler basement membrane: Goodpasture’s syndrome
Anti-smooth muscle: autoimmune hepatitis
Lupus anticoagulant: antiphospholipid syndrome
Anti-cardiolipin: antiphospholipid syndrome

100
Q

A 21 year-old woman presents with painful urination and increased frequency. A urine dipstick reveals nitrites and leukocytes. A UTI is suspected.

What is the most appropriate treatment?
A. Co-amoxiclav
B. Trimethoprim
C. Clarithromycin
D. Ceftriaxone
E. Penicillin
A

B. Trimethoprim

Trimethoprim or nitrofurantoin is used to treat UTIs, the most common cause of which is E.coli.

101
Q

A 50 year-old man presents with constipation for 3 days. He has vomited. His only significant PMHx is open surgery on his bowel 5 years ago.

What is the moat appropriate first step?
A. Laxatives
B. NG tube insertion and i.v. fluids
C. Colonscopy
D. Laparotomy
E. Abdominal x-ray
A

B. NG tube insertion and i.v. fluids

This is a history of a man with bowel obstruction. The first step with bowel obstruction is to make the patient nil by mouth, establish i.v. access and give fluids, and insert an NG tube. The NG tube can then be used to decompress the dilated segment of bowel proximal to the blockage.
From there, AXR is used to monitor the blockage, as it may well resolve on its own. Conservative management involves giving fluid support, anti-emetics, and opiods. If indicated by AXR, surgery can be performed to relieve the blockage.

102
Q

A 40 year-old man presents to his GP after an episode of coughing up blood. This has happened before but there was more blood this time and the patient is worried. He has a history of nosebleeds that have been particularly bad recently. A CXR shows cavitating lesions, U&Es show elevated creatinine, and urinanalysis shows microscopic haematuria.

What is the most likely diagnosis?
A. Alport's syndrome
B. Polyangitis with granulomatosis
C. Churg-Strauss disease
D. Goodpasture's syndrome
E. Post infection glomerulonephritis
A

B. Polyangitis with granulomatosis

This is a history of polyangitis with granulomatosis, which is defined by a triad of upper respiratory tract (nosebleeds), lower respiratory tract (haemoptysis, and kidney (glomerulonephrits) involvement.
Other features of polyangitis with graulomatosis include destruction of the nose, giving it a ‘saddle’ shape, cavitating lesions on CXR, and arthritis.
Polyangitis with granulomatosis is heavily associated with cANCA antibodies

Antibodies to know:
pANCA: primary sclerosing cholangitis, Churg-Strauss syndrome, microscopic polyangitis
cANCA: Polyangitis with granulomatosis
Anti-mitochondrial: primary biliary cirrhosis
Anti-Scl70: diffuse scleroderma
Anti-centromere: CREST syndrome
Anti-Rho and Anti-La: Sjogren’s syndrome
Anti-dsDNA: SLE
Anti-Smith: SLE
Anti-CCP: Rheumatoid arthritis
Anti-glomeruler basement membrane: Goodpasture’s syndrome
Anti-smooth muscle: autoimmune hepatitis
Lupus anticoagulant: antiphospholipid syndrome
Anti-cardiolipin: antiphospholipid syndrome

103
Q

A patient presents with a burning epigastric pain that is worse when she eats. She reports a couple of episodes of dark, foul-smelling stools. She is a heavy smoker. An OGD confirms a peptic ulcer,

What is the most appropriate treatment?
A. Cimetidine with omperazole
B. Omeprazole with ranitidine and clarithrimycin
C. Omperazole with amoxicillin and clarithromycin
D. Cimetidine with amoxicillin and clarithromycin
E. Ompeprazole with flucloxaxillin

A

C. Omperazole with amoxicillin and clarithromycin

For peptic and duodenal ulcers, triple therapy is used. This consists of two antibiotics (either amoxicillin and clarithromycin, or metronidazole and clarithromycin) and a proton-pump inhibitor (omeprazole, lansoprazole).
This drug regimen is taken for a week, with each drug being taken twice daily.

104
Q

A 60 year-old man presents to A&E with central crushing chest pain radiating to the right arm and jaw. An ECG shows no ST-elevation, and an NSTEMI is suspected. 300mg aspirin is given immediately. A GRACE score is calculated and reveals a high 6-month mortality, so coronary angiography is scheduled.

What is the most appropriate next action?
A. Give warfarin
B. Give streptokinase
C. Give clopidogrel and fondaparinux
D. Give clopidogrel and unfractionated heparin
E. Give alteplase

A

D. Give clopidogrel and unfractionated heparin

When treating an NSETMI, the first step is 300mg aspirin.
Fondaparinux is also offered, unless the patient is likely to undergo angiography/ PCI, in which case unfractionated heparin is given.

The next step is to formally assess the 6-month mortality with a GRACE score, which will more definitively guide conservative or invasive treatment.

If the patient is likely to undergo coronary angiography/ PCI, or their GRACE score reveals a mortality of 1.5% or greater, 300mg clopidogrel is also given. In reality clopidogrel is usually given

105
Q

A 37 year-old woman presents to A&E with abdominal pain that began suddenly the day before, and coughing up blood. There was no preceding trauma. On further questioning she has noticed a rash on her lower limbs, and has been wheezy recently, which has woken her up in the night several times. She was asthmatic as a child but has been symptom-free for years. She also admits to fever, malaise, and dry cough. An antibody test is conducted to confirm the doctor’s suspicions.

Which antibody will be found, given the likely diagnosis?
A. AMA
B. ASMA
C. Anti-GBM antibody
D. cANCA
E. pANCA
A

E. pANCA

This is a history of Churg-Strauss syndrome, a vasculitis of the small and medium sized blood vessels associated with asthma, associated with pANCA and characterised by three phases of progression:
Allergic: the patient experiences asthma or rhinitis
Hypereosinophillic: mostly affects lungs and GI causing abdominal pain, GI bleeding, cough, fever, night sweats, weight loss
Vasculitic: blood vessels become inflamed causing clots and infarction, peritonitis, GI perforation, and inflammation of the heart muscle

Antibodies to know:
pANCA: primary sclerosing cholangitis, Churg-Strauss syndrome, microscopic polyangitis
cANCA: Polyangitis with granulomatosis
Anti-mitochondrial: primary biliary cirrhosis
Anti-Scl70: diffuse scleroderma
Anti-centromere: CREST syndrome
Anti-Rho and Anti-La: Sjogren’s syndrome
Anti-dsDNA: SLE
Anti-Smith: SLE
Anti-CCP: Rheumatoid arthritis
Anti-glomeruler basement membrane: Goodpasture’s syndrome
Anti-smooth muscle: autoimmune hepatitis
Lupus anticoagulant: antiphospholipid syndrome
Anti-cardiolipin: antiphospholipid syndrome

106
Q

A 36 year-old man presents with a 2 week history of diarrhoea. He also describes episodes of facial flushing. A thyroid exam is performed which reveals a thyroid nodule and enlarged anterior cervical lymph nodes. Upon further questioning the patient admits to episodes of extreme anxiety, headache, and palpitations occurring together, which have been bothering him for a couple of months. he vaguely remembers his mother having similar symptoms, for which she had to be admitted to hospital. The patient appears to be of normal build, and says his mother is of normal height.

What is the most likely diagnosis?
A. MEN2A
B. Thyrotoxicosis
C. MEN2B
D. Infective endocarditis
E. MEN1
A

A. MEN2A

Multiple endocrine neoplasia is a group of autosomal dominant conditions causing tumours to form in endocrine tissues. This history hints at a medullary thyroid carcinoma, along with a phaeochromocytoma, which points to MEN2A. MEN2B is also possible, but less likely given the absence of marfinoid habitus (tall, thin, gangly).

MEN1
Pituitary tumours
Parathyroid hyperplasia
Pancreatic tumours

MEN2A
Parathyroid hyperplasia
Medullary thyroid cancer
Phaeochromocytoma

MEN2B
Marfinoid habitus
Mucosal neuromas
Medullary thyroid cancer
Phaeochromocytoma
107
Q

A 63 year-old male ex-smoker is admitted with confusion, which is shown to be due to hyponatraemia. His blood pressure is 110/78 and seems well hydrated.

What is the most likely diagnosis?
A. Hyperthyroidism
B. Pneumonia
C. Infective exacerbation of COPD
D. Mesothelioma
E. Small cell carcinoma
A

E. Small cell carcinoma

Small cell carcinoma is an aggressive cancer that is heavily associated with smoking. Small cell is also linked to certain paraneoplastic syndromes, in this case SIADH which is caused by ADH release from the tumour and leads to a euvolemic hyponatraemia.

108
Q

A 20 year-old man presents with pain in his ear that has been progressively worsening over the past few days. Otoscopy reveals a bulging tympanic membrane, indicating otitis media. On questioning, the patient admits to frequent sinus and chest infections, including three hospital admissions for pneumonia. A previous CT scan shows mucous impaction and bronchiecstasis, and a previous sweat test was negative. Nobody in his family has had these issues so far as anyone can remember.

What is the most likely diagnosis?
A. Alpha 1 antitrypsin deficiency
B. Cystic fibrosis
C. Asthma
D. Primary ciliary dyskinesia
E. Down syndrome
A

D. Primary ciliary dyskinesia

Primary ciliary dyskinesia is when a lack of synchronised ciliary movement results in a failure to clear mucous from the body. This leads to frequent chest, sinus, and ear infections, infertility in males, and eventual widespread bronchiecstasis.
Situs inversus may be present - the organs may be mirrored on to the wrong side of the body (e.g. heart is on the right). If situs inversus is present, along with bronchiecstasis and chronic sinusitis, the patient has Kartagener syndrome

109
Q

An 18-year old girl presents to her GP with fever, sore throat and muscle ache of a week’s duration. Her anterior cervical lymph nodes are enlarged. Her tonsils look inflamed and an exudate can be seen. Upon palpation, she has splenomegaly.

What would most likely be seen on a blood film from this patient?
A. Smear cells
B. Auer rods
C. Atypical lymphocytes
D. Reed-Sternberg cells
E. Dacrocytes
A

C. Atypical lymphocytes

This is a history of infectious mononucleosis (the kissing disease) which causes pharyngitis, lymphadenopathy (particularly in the anterior cervical chain), generic flu symptoms, and splenomegaly.
A heterophile antibody test (also called Monspot test) is a quick way to diagnose infectious mononucleosis.

Smear cells are indicative of chronic lymphocytic leukaemia
Auer rods are indicative of acute myeloid leukaemia
Reed-Sternberg cells are indicative of Hodgkin’s lymphoma
Dacrocytes are are indicative of myelofibrosis

110
Q

A 67 year-old man comes in with sudden muscle weakness. There is noticeable facial droop on the right side, and the right side of the face is paralysed, apart from the forehead. Power is 2/5 in the right arm, and 4/5 in the right leg. There is visuospatial neglect on the right.

Which pathology is most likely?
A. Anterior cerebral artery infarct
B. Middle cerebral artery infarct
C. Posterior cerebral artery infarct
D. Berry aneurysm rupture
E. Posterior inferior cerebral artery infarct
A

B. Middle cerebral artery infarct

Here are rough guides for stroke
Anterior cerebral artery: paralysis of contralateral leg>arm and face, disturbance of executive function, and personality
Middle cerebral artery: paralysis of contralateral arm and face>leg, aphasia (left sided lesion), visuospatial neglect, contralateral hemisensory defect
Posterior cerebral artery: homonymous hemianopia, visual agnosia, Anton-Babinski syndrome (patient is blind but believes they can see)

Lateral medullary syndrome is a niche syndrome caused by infarction in the posterior inferior cerebral artery. Symptoms include cerebellar (ipsilateral dysdiadochokinseia, ataxia, dysmetria), spinothalamic (contralateral deficit in heat and pain sensation on the body), trigeminal (ipsilateral deficit in heat and pain sensation on the face, Horner’s syndrome (ipsilateral loss of sympathetic innervation), and vagus and glossopharyngeal (ipsilateral defects causing hoarseness, dysphagia and absent gag reflex).

111
Q

A 28 year-old patient with cystic fibrosis is admitted with a severe lung infection.

Which is the most likely causative organism?
A. Stapphylococcus aureus
B. Streptococcus pneumoniae
C. Legionella pneuomphilia
D. Chlamydia trachomatis
E. Pseudomonas aeruginosa
A

E. Pseudomonas aeruginosa

Pseudomonas is an organism that often causes nosocomial infections. Cystic fibrosis patients are particularly susceptible to pseudomonas infections.
The pathogens which commonly causes nosocomial infections can be remembered as the ESKAPE pathogens:
E - enterococcus
S - staphylococcus
K - klebsiella
A - acinetobacter
P - pseudomonas
E - enterobacter
112
Q

The following are sweat chloride concentrations measured in a 5 year-old child with recurrent chest infections.

Which of these indicates a positive diagnosis of cystic fibrosis?
A. <30 mmol/L
B. >60 mmol/L
C. <50 mmol/L
D. >50 mmol/L
E. 50 mmol/L
A

B. >60 mmol/L

A result of above 60 mmol/L indicates CF. Below 40 mmol/L rules out CF. An in-between measurement is ambiguous.

113
Q

A 55 year-old woman, who has never smoked, presents with a history of weight loss, decreased appetite, and finger clubbing. Her CXR shows a peripheral opacity suggesting bronchogenic carcinoma. She is awaiting a CT chest and bronchoscopy.

What is the most likely diagnosis?
A. Squamous cell carcinoma
B. Adenocarcinoma
C. Small cell carcinoma
D. Large cell carcinoma
E. Carcionoid tumour
A

B. Adenocarcinoma

Adenocarcinoma is the most common lung cancer in never-smokers and usually occurs in the peripheral lung. It is more common in women and is the most likely parenchymal tumour to have pleural involvement.

114
Q

A 67 year-old man comes in with sudden muscle weakness and facial droop. A stroke is suspected. The patient states their symptoms began 3 hours ago.

What is the most appropriate next step?
A. Streptokinase
B. Warfarin with heparin
C. CT head
D. Alteplase followed by aspirin
E. High flow oxygen
A

C. CT head

Whilst this patient is within the 4.5 hour thrombolytic window and so alteplase followed by aspirin would be an appropriate treatment, but first a haemorrhagic stroke must be excluded. If a CT head is not done, and a patient with a haemorrhagic stroke is given thrombolysis, they will die, so it’s an important diagnosis to exclude.

115
Q

A 3 year-old boy is brought in by his father with swelling in his hands and face. A urine dipstick reveals protein but no blood.

What is the most likely diagnosis?
A. Henoch-Schonlein purpura
B. Acute liver failure
C. Post-streptococcal glomerulonephritis
D. Minimal change disease
E. Haemolytic uraemic syndrome
A

D. Minimal change disease

MCD is an idiopathic condition, and is comfortably the most common cause of nephrotic syndrome in young children, and still a prominent one in teenagers

116
Q

A 29 year-old man presents to his GP with lower back pain. He states that the pain has been gradually worsening over a couple of months, is accompanied by stiffness, and is worst in the morning. You notice his eyes seem a little red, and he reports occasional eye pain. Upon auscultation you detect a murmur heard best in expiration at the left sternal border in the 3rd intercostal space.

What is the most likely finding?
A. HLA DR3
B. HLA DR4
C. HLA B27
D. Rheumatoid factor
E. Anti-CCP antibodies
A

C. HLA B27

This is a history of ankylosing spondylitis, an autoimmune arthopathy most common in young men that causes inflammation in, and progressive stiffening of, the spine.
Typical presentation is of low back pain and stiffness that is worse in the morning and improves with exercise.
Like many autoimmune join conditions, it has extra-articular manifestations, which can be remembered as the 5 a’s:
Apical lung fibrosis
Amyloidosis
Achilles tendonitis
Anterior uveitis
Aortic regurgitation

The murmur described in this question is typical of aortic regurgitation.

117
Q

A 45 year-old man presents with severe colicky flank pain radiating to the groin. He is difficult to take a history from, but you gather he recently had some painful inflammation in his toe, and you notice lumps on his ear. A CT KUB shows no abnormalities, but urinalysis shows blood in the urine.

What is the most likely diagnosis?
A. Kidney stone
B. Pyelonephritis
C. Glomerulonephritis
D. Cystitis
E. Muscle spasm
A

A. Kidney stone

Though the CT KUB was normal, this does not rule out a kidney stone, as kidney stones can be radio lucent depending on the material. This man’s PMHx indicates gout, which is a risk factor for uric acid kidney stones.

118
Q

A 40 year-old woman presents to her GP with a history of progressive muscle weakness in the upper limbs over the last few weeks. She has also experienced fever and weight loss. O/E she has inflamed red skin over the knuckles of her hands.

What is the most likely diagnosis?
A. Polymyositis
B. Psoriasis
C. Dermatomyositis
D. Hypothyroidism
E. CREST syndrome
A

C. Dermatomyositis

Dermatomyositis is an inflammatory disorder causing skin rashes (face, particularly eyes, and extensor surfaces) and progressive muscle weakness. Arthritis, lung involvement, and GI bleeding are less common manifestations.

119
Q

A 20 year-old student is brought to A&E with a fever, headache, and neck stiffness. Meningitis is suspected and an LP is performed. The patient’s history suggests a slow onset, less severe form of meningitis, which is suspected to be viral.

Which of these test results supports a diagnosis of viral meningiits?
A. Fibrinous CSF, high lymphocytes, high protein, low glucose
B. Straw-coloured CSF
C. Clear CSF, high lymphocytes, high protein, normal glucose
D. Cloudy CSF, high neutrophils, high protein, low glucose
E. Frank blood in the CSF

A

C. Clear CSF, high lymphocytes, high protein, normal glucose

Bacterial meningitis: Cloudy CSF, high neutrophils, high protein, low glucose
Viral meningitis: Clear CSF, high lymphocytes, high protein, normal glucose
Fungal/ TB meningitis: Fibrinous CSF, high lymphocytes, high protein, low glucose

120
Q

A 55 year-old woman is brought to A&E by her friend because she seems confused and disorientated, and has been displaying personality changes. Her friend states this has never happened before and seems to have come on over the course of one day. The patient is feverish, and you notice red lesions around the corner of her mouth. Shortly after admission she has a seizure

What is the most likely diagnosis?
A. Hepatic encephalopathy
B. Meningio-encephalitis
C. Meningitis
D. Encephalitis
E. Vascular dementia
A

D. Encephalitis

Encephalitis is most commonly cause by herpes simplex-1 and causes inflammation of the brain parenchyma. The patient will commonly experience headache and fever initially, then behavioural abnormalities, focal neurology (e.g. muscle weakness), confusion, and seizures.

121
Q

A 16 year-old girl comes to her GP because she is worried about some of her moles. O/E there are multiple small bumps, along with multiple brown, irregular patches on her torso, and brown nodules in her eyes. She also appears to have an unusual sideways curvature of the spine.

What is the most likely diagnosis?
A. Melanoma
B. Reactive arthritis
C. Neurofibromatosis type 1
D. Neurofibromatosis type 2
E. Wilson's disease
A

C. Neurofibromatosis type 1

Neurofibromatosis causes neurofibromas (here mistaken for moles), café au lait macules (brown patches), Lisch nodules (brown nodules in the eye), and scoliosis.

Neurofibromatosis type 2 involves the growth of bilatetral schwannomas (tumours on cranial nerve VIII) which can lead to hearing loss.

122
Q

A 45 year-old woman presents with a 2 month history of upper abdominal pain, ocurring 2-3 hours after meals. FBC reveals a microcytic anaemia

What is the most likely diagnosis?
A. GORD
B. Duodenal ulcer
C. Gastric ulcer
D. Biliary colic
E. Cholecystitis
A

B. Duodenal ulcer

Duodenal ulcers usually present with pain that is helped by eating, but reoccurs a couple of hours later - this is because the pyloric sphincter in the stomach closes when food enters, stopping acid from entering the duodenum and aggravating the ulcer.
By contrast, a peptic ulcer causes pain that becomes worse on eating.
Both are associated with H. pylori and treated with a PPI and two antibiotics (metronidazole/ amoxicillin with clarithromycin)

123
Q

A 61 year-old man presents with a 3-month history of upper abdominal pain following meals. he describes the pain as burning.

What is the most appropriate investigation?
A. H. pylori breath test
B. FBC
C. OGD endoscopy
D. Trial PPI
E. AXR
A

C. OGD endoscopy

The history of long-term upper abdominal pain soon after meals suggests a peptic ulcer. The best way to investigate this is by visualising it with an endoscopy.

124
Q

A 40 year-old lady presents with retrosternal chest pain. She reports the pain as being worse at night, and is accompanied by an odd taste in her mouth, and a cough, There are no FLAWS symtoms.

What is the most appropriate next step?
A. Upper GI endoscopy
B. Barium swallow
C. Manometry
D. Serum gastrin levels
E. Trial PPI
A

E. Trial PPI

The heartburn, dysphagia, odd taste, and cough are all suggestive of GORD. The first step in the management of GORD is a trial of a PPI. If this does not work, the diagnosis of GORD is reconsidered.

125
Q

A 59 year-old man presents with severe retrosternal burning pain. Upper GI endoscopy shows metaplastic changes in the epithelium.

What is the most likely diagnosis?
A. Gastric ulcer
B. Gastric carcinoma
C. Oesophageal carcinoma
D. GORD
E. Barrett's oesophagus
A

E. Barrett’s oesophagus

126
Q

A 28 year-old woman presents with a 2-year history of mild dysphagia to solids and liquids. She has no weight loss, but does have heartburn and nocturnal cough. PPIs and bronchodilators haven’t helped. A ‘bird beak’ appearance is noticed on barium swallow.

What is the most likely diagnosis?
A. Achalasia
B. Benign stricture
C. Plummer-Vinson syndrome
D. Oeshophageal
E. Stroke
A

A. Achalasia

Achalasia is a failure of the lower oesophageal sphincter to open at the appropriate time, resulting in a loss of oesophageal motility.
GORD can lead to achalasia, and has in the case of this patient, whose nocturnal cough and heartburn indicate GORD.
A bird-beak appearance in the oesophagus on barium swallow indicates achalasia.
A bird beak appearance in the colon on barium follow-through indicates sigmoid volvulus.

127
Q

A 76 year-old woman visits her GP with dysphagia to solids. This has been worsening over the past month. There is no coughing, choking or heartburn. She reports food getting stuck a couple of seconds after swallowing. She attributes her weight loss to not eating properly, which she thinks has caused her loose, dark stools. She feels tired and a blood film shows a microcytic anaemia.

What is the most likely diagnosis?
A. Stroke
B. Oesophageal cancer
C. Pharyngeal pouch
D. Plummer-Vinson syndrome
E. Benign stricture
A

B. Oesophgeal cancer

This is a history of dysphagia and melaena (dark, tarry, foul-smelling stool) on a background of weight loss in an elderly patient, all of which points to oesophageal cancer.

128
Q

A 53 year-old man staggers into A&E having vomited 6 times in 3 hours. He is intoxicated and jaundiced. His friend states that after the 3rd time the vomit began to contain fresh blood. Vitals are stable.

What is the most likely diagnosis?
A. Ruptured oesophageal varices
B. Mallory-Weiss tear
C. Ruptured peptic ulcer
D. Boerhaave syndrome
E. Ruptured varices
A

B. Mallory-Weiss tear

129
Q

A 47 year-old man presents to A&E with haematemesis. His wife reports that he had food poisoning 2 days ago. Suddenly this morning he experienced extreme chest pain and began to vomit blood. HR is 110, and BP is 85/60. Auscultation reveals crackles, and CXR reveals pneumomediastinum.

What is the most likely diagnosis?
A. Ruptured oesophageal varices
B. Mallory-Weiss tear
C. Ruptured peptic ulcer
D. Boerhaave syndrome
E. Myocardial infarction
A

D. Boerhaave syndrome

Boerhaave syndrome is oesophageal rupture due to vomiting.
Mackler’s triad is rarely fully present in Boerhaave syndrome, but consists of subcutaenous emphysema, chest pain, and vomiting.

130
Q

A 22 year-old woman presents with a 2-year history of intermittent diarrhoea and constipation. She complains of bloating and abdominal pain, which eases with defecation.

What is the most likely diagnosis?
A. Coeliac disease
B. Ulcerative colitis
C. Crohn's disease
D. IBS
E. Infectious diarrhoea
A

D. IBS

A history of chronic generic bowel symptoms with no red flags, and particularly with symptoms that are relieved on defecation, supports a diagnosis of IBS.

131
Q

A 26 year-old man presents to GP with weight loss, abdominal pain, and watery diarrhoea. He looks pale and has mouth ulcers.

What is the most likely diagnosis?
A. Coeliac disease
B. Ulcerative solitis
C. Crohn's disease
D. IBS
E. Infectious diarrhoea
A

C. Crohn’s disease

Crohn’s disease usually presents with diarrhoea that is not bloody or mucoid (unlike UC). A pale appearance indicates anaemia which can occur in a flare-up of IBD, and mouth ulcers are also associated with IBD>
The element of this history that makes Crohn’s more likely is the absence of blood and mucous.

132
Q

A 23 year-old woman presents to her GP with a limp. She has recently lost weight and has experienced bloody mucoid diarrhoea. O/E her right knee is tender and swollen, and her eyes are red.

What is the most likely diagnosis?
A. Coeliac disase
B. Ulcerative colitis
C. Crohn's disease
D. IBS
E. Infectious diarrhoea
A

B. Ulcerative colitis

Ulcerative colitis classically causes abdominal pain and bloody mucoid diarrhoea. It can have extra-GI symptoms, such as uveitis, seronegative arthritis, erythema nodosum (inflammation of fat cells that can be seen through the skin, often on the shins) and pyoderma gangrenosum (painful ulceration of the skin with violet margins).
UC is heavily associated with primary sclerosing cholangitis, and signs of UC include apthous mouth ulcers, and clubbing,

133
Q

A 31 year-old man presents with diarrhoea, weight loss, and LIF pain. O/E you note red marks on his shins.

What would be the treatment given the most likely diagnosis?
A. I.V. corticosteroids
B. Oral prednisolone
C. Oral mesalazine
D. Oral azathioprine
E. I.V. cyclosporin
A

B. Oral prednisolone

This is a flare-up of what is probably UC (LIF pain indicates UC as Crohn’s usually spares the rectum, whereas RIF pain indicates Crohn’s because UC usually spares the terminal ileum). First-line treatment for a flare-up is oral steroids.

134
Q

A 35 year-old man presents to his GP following an episode of rectal bleeding. he noticed fresh blood on the paper after wiping. There was no blood in the stool. He adds that his anus is very sore and it is agony to defecate.

What is the most likely diagnosis?
A. Haemorrhoids
B. Anal fissure
C. Crohn's disease
D. Ulcerative colitis
E. Colorectal carcinoma
A

B. Anal fissure

Agonising defecation along with fresh blood should always make you think of an anal fissure - a tear in the mucosa of the anus.

135
Q

A 67 year-old man presents to his GP complaining of rectal bleeding. over the last few months he’s noticed blood mixed in with the stool. he sometime feels like he hasn’t fully emptied his bowels after defecating, and has been feeling tired.

What is the most appropriate next step?
A. Routine referral to colorectal surgeons
B. Urgent referral to colorectal surgeons
C. FBC
D. Abdominal exam
E. Faecal occult blood test

A

D. Abdominal exam

Whilst referral to surgeons and an FBC will be needed, the first step is an abdominal exam. Get into the habit of thinking of bedside tests and exams, then bloods, then imaging.

136
Q

A 5-year old boy of Indian origin presents to A&E with
lymphadenopathy and a mediastinal mass on chest radiology. White cells 180 x109/L (5-17), Hb 93 g/L (115-135), platelets 43x109/L (150-400).

What is the most likely diagnosis?
A. Acute myeloid leukaemia
B. Acute lymphoblastic leukaemia
C. Pericarditis
D. Thromotic thrombocytopenic purpura
E. Tuberculosis
A

B. Acute lymphoblastic leukaemia

Guide to haematological malignancy:
ALL: The patient is a young child with acute onset (weeks) fatigue, bruising, and infections. A blood film will show a high number of blast lymphocytes and crowding out of normal cells. They may also have testicular swelling.

AML: The patient is older and with acute onset (weeks) fatigue, bruising and infections. A blood film will show blast cells of multiple different lineages and crowding out of normal cells. Auer rods will be seen in the cytoplasm of blast cells. AML is also associated with DIC.

CLL: The patient is elderly and with chronic onset fatigue, with weight loss, and fever, though commonly the patient will be asymptomatic and CLL will be an incidental finding. If the disease is caught late, bruising, fatigue, and recurrent infections may be reported. A blood film will show high numbers of mature lymphocytes (with crowding out only in late stages) and smudge/ smear cells.

CML: The patient is elderly and with chronic onset fatigue, with weight loss, and fever, though commonly the patient will be asymptomatic and CLL will be an incidental finding. If the disease is caught late, bruising, fatigue, and recurrent infections may be reported. A blood film will show an elevated number of mature white cells of all lineages. CML is strongly associated with the Philadelphia chromosome, which can be seen on cytogenetic testing of bone marrow aspirate.

NB gout can be associated with leukaemia, as the increased breakdown of abnormal cells leads to hyperuricaemia

137
Q

A 65 year old lady with known CVD presents to the GP with pain in her legs. She finds the pain comes on when she is walking to the shops, but is relieved by rest. She has a 40 pack year smoking history.

What is the most likely diagnosis?
A. Acute limb ischaemia
B. Deep vein thrombosis
C. Varicose veins
D. Peripheral arterial disease
E. Femoral Aneurysm
A

D. Peripheral arterial disease

Smoking increases the risk of arterial disease, and in this case the arteries supplying the patient’s legs have become occluded causing ischaemic pain when she exerts herself.

138
Q

A 69 year old man with a background of hypertension complained of flank pain all day at work. He then has sudden onset abdominal pain that radiates to his back, flank and groin. He arrives in an ambulance
unconscious. The doctor notes Grey Turner’s and Cullen’s signs.

What is the most likely diagnosis?
A. Renal colic
B. Myocardial ischaemia
C. Ruptured AAA
D. Pancreatitis
E. Mesenteric ischaemia
A

C. Ruptured AAA

Sudden abdominal pain radiating to the back and/or flanks is consistent with the rupturing of an abdominal aortic aneurysm. Grey-Turner’s sign is bruising in the flanks, whilst Cullen’s sign is bruising around the umbilicus: both are signs of retroperitoneal bleeding (e.g. haemorrhagic pancreatitis, ruptured AAA).

139
Q

A 50 year old man presents with bleeding gums and
splenomegaly. Over the past month he has been treated twice for pneumonia. On examination he is pale and lethargic. A stained blood film shows blast cells with purple lines visible in the cytoplasm.

What is the most likely diagnosis?
A. Acute lymphoblastic leukaemia
B. Acute myeloid leukaemia
C. Disseminated intravascular coagulation
D. Iron deficiency anaemia
E. Metastatic cancer
A

B. Acute myeloid leukaemia

Guide to haematological malignancy:
ALL: The patient is a young child with acute onset (weeks) fatigue, bruising, and infections. A blood film will show a high number of blast lymphocytes and crowding out of normal cells. They may also have testicular swelling.

AML: The patient is older and with acute onset (weeks) fatigue, bruising and infections. A blood film will show blast cells of multiple different lineages and crowding out of normal cells. Auer rods will be seen in the cytoplasm of blast cells. AML is also associated with DIC.

CLL: The patient is elderly and with chronic onset fatigue, with weight loss, and fever, though commonly the patient will be asymptomatic and CLL will be an incidental finding. If the disease is caught late, bruising, fatigue, and recurrent infections may be reported. A blood film will show high numbers of mature lymphocytes (with crowding out only in late stages) and smudge/ smear cells.

CML: The patient is elderly is often diagnosed during the chronic asymptomatic phase after an incidental finding. A blood film will show an elevated number of mature neutrophils and myelocytes, with a massive white cell count (50-200). Physical examination may well reveal massive hepatosplenomegaly. However CML can accelerate into an acute phase where myeloblasts become visible in peripheral blood, and bleeding, anaemia, or recurrent infections occur. CML is always associated with the Philadelphia chromosome, which can be seen on cytogenetic testing of bone marrow aspirate. NB: the presence of the Philadelphia chromosome is not 100% specific as it may also appear in ALL.

NB gout can be associated with leukaemia, as the increased breakdown of abnormal cells leads to hyperuricaemia

140
Q

A 72-year-old man is having a routine blood test. His doctor is concerned about the results: Hb 100 g/L (130-180), MCV 80 fL (80-100), white cells 100 x109/L (4-11), platelets 210 x109/L (150-400). He orders a blood film, which features smudge cells.

What is the most likely diagnosis?
A. Chronic bacterial infection
B. Chronic myeloid leukaemia
C. Chronic lymphocytic leukaemia
D. Occult malignancy
E. Mononucleosis
A

C. Chronic lymphocytic leukaemia

Smudge cells are mature, but poorly formed lymphocytes that break upon preparation for viewing.

Guide to haematological malignancy:
ALL: The patient is a young child with acute onset (weeks) fatigue, bruising, and infections. A blood film will show a high number of blast lymphocytes and crowding out of normal cells. They may also have testicular swelling.

AML: The patient is older and with acute onset (weeks) fatigue, bruising and infections. A blood film will show blast cells of multiple different lineages and crowding out of normal cells. Auer rods will be seen in the cytoplasm of blast cells. AML is also associated with DIC.

CLL: The patient is elderly and with chronic onset fatigue, with weight loss, and fever, though commonly the patient will be asymptomatic and CLL will be an incidental finding. If the disease is caught late, bruising, fatigue, and recurrent infections may be reported. A blood film will show high numbers of mature lymphocytes (with crowding out only in late stages) and smudge/ smear cells.

CML: The patient is elderly and with chronic onset fatigue, with weight loss, and fever, though commonly the patient will be asymptomatic and CLL will be an incidental finding. If the disease is caught late, bruising, fatigue, and recurrent infections may be reported. A blood film will show an elevated number of mature white cells of all lineages. CML is strongly associated with the Philadelphia chromosome, which can be seen on cytogenetic testing of bone marrow aspirate.

NB gout can be associated with leukaemia, as the increased breakdown of abnormal cells leads to hyperuricaemia

141
Q

A 38 year old lady presents with swelling in her leg, and associated calf tenderness. She has been taking the OCP for several years.

What is the best management for this patient?
A. PCI
B. Embolectomy
C. Aspirin
D. Alteplase
E. Dalteparin
A

E. Dalteparin

NICE guidelines state that a LMWH or Fondaparinux should be used in treatment of a DVT. Warfarin is used in the treatment of a PE.

A Wells score is used to calculate the likelihood of DVT in a patient, with one point being assigned for each of:
Active cancer (treatment ongoing, within the last 6 months, or palliative).
Paralysis, paresis, or recent plaster immobilization of the legs.
Recently bedridden for 3 days or more, or major surgery within the last 12 weeks requiring general or local anaesthetics.
Localized tenderness along the distribution of the deep venous system (such as the back of the calf).
Entire leg is swollen.
Calf swelling by more than 3 cm compared with the asymptomatic leg (measured 10 cm below the tibial tuberosity).
Pitting oedema (greater than on the asymptomatic leg).
Collateral superficial veins (non-varicose).
Previously documented DVT.

A Wells of 2 or more indicates that a DVT is likely.
If DVT is likely, perform a leg vein USS
If DVT is unlikely, perform a D-dimer test

142
Q

A 60 year old male with known atrial fibrillation presents to A&E with a sudden onset of a painful, cold leg. The doctor is unable to feel peripheral pulses, and upon examination notes a loss of sensation and
paralysis.

What is the definitive management?
A. Embolectomy
B. Watch and wait
C. Angioplasty
D. Amputation
E. LMWH
A

D. Amputation

Loss of sensation and paralysis are the key hints that a limb is no longer viable and must be amputated.

143
Q

A 69 year old heavy smoker complains of pain in his leg when he walks to the bus stop. On examination of his leg, you see shiny skin, patchy hair, weak pulses and brittle toenails.

What investigation will help diagnose his condition?
A. Angiography
B. Doppler Ultrasound
C. Magnetic Resonance Angiography
D. ABPI
E. D-Dimer
A

D. ABPI

Ankle-brachial pulse index compares the arterial pressure in the arm with that in the ankle. This is used to tell whether there is stenosis of vasculature in the legs.

144
Q

A 32 year old woman on the OCP complains of pain in her calf for one day. She does not have any chest pain or shortness of breath. The nurse tells you that the A&E doctors assessed the patient, who scored 2 although she cannot remember the name of the score.

What is the most appropriate investigation?
A. D-Dimer
B. MRA
C. Leg Vein USS
D. ABPI
E. CTPA
A

C. Leg Vein USS

This is a history of a DVT

A Wells of 2 or more indicates that a DVT is likely.
If DVT is likely, perform a leg vein USS
If DVT is unlikely, perform a D-dimer test.

A Wells score is used to calculate the likelihood of DVT in a patient, with one point being assigned for each of:
Active cancer (treatment ongoing, within the last 6 months, or palliative).
Paralysis, paresis, or recent plaster immobilization of the legs.
Recently bedridden for 3 days or more, or major surgery within the last 12 weeks requiring general or local anaesthetics.
Localized tenderness along the distribution of the deep venous system (such as the back of the calf).
Entire leg is swollen.
Calf swelling by more than 3 cm compared with the asymptomatic leg (measured 10 cm below the tibial tuberosity).
Pitting oedema (greater than on the asymptomatic leg).
Collateral superficial veins (non-varicose).
Previously documented DVT.

145
Q

A 25-year-old man presents with a swelling on his neck. The area is usually painless but he has noticed it sometimes becomes tender after work when he socializes with friends. Over the past 2 months he has lost 5kg and complains of night sweats. His only past medical history is glandular fever in his early teens.

What is the most likely diagnosis?
A. Hodgkin lymphoma
B. Non-Hodgkin lymphoma
C. Burkitt lymphoma
D. Mantle cell lymphoma
E. MALT lymphoma
A

A. Hodgkin lymphoma

In theory, Hodgkin’s lymphoma more commonly causes B-symptoms (systems of systemic illness e.g. FLAWS) whereas non-Hodgkin’s lymphoma more usually features with infiltratve symptoms.

In reality, a better way to differentiate the two is that Hodgkin’s lymphoma is rarer than non-Hodgkin’s and can occur in younger people, whereas non-Hodgkin’s lymphoma rarely occurs in younger people (peak incidence is 70).

Reed-Sternberg cells will be seen on the blood film of somebody with Hodgkin’s lymphoma: if the question mentions these as being present, it is definitely Hodgkin’s lymphoma.

The risk of both Hodgkin’s and non-Hodgkin’s lymphoma is increased in those who have previously had glandular fever.

146
Q

A 60 year old patient presents to A&E with central crushing chest pain, radiating to the jaw. His ECG is normal.

Which is the best next step?
A. Creatine Kinase
B. Repeat ECG
C. Discharge
D. Exercise ECG
E. Troponin
A

E. Troponin

Troponin is an enzyme released from heart muscle when it is damaged, and so will give an indication of whether the patient has had an MI or is experiencing angina.

147
Q

A 46 year old diabetic man presents to A&E following collapse. The patient is very distressed and is sweating. On the way to the hospital, his wife had to stop the car to allow him to vomit. His ECG is normal but his 12 hour troponins are positive.

What is the most likely diagnosis?
A. Inferior STEMI
B. Anterior STEMI
C. NSTEMI
D. Unstable Angina
E. Ventricular Wall Aneurysm
A

C. NSTEMI

This history is suggestive of an MI (collapse, sweating, vomiting, positive troponin) which must be an NSTEMI as ECG is normal. The absence of chest pain is probably due to autonomic neuropathy as a consequence of the patient’s diabetes.

148
Q

A 56 year old overweight man with a history of high cholesterol comes in complaining of central crushing chest pain that came on at rest. He has had a similar pain before but only when playing tennis. His ECG shows ST depression and a 12 hour troponin is negative.

What is the most likely diagnosis?
A. Inferior STEMI
B. Anterior STEMI
C. NSTEMI
D. Unstable Angina
E. Ventricular Wall Aneurysm
A

D. Unstable Angina

Chest pain occurring at rest indicates unstable angina: i.e. the patient’s arteries have become so occluded they can’t supply enough blood to his heart even when he’s not exerting himself. The patient has clear risk factors for cardiovascular disease, and the pain he’s felt whilst playing sport has been caused by stable angina, which has now worsened. A negative troponin makes a myocardial infarction unlikely.

149
Q

A 50 year old man presents to his GP with central chest pain. The ECG shows a STEMI. His sats are 96%. What medication should the GP give whilst waiting for an ambulance?

A. Fondaparinux 2.5mg
B. Oxygen
C. Propanolol
D. Aspirin/Clopidogrel 300mg
E. Ramipril
A

D. Aspirin/Clopidogrel 300mg

The first step in the management of a STEMI is a 300mg loading dose of aspirin, or clopidogrel if aspirin is contraindicated.

150
Q

A 70 year old female with known hypertension and hypercholesterolaemia presents with central crushing chest pain, which radiates to the left arm. The pain started 2 hours ago. Her ECG shows LBBB.

What is the most appropriate management?
A. Thrombolysis
B. Angiography
C. Fibrinolysis
D. PCI
E. CABG
F. Fondaprinux
A

D. PCI

PCI should be attempted as soon as possible in the case of a STEMI. If PCI cannot be delivered within 2 hours of the time when fibrinolysis could be given, a thrombolytic drug should be considered.

151
Q

A 78 year old woman is bought to A&E following chest pain. Her ECG shows ST depression and T-wave inversion.

What is the most appropriate management?
A. PCI
B. Fibrinolysis
C. Fondaparinux
D. CABG
E. Simvastatin
F. Thrombolysis
A

C. Fondaparinux

This is either an NSTEMI or unstable angina, but either way the initial management is the same because they are very closely related conditions
Oxygen if there is evidence of hypoxia or pulmonary oedema
Nitrates for pain relief
Aspirin and clopidogrel with one of: unfractionated heparin, LMWH, or fondaparinux
Beta blockers
Eptifibatide, abciximab, or tirofiban may also be used in particularly high risk patients

152
Q

A man is being discharged following an MI.

Which of the following drugs should not make up a part of his post MI management?
A. ACEi
B. Aspirin
C. Clopidogrel
D. Heparin
E. Statin
F. B-Blocker
A

D. Heparin

Standard therapy for everyone who has had an MI is to be offered the following drugs:
ACEi
Statins
Dual antiplatelet therapy (aspirin + 1 other)
Beta blocker

153
Q

A 60-year old woman develops acute onset back pain on a background of rheumatoid arthritis. She also reports feeling more short of breath. Blood tests reveal: Hb 98 g/L (115-165), MCV 80 fL (80-100), white cells 9 x109/L (4-11), Ca2+ 3.0 mmol/L (2.2-2.6), raised
urea and creatinine.

What is the next most important test to confirm the diagnosis?
A. MRI back &amp; spine
B. Blood film
C. Serum/urine electrophoresis
D. Immunophenotyping
E. ESR
A

C. Serum/urine electrophoresis

This is a history of multiple myeloma. To diagnose MM, electrophoresis is used to identify the abnormal light chain proteins produced by the abnormal plasma cells.

Multiple myeloma is a cancer of plasma cells (mature B-cells) that reside in the bone marrow. The effects of MM can be remembered using CRAB:

C - calcium: calcium is released from bone as it is broken down by the cancer, increasing serum calcium. This leads to polyuria, polydipsia, constipation, confusion, kidney stones, and altered mental status (stones, abdominal moans, and psychic groans).

R - renal: malignant plasma cells secrete malformed proteins which damage the kidneys causing kidney failure. This in turn may lead to uraemia, causing nausea, and electrolyte imbalances e.g. hyperkalaemia, which can cause cardiac arrhythmia. In addition to the effect of proteins, the elevated serum calcium leads to calcium deposition in the kidneys, worsening the renal failure. These proteins will also cause rouleaux formation on a blood film.

A - anaemia: infiltration of healthy bone marrow by malignant plasma cells causes a decrease in RBC output. Other cell lineages will also be affected, including white cells, meaning that people with MM are more vulnerable to infection.

B - bone: bone pain is a very common complaint in MM and helps to differentiate the other symptoms from other causes. In MM the malignancy can spread to the extent where it compresses the spinal cord, causing radicular pain, bladder and bowel incontinence, loss of sensation, and paraplegia.

154
Q

A 75-year-old man with dyspnoea and chest pain on exertion presents to his GP. On examination he has a slow rising pulse and his blood pressure is 120/110. On auscultation an ejection systolic murmur is heard which radiates to both carotids.

What is the most likely diagnosis?
A. Acute myocardial infarction
B. Aortic stenosis
C. Aortic regurgitation
D. Angina
E. Mitral stenosis
A

B. Aortic stenosis

A narrow pulse pressure (difference between systolic and diastolic blood pressures), a slow rising pulse, and an ejection systolic murmur radiating to the carotids all indicate aortic stenosis.

155
Q

A 75-year-old man presents with flushed, red cheeks and shortness of breath when lying flat. His past medical history includes childhood scarlet fever which led to sore joints and a red rash. On examination: irregularly irregular pulse, mid-diastolic murmur, opening snap.

What is the most likely diagnosis?
A. Mitral stenosis
B. Mitral regurgitation
C. Aortic stenosis
D. Aortic regurgitation
E. Tricuspid stenosis
A

A. Mitral stenosis

The flushed red cheeks described are as a result of a malar rash, a flushing of the face that spares the nasolabial folds.
SOB when lying flat is called orthopnoea and indicates pulmonary oedema.
A childhood history of scarlet fever or rheumatic fever (often hinted at by an illness that causes choreic movements) is significant because they can both be caused by Streptococcus pyogenes which can lead to rheumatic heart disease.
In this case the rheumatic heart disease has caused mitral stenosis (mid-diastolic murmur with loud first heart sound), which in turn has led to atrial fibrillation (irregularly irregular heart rate).

156
Q

A 25-year-old intravenous drug user presents to A&E with SOB. She was treated previously for infective endocarditis. On examination she has ascites and a palpable liver. On auscultation there is a pan-systolic murmur louder on inspiration.

What is the most likely diagnosis?
A. Pulmonary regurgitation
B. Mitral regurgitation
C. Tricuspid regurgitation
D. Mitral stenosis
E. Pulmonary stenosis
A

C. Tricuspid regurgitation

A murmur that is heard louder on inspiration will be on the right side of the heart: a way to remember this is with RILE (right - inspiration, left - expiration)
I.V. drug users are at increased risk of right heart endocarditis. A pan-systolic murmur suggests either tricuspid or mitral regurgitation, which in this case is most likely to be tricuspid.

157
Q

A 9 year-old patient presents with some mild dysphagia and a lump in her neck. Upon palpation the mass is distinct, smooth and symmetrical, is in the midline, and moves upon swallowing and protrusion of the tongue.

What is the most likely diagnosis?
A. Grave's disease
B. Toxic nodular goitre
C. Asymptomatic goitre
D. Thyroglossal cyst
E. Pharyngeal pouch
A

D. Thyroglossal cyst

If it’s a neck lump and it moves with tongue protrusion and swallowing, it’s a thyroglossal cyst

158
Q

A 36 year-old woman is admitted to A&E with a blood pressure of 80/50 and a heart rate of 120bpm. A rushed history taken from her reveals she has been losing weight for the past couple of months, though as she gave birth two months ago, this did not worry her. She has no PMHx bar a visit to her GP 1 month ago because of a failure to begin breast feeding.

What is the most likely cause?
A. Thyrotoxicosis
B. Sepsis
C. Hypothyroidism
D. Sheehan's syndrome
E. Malignancy
A

D. Sheehan’s syndrome

Sheehan’s syndrome is ischaemic damage to the pituitary gland in the context of blood loss and consequent hypovolaemia during childbirth.
In this case the pituitary has been badly damaged enough that it can no longer release sufficient ACTH, causing secondary adrenal failure and consequent Addisonian crisis. The failure to begin lactating is due to failure of the pituitary to produce prolactin.

159
Q

A 55 year-old man is admitted to A&E with sudden onset SOB and chest pain. aTTP and PT are elevated, and D-dimer is extremely elevated.

What is the most likely underlying cause of this event?
A. CLL
B. CML
C. AML
D. Non-Hodgkin's lymphoma
E. Hodgkin's lymphoma
A

C. AML

AML is strongly associated with DIC which has caused the elevated clotting blood tests and pulmonary embolism (SOB, chest pain) seen in this patient.

160
Q

A 32 year-old man presents with palpitations. He seems agitated while you take a history from him, and upon examination he has a tremor, is sweating profusely, and is tachycardic. He admits to some diarrhoea recently, and to having a sore throat two weeks ago which he saw the GP for, but was given no medication.
T3/T4 is elevated
TSH is low

Which investigation will confirm the most likely diagnosis?
A. Thyroid examination
B. FBC
C. Serology for thyroid stimulating antibodies
D. Radioiodine uptake scan
E. MRI

A

D. Radioiodine uptake scan

This is a history of viral (deQuervain’s) thyroiditis, with a patient becoming acutely hyperthyroid in the context of a recent viral throat infection.
A radioiodine scan will show little or no uptake because the thyroid follicular cells have been damaged and so leak T3 and T4, whilst being unable to absorb iodine. The patient will become hypothyroid and then resolve to being euthyroid. The condition is self-resolving and treatment is symptomatic.

161
Q

Which of these would indicate critical limb ischaemia?

A. Pain on walking <10m
B. Pain on walking <5m
C. Pain waking the patient at night
D. Infection in the toes
E. Shooting sensations down the back of the legs
A

C. Pain waking the patient at night

Critical limb ischaemia is a triad of rest pain, arterial ulcers, and gangrene. Pain that wakes the patient at night is, of course, pain at rest. Patients will report that hanging their foot down off the bed improves their symptoms.

162
Q

A 42 year-old woman presents to her GP 6 weeks after going on safari in Africa for her holidays. She has noticed that her urine has turned dark, and she has started feeling feverish (which comes in cycles), achy, and very tired.

Which test would confirm the most likely diagnosis?
A. PCR
B. Ziehl-Neelson stain
C. Congo Red stain
D. Sudan Black B stain
E. Thick blood film
A

E. Thick blood film

This is a history of malaria, which can be identified on a Giemsa stained thick blood film. A thin blood film would be used to identify the particular species of malaria.
Ziehl-Neelson stain reveals TB
Congo Red stain reveals amyloidosis
Sudan Black B stain helps to diferentiate myeloblasts from lymphoblasts

163
Q

A 58 year-old black woman presents to her GP because she has noticed a dark streak on the palm of her left hand, which she thinks may have increased in size over the past month.

Which is the most likely diagnosis?
A. Acral lentiginous melanoma
B. Nodular melanoma
C. Surface spreading melanoma
D. Neurofibromatosis type I
E. Melanoma in situ
A

A. Acral lentiginous melanoma

Black and asian people rarely develop melanoma, but when they do it is most likely an acral lentiginous melanoma, which is also rare in white people.

It’s worth noting that the indicator of prognosis in melanoma is the depth of invasion of the tumour, previously classified using the Breslow depth, now classified using the AJCC system.

164
Q

An 80 year-old man presents to his GP with three new growths his wife has noticed on his back. They are dark and have a greasy appearance. They seem distinct from the tissue around them, and have a ‘stuck-on’ appearance.

What is the most likely diagnosis?
A. Nodular melanoma
B. Squamous cell carcinoma
C. Basal cell carcinoma
D. Surface spreading melanoma
E. Sebhorreic wart
A

E. Sebhorreic wart

Sebhorreic warts are benign growths, common in the elderly, that have a greasy, ‘stuck-on’ appearance.

165
Q

A 45 year-old caucasian woman presents with nausea, vomiting, and palpitations. U&Es show raised potassium and creatinine. The only significant PMHx is hypertension diagnosed 1 month ago for which she is taking one drug. Urine dipstick reveals no leucocytes, nitrites, or blood.

What is the most likely diagnosis?
A. Hypertensive nephropathy
B. Nephrotic syndrome
C. Glomerulonephritis
D. Acute tubular necrosis
E. Drug effect
A

E. Drug effect

First line treatment for hypertension in people under 55/ non afro-carribean people is an ACEi. In patients with an underlying renal artery stenosis (which up until this point can be asymptomatic) an ACEi can cause an AKI.

166
Q

A patient presents to A&E and is diagnosed with a stroke.

Which of these options is most suggestive of a middle cerebral artery stroke?
A. Total blindness with confabulation
B. Weakness in the arm>leg, hemispatial neglect
C. Executive dysfunction (loss of inhibition, cognitive defects, impaired working memory)
D. Weakness in the leg>arm, bitemporal hemianopia
E. Ataxia and nystagmus

A

B. Weakness in the arm>leg, hemispatial neglect

Anterior stroke: cognitive dysfunction, personality changes, impaired decision making, disinhibition, weakness in leg>arm and face

Middle stroke: weakness in arm and face>leg, hemi-spatial neglect, aphasia

Posterior stroke: homonymous hemianopia, visual agnosia, proposagnosia, Anton-Babinski syndrome (patients believe they can see but are in fact blind)

NB: when a stroke causes facial paralysis, the forehead is always spared.

167
Q

A 69 year-old alcoholic is admitted to A&E with epigastric pain radiating through to the back that is severe enough to make her curl into the foetal position. Alcoholic pancreatitis is diagnosed. Shortly after admission, she develops hypotension, fever, tachypnoea and her oxygen saturations drop.

What is the most likely cause of the deranged lung function?
A. Pleural effusion
B. Cardiac pulmonary oedema
C. Pneumonia
D. ARDS
E. Metabolic acidosis
A

D. ARDS

ARDS is essentially sepsis in the lungs, and can develop after any systemic inflammatory condition (sepsis, trauma, drug reaction). Inflammation in the lungs leads to oedema, which deranges lung function.

168
Q

A 24 year-old woman presents with chest pain and regurgitation of food. She states that the pain is worse when lying down and is accompanied by a strange taste in the back of her mouth. Sometimes when she eats solid food, it will painlessly regurgitate a few seconds later.

What is the most appropriate investigation to confirm the most likely diagnosis?
A. Barium swallow
B. Reassure and discharge
C. CXR
D. OGD
E. Colonoscopy
A

A. Barium swallow

A history of GORD (chest pain, strange taste, worse when lying down) along with painless regurgitation indicates a hiatus hernia. Hiatus hernias can be seen on OGD and CXR sometimes, but the best investigation to confirm is a barium swallow.

169
Q

A 70 year-old man presents with haemoptysis. On questioning he admits weight loss and chronic cough, along with abdominal pain, low mood, polyuria, and polydipsia. His only signficant PMHx is treatment for a kidney stone 1 month ago. He has a 40 pack-year history, and a CXR shows a peri-hilar cavitating lesion.

What is the most likely diagnosis?
A. Lung metastases
B. Mesothelioma
C. Adenocarcinoma
D. Large cell lung carcinoma
E. Squamous cell lung carcinoma
A

E. Squamous cell lung carcinoma

Squamous cell lung carcinoma is the second most common lung cancer in smokers, and causes cavitating lesions. It can also classically release PTH related peptide which causes hypercalcaemia, leading to the abdominal pain, low mood, polyuria, and polydipsia seen here

170
Q

A patient presents with a 2 month history of a dry cough, low grade fever, and malaise. A Coombs DAT test is positive.

What is the most likely causative organism?
A. Streptococcus pneumoniae
B. Mycoplasma pneumoniae
C. Haemophilus influenza
D. Legionella pneumoniae
E. Chlamydia pneumophilia
A

B. Mycoplasma pneumoniae

This is a history of an atypical pneumonia (insidious onset, dry cough, low fever, malaise) which has three main causative organisms:

Legionella: increased risk in smokers and people who have prolonged contact with stagnant water (e.g. plumbers), causes hyponatraemia and associated bradycardia and confusion

Mycoplasma pneumoniae: causes cold aglutinin anaemia (Coombs DAT test is for agglutination anaemia) and can be associated with transverse myelitis

Chlamydia pneumophilia: we don’t have to know

NB: another atypical organism is chlamydia psittacai, which should be suspected if the patient has frequent contact with birds.

171
Q

Which of the following is not a feature of rheumatoid arthritis?

A. Z thumb
B. Heberden's nodule
C. Swan neck deformity
D. Rheumatoid nodule
E. Boutonniere deformity
A

B. Heberden’s nodule

Heberden’s nodules are swellings at the distal inter-phalangeal joint that indicate osteoarthritis.

172
Q

Which of the following is the best diagnostic marker of rheumatoid arthritis?

A. CRP
B. ESR
C. FBC
D. Rheumatoid factor
E. Anti-CCP antibodies
A

E. Anti-CCP antibodies

Whilst CRP and ESR will probably be raised in RhA, they are not at all specific. Rheumatoid factor is only present in around 2/3 of cases, but anti-CCP antibodies are a specific and sensitive test for RhA.

173
Q

A 35 year-old retired footballer presents with a sore knee. He states that the pain is worse on moving and the knee ‘creaks’. He is otherwise well.

What would be seen on an x-ray?
A. Salt and pepper appearance
B. Looser’s zones
C. Soft tissue opacity, gas in soft tissue
D. Joint space narrowing, osteophytes, subchondral cysts
E. Joint space narrowing, marginal bony erosions, osteopenia

A

D. Joint space narrowing, osteophytes, subchondral cysts

A salt and pepper appearance is due to bone resorption secondary to hyperparathyroidism.
Looser’s zones are areas of osteopenia seen in osteomalacia.
Soft tissue opacity with gas in soft tissue is the appearance of necrotising fascitis
Joint space narrowing, marginal bony erosions, and osteopenia are the findings of rheumatoid arthritis on x-ray

174
Q

A 50 year-old man presents with a sore knee. The knee is red, hot, and tender to touch, and there is pain on passive movement. He is apyrexic. Fluid is aspirated from the knee an no crystals are seen on polarised light microscopy. he has a history of rheumatoid arthritis for which he takes methotrexate.

What is the most likely diagnosis?
A. Pseudogout
B. Gout
C. Septic arthritis
D. Psoriatic arthritis
E. Osteoarthritis
A

C. Septic arthritis

Septic arthritis is characterised by redness, swelling, and tenderness, usually in a single joint which also exhibits pain on passive movement. The patient may or may not be systemically unwell. Definitive diagnosis is done by analysing joint aspirate.

Risk factors for septic arthritis include: rheumatoid arthritis, diabetes, I.V. drug use, unsafe sex, skin infection, prosthetic joints, and chronic gout.

Neisseria gonorrhea is a common cause of septic arthritis in younger patients who are sexually active, with no other risk factors for septic arthritis.

In a older patient, particularly in those with pre-existing arthritis, immunosuppression, I.V. drug use, skin infection, or prosthetic joints, staphylococcus aureus is the most likely causative organism

175
Q

A 26 year-old man presents with a sore knee, he has just returned from a business trip to Thailand. The knee is red, hot, and tender to touch, and there is pain on passive movement.

What is the most likely causative organism?
A. Staphylococcus aureus
B. Streptococcus pyogenes
C. Hepatitis A
D. Neisseria meningitides
E. Neisseria gonorrhoea
A

E. Neisseria gonorrhoea

Neisseria gonorrhea is a common cause of septic arthritis in younger patients who are sexually active, with no other risk factors for septic arthritis.

Septic arthritis is characterised by redness, swelling, and tenderness, usually in a single joint which also exhibits pain on passive movement. The patient may or may not be systemically unwell. Definitive diagnosis is done by analysing joint aspirate.

Risk factors for septic arthritis include: rheumatoid arthritis, diabetes, I.V. drug use, unsafe sex, skin infection, prosthetic joints, and chronic gout.

In a older patient, particularly in those with pre-existing arthritis, immunosuppression, I.V. drug use, skin infection, or prosthetic joints, staphylococcus aureus is the most likely causative organism

176
Q

A 17 year-old schoolgirl presents to her GP with fever, malaise, and a sore throat. O/E her tonsils are inflamed with a grey exudate, her anterior cervical lymph nodes are swollen, and her spleen is enlarged. A heterophile antibody test confirms the diagnosis.

What disease is the patient now at increased future risk of?
A. Breast cancer
B. Chronic lymphocytic leukaemia
C. Hodgkin's lymphoma
D. Acute myeloid leukaemia
E. TB
A

C. Hodgkin’s lymphoma

This is a history of glandular fever (confirmed by a heterophile or Paul Bunnel test), which is caused by Epstein-Barr virus. A previous infection with Epstein-Barr virus is an important element in a history of someone with possible Hodgkin’s lymphoma. EBV also predisposes to Burkitt’s lymphoma.

177
Q

A 56 year-old diabetic patient attends his GP practice for a check-up. His blood pressure is 140/80, but his Hba1c is higher than you would like. Fundoscopy shows hard exudates, microaneurysms, blot haemorrhages, and cotton wool spots.

What form of retinopathy is this most likely to be?
A. Background diabetic retinopathy
B. Grade II hypertensive retinopathy
C. Pre proliferative diabetic retinopathy
D. Grade III hypertensive retinopathy
E. Proliferative diabetic retinopathy

A

C. Pre proliferative diabetic retinopathy

Hard exudates, micro aneurysms and blot haemorrhages are all features of background diabetic retinopathy, but the presence of cotton wool spots indicates that this is pre-proliferative diabetic retinopathy. The presence of new blood vessels would indicate proliferative retinopathy.
Hypertensive retinopathy has four grades:
Grade I: twisting of vessels, silver wiring
Grade II: AV nipping
Grade III: cotton wool spots, flame haemorrhages
Grade IV: papilloedema

178
Q

A patient is brought to A&E with palpitations. An ECG shows a regular, narrow-complex tachycardia with no visible p-waves. The patient describes an initial feeling of breathlessness that he tried to treat with his inhaler, but says it had no effect. Vagal manouevres have failed to slow his heart. BP is 120/78.

What is the most appropriate next step?
A. Amiodarone
B. Verapamil
C. DC cardioversion
D. Magnesium
E. Adenosine
A

B. Verapamil

Usually the treatment for supraventricular tachycardia begins with vagal manoeuvres, then proceeds to adenosine, and dc cardioversion if the patient is haemodynamically unstable.
Adenosine is contraindicated in asthmatics, in which case verapamil should be used instead.
Amiodarone is used in the treatment of broad complex tachycardia.
Magnesium is given for broad complex tachycardia particularly when the ECG suggests Torsades de Pointes, as low magnesium can cause arrhythmia.

179
Q

A 60 year-old woman presents with SOB, dizziness, and a pink frothy cough that have come on gradually over the past 2 months. An ECG is normal, but an echocardiogram reveals reduced ejection fraction. The echo allows you to diagnose aortic regurgitation.

Which of the following signs is not associated with aortic regurgitation?
A. Visible fluctuation in colour of the nail beds
B. A pan-systolic murmur in the aortic region
C. A water hammer pulse
D. A soft diastolic murmur heard best at the apex
E. An early diastolic murmur heard best at the left sternal edge in the 3rd space.

A

B. A pan-systolic murmur in the aortic region

Visible fluctuation in colour of the nail beds = Quincke’s sign.

A water hammer pulse = Corrigan’s pulse/ collapsing pulse.

A soft diastolic murmur heard best at the apex = Austin-Flint murmur, occurs in severe aortic regurgitation when blood jets back through to the mitral valve and prevents it closing, causing a similar murmur to mitral stenosis.

An early diastolic murmur heard best at the left sternal edge in the 3rd space = the classic murmur of aortic regurgitation that is tested for in a cardio exam by asking the patient to lean forward an exhale fully while listening at the left sternal edge in the 3rd intercostal space.

180
Q

A 54 year-old known alcoholic man presents with abdominal pain. His abdomen is distended and there is shifting dullness, though his notes tell you this is nothing new. He looks distinctly unwell and is feverish. He admits to some nausea and has vomited twice. Throughout your interaction with him, he seems confused.

What is the most likely diagnosis?
A. Hepatitis A
B. Bowel obstruction
C. Hepatitis B
D. Spontaneous bacterial peritonitis
E. Nephrotic syndrome
A

D. Spontaneous bacterial peritonitis

Liver cirrhosis increases the resistance of liver tissue to blood flow, which increases the pressure in the hepatic portal vein causing fluid movement into the abdomen. To compound this, cirrhotic livers tend not to function well and produce less albumin. Hypoalbuminaemia decreases the oncotic pressure within the portal vein, allowing even more fluid to leave and enter the abdomen.

Patients with ascites are vulnerable to infection of the ascitic fluid (e.g. by E.coli and Klebsiella), leading to spontaneous bacterial peritonitis. Paracentesis is used to diagnose SBP, and the fluid is analysed - if the fluid contains >250 neutrophils per microlitre the diagnosis is confirmed.

The confusion described in this scenario is due to hepatic encephalopathy, which is triggered by SBP in a patient with cirrhosis, and can sometimes be the only manifestation of SBP.

181
Q

A 55 year old woman who was recently admitted to hospital with a severe viral illness starts presenting with mucocutaneous bleeding and petchiae. She has no previous history of this before the illness.

What is the most likely diagnosis?
A. Von Willebrand disease
B. Bernard-Soulier syndrome
C. Immune thrombocytopenic purpura
D. Disseminated intravascular coagulation
E. Deep vein thrombosis
A

C. Immune thrombocytopenic purpura

ITP is an autoimmune reaction against platelets that is defined as thrombocytopenia in the absence of a known cause.
Risk factors include: female gnder, preceding viral illness, hepatitis C, immunosuppression (e.g. HIV), and other autoimmune disorders.

182
Q

A 76-year-old woman is brought into A&E with central crushing chest pain that radiates to her jaw and left arm. An ECG is performed, which shows ST elevation in leads ll, lll and aVF. Her SaO2 is 92% on room air. Before she is sent to the cathlab for percutaneous coronary intervention, she is started on a combination of drugs.

Which of the following should not be routinely given? 
A. Morphine 
B. Oxygen 
C. Aspirin 
D. Clopidogrel 
E. Warfarin
A

E. Warfarin

The standard treatment of a STEMi is:

Oxygen if there is evidence of hypoxia
Diamorphine and metoclopramide for pain management
Nitrates for ischaemic symptom relief
Aspirin & clopidogrel to prevent further platelet clotting

Patients should undergo PCI if within 12 hours of presentation of symptoms, and those patients undergoing PCI should also receive unfractionated heparin or a LMWH (e.g. enoxaparin). If PCI cannot be delivered within 120 minutes of the time that fibrinolysis can be delivered, fibrinolysis should be used along with one of unfractionated heparin, LMWH, or Fondaparinux.

Coronary angiography with follow on PCi should still be offered for patients presenting >12 hours after presentation of symptoms if there is evidence of continuing myocardial ischaemia. If a patient will not be receiving reperfusion therapy, one of unfractionated heparin, LMWH, or Fondaparinux should be given. Glycoprotein IIb/IIIa inhibitors may also be given in high risk patients (e.g. abciximab)

Long-term for secondary prevention aspirin and clopidogrel will be continued indefinitely, and a beta blocker, a statin, and an ACEi will be added.

NB: be careful of giving opiates to desaturating patients, as it could further decrease their oxygenation. however in this case the patient is about to be sent for surgery, and so will be ventilated soon anyway.

183
Q

A 54-year-old man has been brought into A&E with a suspected acute coronary syndrome. An ECG is performed, which reveals ST elevation in leads I, aVL, V5 and V6.

Which coronary artery has been occluded?
A. Left main stem
B. Left anterior descending coronary artery
C. Left circumflex coronary artery
D. Right coronary artery
E. Posterior descending artery

A

C. Left circumflex coronary artery

The territories of the heart are as follows:
Septal anterior: V1, V2 Left anterior descending artery
Anterior: V3, V4 Left anterior descending artery
Lateral: V5, V6, I, aVL Left circumflex artery
Inferior: II, III, aVF Right coronary artery

Posterior: tall R-waves and ST-depression in V1-V3, usually supplied by a branch of the right coronary artery

184
Q

A 54-year-old man is complaining of sharp, central chest pain that has arisen over the last 24 hours. On inspection, the patient is sitting forward on the examination couch. On auscultation, a scratching sound is heard – loudest over the lower left sternal edge, when the patient is leaning forward. He has a past medical history of a ST-elevation MI which was diagnosed, and treated with PCI, 6 weeks ago.

What is the most likely diagnosis?
A. Viral pericarditis
B. Constrictive pericarditis
C. Cardiac tamponade
D. Dressler syndrome
E. Tietze syndrome
A

D. Dressler syndrome

Dressler syndrome is pericarditis post-MI, believed to be caused by an autoimmune inflammatory reaction. it is typically self-resolving, and treatment is usually just aspirin.

185
Q

A 27-year-old man presents complaining of sharp chest pain. He mentions that he has taken a few days off work recently because of the flu.

What would you expect to see on his ECG?
A. ST elevation in leads II, III and aVF
B. Widespread saddle-shaped ST elevation
C. ST depression
D. Tented T waves
E. Absent P waves

A

B. Widespread saddle-shaped ST elevation

The sharp chest pain in combination with recent viral illness indicates pericarditis. This causes widespread saddle-shaped ST elevation on an ECG. A pericardial rub may be heard on auscultation of the heart, and the pain will typically be improved by sitting forward.

186
Q

A 46-year-old man has been admitted to A&E after experiencing palpitations, which began about 4 hours ago. An ECG is performed, which reveals atrial fibrillation. He has previously had a STEMI. He refuses DC cardioversion.

What is the next most appropriate treatment option? 
A. Amiodarone 
B. Low molecular weight heparin 
C. Warfarin 
D. Flecainide 
E. Digoxin
A

A. Amiodarone

In a patient who has been experiencing symptoms of atrial fibrillation for <48 hours, there is a low probability of a clot having formed in the atria in that time, so it is safe to attempt cardioversion as there is a low risk of embolisation. Cardioversion can be attempted electrically (DC cardioversion) or chemically (flecainide, unless there is known ischaemic heart disease, in which case amiodarone should be used)

In a patient who has been experiencing symptoms of atrial fibrillation for >48 hours, there is too high a chance that a clot has formed in the atria, so it is not safe to attempt cardioversion as there is a chance of embolisation. Instead, the patient should be anti-coagulated for 3-4 weeks with warfarin, and rate control should be achieved with beta blockers and digoxin. After 3-4 weeks, it is safe to attempt cardioversion.

187
Q

A 27-year-old man presents with palpitations and light-headedness. An ECG shows features consistent with a supraventricular tachycardia. Adenosine is administered and the SVT is terminated. A repeat ECG shows a short PR interval and a QRS complex with a slurred upstroke.

What is the diagnosis? 
A. Brugada syndrome 
B. LBBB 
C. Romano-Ward syndrome 
D. Wolff-Parkinson-White syndrome 
E. Complete heart block
A

D. Wolff-Parkinson-White syndrome

WPW is due to an accessory pathway in the heart - the bundle of Kent - that allows abnormally fast depolarisation of the ventricles, resulting in the slurred upstroke appearance (delta wave) on an ECG. If impulses travel back up through the accessory pathway towards the atria, an AV reentrant tachycardia can start, as the impulses become independent of the SA node.

188
Q

A 52-year-old man was watching TV yesterday when he suddenly become very aware of his heart beating rapidly. This lasted for around 45 mins and then subsided spontaneously. It has happened several times over the past 2 months. An ECG reveals no abnormalities. However, due to the strong suspicion of atrial fibrillation, the patient is placed on a 24-hr tape, which confirms the diagnosis.

Which scoring system should be used to determine the benefit of long-term anti-coagulation in this patient? 
A. QRISK2 score 
B. ABCD2 Score 
C. GRACE score 
D. CHA2DS2-VASc score 
E. CURB-65 score
A

D. CHA2DS2-VASc score

This score assess the risk of stroke in a patient with atrial fibrillation. A score of 1 or greater indicates that anti-coagulation would be appropriate.

189
Q

A 21-year-old woman has fainted 4 times in the past 3 months. She becomes sweaty and nauseous before she faints and is usually unconscious for a few seconds. Her friends have told her that she looks abnormally pale before she collapses. She doesn’t know if she jerks whilst unconscious, but has not lost control of her bladder or bitten her tongue. When she regains consciousness, she feels slightly dizzy but does not feel confused.

What is the most likely cause of her fainting?
A. Hypoglycaemia
B. Epileptic seizure
C. Vasovagal syncope
D. Arrhythmia
E. Hypertrophic obstructive cardiomyopathy

A

C. Vasovagal syncope

In the absence of signs that would indicate other differentials of collapse (aura, palpitations, post-ictal phase) the most likely differential is vasovagal syncope. When taking a history of collapse, it is always important to thoroughly explore the before (aura, palpitations), during (LOC, incontinence, tongue biting), and after (post-ictal phase) of the collapse, and also to enquire about missed meals.

190
Q

A 52-year-old man has collapsed 3 times in the past couple of months. His father died of a heart condition when he was 56 years old, although he cannot recall the details of the condition. On examination, a jerky carotid pulse is palpated and a crescendo-decrescendo murmur is heard that doesn’t radiate to the carotid arteries.

What is the most likely diagnosis? 
A. Aortic stenosis 
B. Hypertrophic obstructive cardiomyopathy 
C. Left heart failure 
D. Mitral regurgitation 
E. Constrictive pericarditis
A

B. Hypertrophic obstructive cardiomyopathy

Collapse in a patient with a family history of sudden early death should make you think of cardiac abnormalities such as long QT syndrome and HOCM.
Signs of HOCm include a jerky carotid pulse, a double apex beat, and an ejection systolic murmur that DOES NOT radiate to the carotids.

The reason for this is that the septum is enlarged in HOCM, which narrows the left ventricle outflow tract leading to a suction effect (Venturi effect) that prevents the mitral valve from closing during systole. This effectively creates mitral regurgitation, with an associated murmur, but only during systole. This murmur will therefore not radiate to the carotids. This explains why the murmur is crescendo-decrescendo, instead of being a fixed sound - the functional mitral regurgitation is dependent on the flow velocity (which creates the Venturi effect).

191
Q

A 76-year-old man is found collapsed in the care home and has a suspected hip fracture. He says that he temporarily lost consciousness as he got up from his arm chair and came about, a matter of seconds later, on the floor. He has never experienced a fall before. He has a past medical history of a total knee replacement and heart failure which is treated with ramipril, furosemide and bisoprolol.

What is the most likely cause of his collapse?
A. Vasovagal syncope 
B. Medication side-effect
C. Arrhythmia 
D. Anaemia
E. Dilated cardiomyopathy
A

B. Medication side-effect

Medications to control blood pressure can be too effective (particularly diuretics) - causing hypotension and collapse.

192
Q

A 14 year-old girl is brought to A&E by her father after she noticed blood in her urine. She also says she has been urinating less. A urine dipstick confirms haematuria with no other abnormalities, and upon taking obs you notice her blood pressure is elevated. She has never been admitted to hospital and has only had to visit her GP once for a sore throat a month ago.

What is the most likely diagnosis?
A. Minimal change disease
B. IgA nephropathy
C. Glomerulonephritis
D. Nephrotic syndrome
E. Goodpasture's syndrome
A

C. Glomerulonephritis

This is post-Streptococcal glomerulonephritis, which is a result of a type III hypersensitivity reaction (complement activation and immmune complex formation) causing inflammation in the kidneys. This leads to glomerulonephritis which can cause hypertension, oedema, haematuria and oligouria.
It can sometimes be difficult to differentiate post streptococcal glomerulonephritis from IgA nephropathy.
IgA nephropathy is the most common cause of glomerulonephritis worldwide and presents with symptoms within a few days of an upper respiratory tract infection. Glomerulonephritis typically takes around a month to develop after an infection.

193
Q

A 34 year old woman presents with sudden onset chest pain. An ECG shows sinus tachycardia, but no ST changes. Blood pressure is normal, though resp rate is elevated. A closer inspection of the patient reveals a swollen and tender left calf. A Wells score shows a high probability of PE and dalteparin is given immediately. CTPA confirms a PE.

What is the most appropriate next step?
A. I.V. thrombolysis
B. Thrombolectomy
C. Add fondaparinux
D. Add warfarin
E. Give 300mg loading dose aspirin
A

D. Add warfarin

Warfarin is used in patients with no contraindications for at least 3 months after a pulmonary embolism. It is not given on its own at first, because the initial effect is pro-thrombotic: warfarin reduces protein C and S levels, reducing degradation of activated clotting factors. Though the anticoagulant effects are greater, they take longer to manifest - the body has to run out of 2, 7, 9, and 10 for the effect to be apparent as warfarin inhibits vitamin k recycling, preventing manufacture of new factor 2, 7, 9, and 10

I.V. thrombolysis or thrombolectomy is only given if the patient is haemodynamically unstable.
Fondaparinux can be used instead of LMWH as a first step.
300mg aspirin is an immediate step taken in management of acute coronary syndrome.

194
Q

A 59 year-old man presents with increased urinary frequency over the past 3 months, He says his stream is quite slow and takes a while to get going. Afterwards he describes terminal dribbling and a feeling of incomplete emptying. DRE reveals a smooth, bilaterally enlarged prostate with a palpable midline sulcus.

What is the most appropriate treatment?
A. Hydroxocobalamin
B. Sotalol
C. Tamulosin
D. Trimethoprim
E. Furosemide
A

C. Tamulosin

Tamulosin is an alpha blocker which relaxes the smooth muscle of the internal urinary sphincter and prostate capsule, making urinating easier.

195
Q

A 22 year-old man, who is named Shoe for some reason, presents to A&E after a rugby match. He describes severe scrotal pain that forced him to leave the field and vomit. O/E the left hemiscrotum looks swollen and inflamed.

What is the most appropriate next step
A. Doppler ultrasound
B. MRI
C. CT scan
D. Analgesia and wait
E. Exploratory surgery
A

E. Exploratory surgery

This is a history of testicular torsion, which is a surgical emergency. If not dealt with within 6 hours, the ischaemic damage will result in loss of the testis. Whilst a Doppler USS is an effective investigation, this is a clear history and unless you can organise a Doppler immediately, you should proceed directly to surgery.

196
Q

A 29 year-old man visits his GP after finding a testicular lump. On palpation the lump is firm and non-tender. The patient underwent orchidoplexy as a child, to correct an undescended testis. The GP suspects testicular cancer.

Which of the following tests would  most likely be abnormal in testicular cancer?
A. Beta hCG
B. Ca19-9
C. CRP
D. Ca53
E. Platelet count
A

A. Beta hCG

Beta hCG is raised in pregnancy, and is used in pregnancy tests but can also be secreted by testicular malignancies. Alpha fetoprotein can also indicate testicular cancer (or liver cancer).

197
Q

A 53 year old man suffers a ruptured aortic
aneurysm and is rushed into theatre. He
undergoes a successful operation and is
recovering on the wards. 1 day after the operation
he becomes oliguric with elevated urea and
creatinine. After 1 week his urine output
increases but his GFR remains low at 30ml/min.

What is the most likely diagnosis?
A. Acute tubular necrosis
B. Rhabdomyolysis
C. Glomerulonephritis
D. Haemolytic-uraemic syndrome
E. Haemorrhage
F. Renal stones
G. Thrombotic thrombocytopenic purpura
H. Sepsis
I. Vasculitis
J. Dehydration
A

A. Acute tubular necrosis

The hypovolaemia resulting from the bleeding has presumably caused ischaemic damage to the kidneys, causing necrosis.

198
Q

A 10 year old girl presents to A&E with
irritability, abdominal pain and reduced urine
output. Her parents says she has had diarrhoea
for the last few days.

What is the most likely diagnosis?
A. Acute tubular necrosis
B. Rhabdomyolysis
C. Glomerulonephritis
D. Haemolytic-uraemic syndrome
E. Haemorrhage
F. Renal stones
G. Thrombotic thrombocytopenic purpura
H. Sepsis
I. Vasculitis
J. Dehydration
A

D. Haemolytic-uraemic syndrome

This is a history of haemolytic-uraemic syndrome (triad of microangiopathic haemolytic anaemia, thrombocytopenia, and acute renal impairment/ uraemia) which is most common in children, and generally presents with nausea, fatigue, and potentially bleeding, within a few days of gastrointestinal infection. The most common causative organism is E. coli (particularly serotype 0157:H7), with the second most common being Shigella.

199
Q

A young woman presents with a raised urea and
creatinine after a seizure. She is noted to have a fever
and icterus. Her urine output is low.

What is the most likely diagnosis?
A. Acute tubular necrosis
B. Rhabdomyolysis
C. Glomerulonephritis
D. Haemolytic-uraemic syndrome
E. Haemorrhage
F. Renal stones
G. Thrombotic thrombocytopenic purpura
H. Sepsis
I. Vasculitis
J. Dehydration
A

G. Thrombotic thrombocytopenic purpura

TTP is typically associated with antibodies against an enzyme that cleaves Von-Willebrand factor, leading to an abnormally high amount of VWf in the blood. This causes clotting in small blood vessels, and a consequent thrombocytopenia leading to bleeding.

Microangiopathic haemolytic anaemia is a feature of TTP which is caused by damage to RBCs as they pass through vessels that have been occluded by thrombi. This causes the RBCs to shear, resulting in a haemolytic anaemia, which may also cause s pre-hepatic jaundice.

Haemolytic-uraemic syndrome may present similarly to TTP, but usually occurs in children and will feature a history of gastrointestinal infection.

The classic pentad of TTP rarely occurs together but is as follows:
Fever
Microangiopathic haemolytic anaemia
Mental state changes
Renal impairment
Thrombocytopenia
200
Q

A 84 year old woman is found on the floor of her
flat by her neighbour. She had a fall 3 days prior to
her ‘rescue’ and had been unable to get up or raise the
alarm. At hospital, she is assessed and found to have
acute kidney injury.

What is the most likely diagnosis?
A. Acute tubular necrosis
B. Rhabdomyolysis
C. Glomerulonephritis
D. Haemolytic-uraemic syndrome
E. Haemorrhage
F. Renal stones
G. Thrombotic thrombocytopenic purpura
H. Sepsis
I. Vasculitis
J. Dehydration
A

B. Rhabdomyolysis

Rhabdomyolysis (literally translated from Greek as ‘striped muscle breakdown’) is breakdown of skeletal muscle, often as a consequence of trauma. In this case the patient has sustained a fall and been immobilised on the floor for several days. This has caused a crush injury, leading to breakdown of the crushed skeletal muscle.

Skeletal muscle releases myoglobin when it breaks down, which is toxic to the kidneys. Creatine kinase is also released from skeletal muscle when it breaks down, and so can be used to test for potential rhabdomyolysis in cases like this one.

201
Q

A 72 year old gentleman is admitted to the ward
following a five day history of diarrhoea and
vomiting. The patient has become more confused and
has a low urine output. These symptoms improve with
administration of fluids.

What is the most likely diagnosis?
A. Acute tubular necrosis
B. Rhabdomyolysis
C. Glomerulonephritis
D. Haemolytic-uraemic syndrome
E. Haemorrhage
F. Renal stones
G. Thrombotic thrombocytopenic purpura
H. Sepsis
I. Vasculitis
J. Dehydration
A

J. Dehydration

This is a fairly clear case of dehydration leading to confusion. Dehydration should always be a differential when an elderly patient presents with confusion, even without diarrhoea and vomiting, as elderly patients are often less mobile or cognitively impaired, and so might not be able to keep themselves hydrated.

202
Q

A 6 year old girl presents to A&E with fever and abdominal pain. She is confused, has swollen ankles and has been vomiting. FBC/Obs shows anaemia, fever, a raised WCC, and low platelets. Lactate and blood pressure are normal.

Which is the most likely responsible organism?
A. Shigella
B. Klebsiella
C. Streptococcus pyogenes
D. Escherichia coli
E. Staphylococcus aureus
A

D. Escherichia coli

This is a history of haemolytic-uraemic syndrome (triad of microangiopathic haemolytic anaemia, thrombocytopenia, and acute renal impairment) which is most common in children, and generally presents with nausea, fatigue, and potentially bleeding, within a few days of gastrointestinal infection. The most common causative organism is E. coli (particularly serotype 0157:H7), with the second most common being Shigella.

203
Q

A 5-year-old boy presents with a short history of facial oedema that has now
progressed to total body swelling involving the face, abdomen, scrotum, and
feet. A urine dip shows protein +++.

What is the most likely diagnosis?
A. Minimal Change Disease
B. IgA Nephropathy
C. Post Streptoccocal
D. Membranoproliferative
Glomerulonephritis
E. Focal Segmental Glomerulosclerosis
A

A. Minimal Change Disease

Minimal change disease is characterised by loss of podocyte foot processes seen on electron microscopy of a kidney biopsy. It causes a nephrotic syndrome and most commonly affects children, and is the most common cause of renal disease in children.

204
Q

A 25 year old lady comes to A&E extremely worried as she has noticed blood in her urine. She tells you that she has had a bit of a cold for the
last two days.

What is the most likely diagnosis?
A. Minimal Change Disease
B. IgA Nephropathy
C. Post Streptoccocal
D. Membranoproliferative
Glomerulonephritis
E. Focal Segmental Glomerulosclerosis
A

B. IgA Nephropathy

IgA nephropathy causes a nephritic syndrome (proteinuria, haematuria, oligouria, hypertension) within a few days of an upper respiratory tract infection.

205
Q

An 8 year old girl attends the A&E with a rash on her buttocks, around
her elbows and on her legs. This was preceded by sudden onset
abdominal pain. A urine dip reveals haematuria.

Which antibody is
likely to be responsible for her condition?
A. Anti-GBM
B. IgG
C. IgA
D. P-ANCA
E. C-ANCA
A

C. IgA

Henoch Schonlein purpura is an IgA mediated vasculitis characterised by a triad of arthritis, purpura, and abdominal pain. There may also be renal involvement causing haematuria.

206
Q

22 year old man has noticed smaller volume of darker urine. He has not felt quite right since a painful sore throat about a month ago.

BP 145/90 Urea 8.8, creatinine 116, albumin 45g/l. Urine dip protein
1+, blood 2+ There is a low C3 noted (complement).

What is the most likely diagnosis?
A. IgA Nephropathy
B. Post Streptoccocal
C. Bladder Carcinoma
D. Focal Segmental Glomerulosclerosis
E. UTI
A

B. Post Streptoccocal

Post streptococcal glomerulonephritis is a nephritic syndrome that follows an upper respiratory tract infection. Renal symptoms usually follow infection by 4-6 weeks in contrast to IgA nephropathy where symptoms follow infection by a few days.
Post-streptococcal glomerulonephritis involves complement-mediated immune complex deposition, hence there will be low C3 because complement has been depleted.
There would also be a positive anti-streptolysin O titre (measurement of antibodies made against streptococcal infection).

207
Q

A 45 year old man has presented to the GP with dark urine. He has been
becoming progressively more tired over the last few weeks. On direct questioning, he reveals he has coughed up blood a few times.

What is the most likely diagnosis?
A. Minimal Change Disease
B. Focal Segmental Glomerulosclerosis
C. Membranous Disease
D. Goodpasture’s Syndrome
E. IgA Nephropathy
A

D. Goodpasture’s Syndrome

Goodpasture’s syndrome is an autoimmune condition defined by anti glomerular basement membrane antibodies (targeting type IV collagen) that damage the basement membrane in the alveoli and glomeruli. Involvement of both the lunga and kidneys (without upper airway involvement - nosebleeds, rhinitis) should prompt thoughts of Goodpasture’s syndrome.

208
Q

A 27 year old man has had regular nosebleeds for the past 3 months. He has also noticed some blood in the urine.
BP 140/90, Urea 11mmol/l, Creatinine 166, albumin 37. Urine dip blood 3+. C-ANCA +ve

What is the most likely diagnosis?
A. Goodpasture’s syndrome
B. Microscopic polyangitis
C. Focal Segmental Glomerulosclerosis
D. Granulomatosis with polyangitis
E. Nephritic Syndrome
A

D. Granulomatosis with polyangitis

Granulomatosis with polyangitis is a vasculitic disease comprised of a triad of upper respiratory tract symptoms (rhinitis, nosebleeds), lower respiratory tract symptoms (haemoptysis), and renal symptoms (glomerulonephritis).
There are few diseases that link pulmonary and renal symptoms, particularly in the absence of other organ involvement.
The main other differential for the oresentation in this question is Goodpasture’s syndrome. The discriminating factor is that Goodpasture’s syndrome does not cause upper repiratory tract inbvolvement (e.g. nosebleeds). There may also be other factors of granulomatosis in an SBA that help to differentiate it (cANCA positive, cavitating lesions on CXR, saddle-nose).

209
Q

A 27 year old man has had regular nosebleeds for the past 3 months. He
has also noticed some blood in the urine.
BP 140/90, Urea 11mmol/l, Creatinine 166, albumin 37. Urine dip
blood 3+. C-ANCA +ve

What other feature would support the diagnosis?
A. Loss of podocytes
B. No changes
C. Crescents on light microscopy
D. Raised ASOT
E. Diffuse immune complex deposition
A

C. Crescents on light microscopy

Granulomatosis with polyangitis is a vasculitic disease comprised of a triad of upper respiratory tract symptoms (rhinitis, nosebleeds), lower respiratory tract symptoms (haemoptysis), and renal symptoms (glomerulonephritis).
It is one of the causes of rapidly progressive glomerulonephritis (the other main ones being Goodpasture’s syndrome, and microscopic polyangitis) which is also known as cresenteric glomerulonephritis because it causes crescent-shaped scarring visible on microscopic examination of a biopsy.

210
Q

A 35 year-old man presents to his GP because of a series of mouth ulcers that have been recurring over the past few months. He also admits sheepishly that he has noticed ulcers on his penis, which he thinks may be linked to the unprotected sex he’s been having. On further questioning there is no discharge or pain on urinating. You notice that his eyes seem bloodshot.

What is the most likely diagnosis?
A. Chlamydia trachomatis infection
B. Crohn's disease
C. Neisseria gonorrhea infection
D. Treponema pallidum infection
E. Behcet's syndrome
A

E. Behcet’s syndrome

Behcet’s syndrome is a vasculitide defined by a triad of recurrent oral ulcers, genital ulcers, and uveitis.

211
Q

A 25 year old female returns from elective in India. She has had diarrhoea after eating a fish curry three days ago. She is also feverish, with a slight yellowing to her eyes.

What is the most likely diagnosis?

A. Gallstones
B. Hepatitis B
C. Hepatitis C
D. Hepatitis A
E. Ascending cholangitis
A

D. Hepatitis A

Hepatitis A is transmitted via the faeco-oral route and is stereotypically acquired after eating contaminated shellfish. It causes an acute infection which has led to the jaundice and fever described here

212
Q

An overweight, 30 y/o female presents with a history of right sided abdominal pain, worse after eating fatty foods. It is not relieved by changing position. She has tried paracetamol for the pain which seems
to help a little. She is otherwise well and talkative.

Which is the best investigation to confirm her diagnosis?

A. Abdominal examination
B. CT-KUB
C. Liver biopsy
D. USS of biliary tree
E. ERCP
A

D. USS of biliary tree

This is a history of cholecystitis, the giveaways being the tendency of the pain to worsen when eating fatty foods. The 5f’s are the risk factors of cholecystitis to remember: fat (BMI>30), fertile (has had children), forty (older than 40), fair (Caucasian), and female. The biliary tree can be well visualised with an USS, so it is commonly used to diagnose biliary issues as it is non-invasive

213
Q

A 73-year-old woman attends the GP because of a ‘spot’ on her nose that isn’t healing. The lesion is nodular and flesh-coloured with a pearly rolled edge, surface telangiectasia, and central ulceration.

What is the most likely diagnosis?

A. Squamous cell carcinoma
B. Malignant melanoma
C. Rodent ulcer
D. Seborrheic wart
E. Intradermal naevi
A

C. Rodent ulcer

A rodent ulcer is another name for a basal cell carcinoma (this seems a bit of a trick but has come up before). A basal cell carcinoma has a very distinctive appearance: flesh coloured, with rolled edges, a pearly appearance, and visible telangectasia are all characteristic. A basal cell carcinoma rarely invades or metastasises and grows slowly, so this can be managed with a routine dermatology referral for removal.

214
Q

A 63-year-old man is brought into the A&E department by his wife
complaining of a central crushing chest pain. An ECG is performed
which shows ST elevation in leads II, III and aVF.

Which vessel is probably being occluded?

A. Right Coronary Artery
B. Left Anterior Descending Coronary Artery
C. Circumflex Coronary Artery
D. Posterior Descending Coronary Artery
E. Left Main Stem
A

A. Right Coronary Artery

This is important to learn as it will definitely be a question in the exam, so memorise which leads monitor which regions of the heart, and which arteries correspond:

Septal - V1, V2
Anterior - V3, V4
Lateral - V5, V6, I, aVL
Inferior - II, III, aVF

Septal and anterior regions are supplied by the left anterior descending artery
The lateral region is supplied by the circumflex artery
The inferior region is supplied by the right coronary artery

215
Q

A 52 year old male with COPD presents to A&E with sudden onset
breathlessness and left sided chest pain. On examination, the patient has reduced chest expansion and breath sounds. A chest X-ray shows a 3cm left sided pneumothorax.

What is the most appropriate management?

A. High Flow O2
B. Aspiration
C. Watch and wait
D. Intercostal Drain
E. Immediate needle decompression
A

D. Intercostal Drain

Management of a pneumothorax depends on its size, and whether it is primary (no underlying causative condition), or secondary (e.g. to COPD).

Primary (no underlying cause):
<2cm: discharge and repeat CXR later
>2cm/ SOB: aspiration (if unsuccessful - chest drain)

Secondary (known cause e.g. COPD)
>2cm: aspiration (if unsuccessful - chest drain)
<2cm: chest drain

216
Q

Mr. Z is a 70 year old non-smoker who presents to the GP with SOB associated with a dry cough, fatigue, fever, sweats, and weight loss developing over the last few months. He is a retired ship-builder and spends most of his days at home with his wife. Physical examination
reveals decreased breath sounds in the right lung base associated with dullness to percussion.

What is the most likely diagnosis?

A. Tuberculosis
B. Small cell lung carcinoma
C. Bronchiectasis
D. Cystic Fibrosis
E. Mesothelioma
A

E. Mesothelioma

More than 80% of mesothelioma cases are due to asbestos exposure. Ship-building is one occupation in which the patient would have been exposed to asbestos. The patient has symptoms indicating cancer, but small-cell lung carcinoma is unusual in a non-smoker. TB is a possible differential, but the cough is non-productive and there is no travel history to indicate exposure.

217
Q

Mr. M is a 40 year old male recently returned from South East Asia presents to the GP with a 2 month history of weight loss, night sweats, intermittent fever and dry cough. On auscultation you hear crackles and bronchial breathing and on further inspection you discover the presence of erythema nodosum. You request further investigations to confirm your diagnosis.

What is the most likely result that confirms your diagnosis?

A. Negative Nucleic acid amplification test (NAAT)
B. Positive HIV test
C. Positive sputum smear showing acid fast bacilli
D. Low haemoglobin on FBC
E. Normal chest X ray

A

C. Positive sputum smear showing acid fast bacilli

This is a history of TB, with a history of recent travel to an endemic region, and infective symptoms with night sweats and weight loss. Staining using a Ziehl-Neelson stain will reveal acid-fast bacilli in the case of TB.

218
Q

A 75-year-old male with shortness of breath and a fever is admitted to the Acute Medical Unit. On examination the patient appears thin and is sweaty and warm to touch. On his finger nail you see a dark red line.
When listening to his heart you hear an early diastolic murmur. His past medical history indicates renal failure for which he has had a transplant.

What is the most appropriate next step in his management?

A. Take blood cultures
B. Urinalysis
C. Check GFR and U&amp;Es
D. Perform an echocardiogram
E. Give antibiotics
A

A. Take blood cultures

This is a history of infective endocarditis in a patient who has presumably been taking immunosuppressive drugs after his renal transplant. This has left him vulnerable to infection which has caused the fever, splinter haemorrhage, and diastolic murmur. Blood cultures will allow identification of the responsible organism.

219
Q

A 42 year old women presents with bilateral mild breast pain. Pain is worse just before the start of her menstrual period, and improves a few days after. She has 2 children and is not on any oral contraceptives. O/E there is diffuse nodularity throughout both breasts.

What is the most likely diagnosis?

A. Duct ectasia
B. Fat necrosis
C. Fibrocystic changes
D. Breast cancer
E. Bilateral fibroadenomas
A

C. Fibrocystic changes

Fibrocystic changes (also known as fibroadenosis) present with diffuse lumps within the breast that are painful: the pain changes with the menstrual cycle

220
Q

A 56-year old retired builder presents to the GP with a 2-month history of shortness of breath and a dry cough. The gentleman claims to have never smoked, is afebrile and has no other constitutional symptoms. Upon
examination there is marked clubbing of both hands and bi-basal inspiratory crepitations are noted upon auscultation. A chest X-ray is performed which shows bi-basal reticulonodular shadowing.

What is the most likely diagnosis?
A. Squamous cell carcinoma of the lung
B. Large cell carcinoma of the lung
C. Bronchiectasis
D. Pneumoconiosis
E. Pneumonia
A

D. Pneumoconiosis

Pneumonoconiosis refers to pulmonary fibrosis caused by inhalation of dust into the lungs. Given the absence of a smoking history, both large cell carcinoma and squamous cell carcinoma are unlikely. Furthermore there are no infective signs that would support a diagonsis of pneumonia or bronchiectasis.

The findings of clubbing, reticulonodular shadowing, and bi-basal crepitations indicate fibrosis. Th previous occupation as a builder strongly suggests pneumonoconiosis

221
Q

A 42 year old female presents to A&E with sudden onset chest pain and shortness of breath. The pain is worse on inspiration. Cardiac and respiratory examinations are normal. ECG shows sinus tachycardia. HR 120bpm, RR 24.

What is the most likely diagnosis?

A. STEMI
B. Pneumothorax
C. PE
D. Aortic dissection
E. Musculoskeletal strain
A

C. PE

The most common ECG finding of PE is sinus tachycardia; right axis deviation can also be seen. The classic ECG finding of PE is a large S wave in lead I, a Q wave in lead III and an inverted T wave in lead III. However this pattern only actually appears in about 10% of cases.

Similarly, although there are certain CXR findings associated with PE, most only occur in a small proportion of cases.

This history of sudden onset pleuritic pain (worse on inspiration) with sinus tachycardia is strongly suggestive of PE.

222
Q

A 65-year-old male presents to his GP with persistent fatigue. A full blood count and haematinics are requested.
Hb: 95 (135-180)
MCV: 65 (80 – 100)
WCC: 5 (4 – 11)
Platelets: 300 (150 – 400).
Haematinics show a low serum iron, low ferritin, high transferrin and high transferrin saturation.

What is the next most appropriate step?

A. Haemoglobin electrophoresis
B. Measure CRP and ESR
C. Arrange for OGD and colonoscopy
D. Prescribe iron tablets
E. Test for IGA tissue transglutaminase antibodies
A

C. Arrange for OGD and colonoscopy

In a male patient, or a female patient who is not menstruating, unexplained anaemia is concerning, especially if it’s microcytic. That is because is may indicate a GI bleed, hence endoscopy is to be ordered in this case.

223
Q

A 27-year-old man presents to A&E with a painful scrotum. The pain has come on over the past 24 hours and it has become extremely painful, making it difficult for him to walk. On examination, his right testicle is swollen and extremely tender. You also notice some purulent discharge coming from his urethral meatus.

What is the most likely cause of his symptoms?

A. Escherichia coli
B. Neisseria meningitidis
C. Chlamydia trachomatis
D. Testicular torsion
E. Epididymal cyst
A

C. Chlamydia trachomatis

Though an acutely painful scrotum may prompt you to jump to testicular torsion, the onset of epididymitis can also be fast, and it will be very painful. Furthermore the presence of discharge implies an infective cause.

Epididymitis is an inflammation of the epididymis. In a young patient the most likely causative organisms are STIs (chlamydia, gonorrhea), whereas in an older patient the most likely organisms are enteric pathogens (E. coli, Klebsiella)

224
Q

A 90-year-old man is admitted to ITU with severe community acquired pneumonia, requiring cardiovascular support. His blood pressure regularly drops below 90/60 mmHg. He has a raised temperature, raised lactate. His urine output remains considerably reduced despite continuous fluid infusions. Microscopy of the urine reveals brown cell casts.

What is the most likely cause of the patient’s reduced urine output?

A. Pre-renal AKI
B. Post-renal AKI
C. Glomerulonephritis
D. Acute Tubular Necrosis
E. Acute Interstitial Nephritis
A

D. Acute Tubular Necrosis

This patient has experienced a prolonged period of time with low blood pressure (as a result of the sepsis from the pneumonia), which has caused ischaemic damage to his kidneys. The giveaway is the presence of the brown cell casts: these are a type of granular cast and will be typically described as muddy brown - they indicate acute tubular necrosis.

225
Q

A 32 year old man presents with pain and stiffness of the hands. This is particularly bad in the mornings, and eases with activity. His past medical history is notable for severe dandruff. Examination of the hands reveals joint inflammation, predominantly of the distal interphalangeal joints. You also note pitting of the nails, some of which are detached from the nail bed distally.

What is the most likely diagnosis?

A: Rheumatoid arthritis
B: Osteoarthritis
C: Psoriatic arthritis
D: Reactive arthritis
E: Limited cutaneous systemic sclerosis
A

C: Psoriatic arthritis

Psoriatic arthritis is a form of arthritis which is associated with psoriasis (as you might guess). The condition gets progressively more severe over time and can eventually lead to arthritis mutilans (telescoping of the fingers, where the fingers collapse in on themselves lengthways).

Nail pitting, severe dandruff, and oncholysis (painless separation of nail from the nailbed) are all signs of psoriasis which, together with the joint stiffness, indicates psoriatic arthritis.

226
Q
An ABG is taken on room air.
pH: 7.3 (7.35 – 7.45)
PaO2: 12.8 (11 - 13)
PaCO2: 4.8 (4.7 – 6.0)
HCO3: 16 (24 – 30)
BE: -8 (-2 - +2)

What does this ABG show?

A. Metabolic Acidosis
B. Respiratory Acidosis
C. Metabolic Alkalosis
D. Respiratory Alkalosis
E. Mixed respiratory and metabolic acidosis
A

A. Metabolic Acidosis

The low pH indicates acidosis.
PaO2 and PaCO2 are both within normal limits, however HCO3 is low.
This indicates the acidosis has a metabolic cause, as the bicarbonate measurement is deranged.
PaCO2, although within normal limits, is low, which may indicate that respiration is partially compensating for the metabolic acidosis.

227
Q

An 18 year old girl with a past medical history of anorexia nervosa is brought to A&E by her mother after she told her she had taken a box of 16 paracetamol tablets 2 hours ago, intending to end her life. She has no symptoms other than mild nausea and lethargy.

What is the next most appropriate step in management?

A. IV Naloxone
B. Gastric lavage
C. Wait until 4 hours post ingestion and measure paracetamol levels
D. N-acetylcysteine
E. Reassure and discharge
A

C. Wait until 4 hours post ingestion and measure paracetamol levels

Although N-acetyl cysteine is the treatment for paracetamol overdose, the first step is to wait till the 4 hour post-ingestion mark and measure paracetamol measures. If the amount of paracetamol is above a threshold, treatment will be necessary, but otherwise no medication need be given. Measurement will also indicate how much N-acetyl cysteine will be needed.

228
Q

A 78-year-old woman is admitted to your ward following a 3-day history of shortness of breath and a productive cough of white frothy sputum. On auscultation of the lungs, you hear bilateral basal coarse inspiratory crackles. You suspect that the patient is in congestive cardiac failure. You request a chest x-ray. Which of the following signs is not typically seen on chest x-ray in patients with congestive cardiac failure?

A. Lower lobe diversion
B. Cardiomegaly
C. Pleural effusions
D. Alveolar oedema
E. Kerley B lines
A

A. Lower lobe diversion

All other options are CXR findings in heart failure. Upper lobe diversion of blood (due to congestion) is seen in heart failure, rather than lower lobe diversion.

The main CXR features of heart failure can be remembered using the acronym: Alveolar oedema, Kerley B lines, Cardiomegaly, upper lobe Diversion, Effusion (pleural).

229
Q

A 74 year old female attends a routine rheumatology appointment for her longstanding rheumatoid arthritis. On examination you note a classic ‘rheumatoid hand’.

Which joint is NOT likely to be inflamed in this condition?

A. Wrist joint
B. Metacarpophalangeal joint
C. Proximal interphalangeal joint
D. Thumb metacarpophalangeal joint
E. Distal interphalangeal joint
A

E. Distal interphalangeal joint

The distal interphalangeal joint is always spared in rheumatoid arthritis. Several other specific deformities are characteristic of rheumatoid arthritis: Z deformity of the
thumb, ulnar deviation of the metacarpophalangeal joints, boutonniere deformity, swan-neck deformity and radial deviation of the wrist.

230
Q

A 59 year-old man suffering from autoimmune hepatitis attends the GP for a check up.

Which of the following is the best marker of liver function?

A. ALT (Alanine Transaminase)
B. AST (Aspartate Transaminase)
C. GGT (Gamma glutamyl transferase
D. INR (International normalised ratio)
E. Serum vitamin K level
A

D. INR (International normalised ratio)

Liver function tests are badly named - they don’t indicate liver function, they indicate the degree of cellular damage within the liver, as the enzymes measured in LFTs are released when hepatocytes are damaged.

The liver has a considerable range of synthetic functions, including manufacture of clotting factors. INR is the result of a patient’s Prothrombin time divided by the standard Prothombin time, and so estimates the production of clotting factors, and therefore is a good marker of liver function.

231
Q

A 51-year-old man is brought to A&E having collapsed at home. An ECG is performed, which reveals a regular narrow complex tachycardia with absent P waves. He is unconscious on admission and peripheral cyanosis is observed. BP: 84/56 mm Hg; HR = 136 bpm.

What is the most appropriate management option for this patient?

A. Vagal manoeuvres
B. I.V. adenosine
C. Verapamil
D. D.C. Cardioversion
E. I.V. beta-blocker
A

D. D.C. Cardioversion

The treatment of an SVT is one of the few sets of management that you need to know for third year. Normally the sequence is: vagal manoeuvres then adenosine (or verapamil if adenosine is contraindicated). If these fail then beta blockers or amiodarone may be considered.

However in this case the patient is haemodynamically unstable and so must be cardioverted.

232
Q

A 65 year old gentleman comes in for an ultrasound scan for a AAA following a letter in the post. The scan shows a AAA measuring 5.2 cm.

Which of the following is the most appropriate next step?

A. Endovascular Repair
B. Follow up USS in 2 years
C. Open aortic surgery
D. Follow up USS in 3 months
E. Abdominal CT
A

D. Follow up USS in 3 months

AAA’s are monitored using ultrasound scans and no action is taken to resolve them until either they exceed 5.4cm, or they become symptomatic, or they between 4.0 - 5.4cm and have grown by more than 1.0cm in a single year.

  1. 0 - 4.4cm USS follow-up once every 2 years
  2. 5 - 5.4cm USS follow-up once every 3 months

The options for repair are endovascular repair (EVAR) or open surgery. EVAR is associated with fewer perioperative deaths, but has more long-term complications and is more expensive. Consequently, open surgical repair is recommended instead of EVAR by NICE in most circumstances.

233
Q

A 40 year old patient presents to clinic with lupus pernio and a rash on the legs which resembles target shapes. As they have also had some difficulty breathing, it is arranged for them to have a chest radiograph, which shows bihilar lymphadenopathy.

What is the most likely diagnosis?

A. Erythema nodosum
B. Sarcoidosis
C. Systemic Lupus Erythematous
D. Sjogren’s
E. Bullous Pemphigoid
A

B. Sarcoidosis

Sarcoidosis is an condition characterised by a build-up of non-caseating granulomas in multiple organs, but most classically the lungs.
The giveaway features in this question are the mentions of lupus pernio (poorly named, as it is actually pathognomic of sarcoidosis), and the presence of bihilar lymphadenopathy. In third year SBAs, bihilar lymphadenopathy usually indicates sarcoidosis, but may indicate lymphoma. The target -shaped rashes are annular sarcoidosis, another cutaneous manifestation of the condition.

234
Q

A 40 year old woman is brought into accident and emergency. She was found lying alone on the street by passers-by who called the ambulance. On examination, you note a respiratory rate of 10 and pinprick pupils. Her patient record indicates that she is HIV positive.

Which is the most appropriate management?

A. Mechanical ventilation
B. IV naloxone
C. N-acetylcysteine
D. Methadone
E. IV naltrexone
A

B. IV naloxone

Respiratory depression combined with pinprick pupils indicates opiate overdose, the treatment for which is naloxone - an opioid blocker.

Mechanical ventilation may be required, but it is not the definitive management of overdose. Naltrexone and methadone are used to wean opioid-dependent patients after the acute overdose has been resolved.

235
Q

A 47 year-old man presents with a month history of non-productive cough and general malaise with an accompanying fever. He mentions he recently started collecting exotic birds.

What is the most likely diagnosis

A. Pulmonary fibrosis
B. Chlamydia Psitticai infection
C. Mycoplasma Pneumoniae infection
D. Asthma
E. Extrinsic allergic alveolitis
A

B. Chlamydia Psitticai infection

This could be either extrinsic allergic alveolitis (bird-fancier’s lung) or Chlamydia Psitticai infection (Psittacosis) given the history of a dry cough and malaise, but the fever indicates infection.

Chlamydia Psitticai causes an atypical pneumonia, hence the lack of a productive cough. A CXR would help differentiate between the two, as one-sided localised shadowing would indicate pneumonia, whereas diffuse reticulo-nodular shadowing would indicate extrinic allergic alveolitis.

236
Q

A 16-year-old girl presents to the GP feeling generally unwell for two weeks. She reports fevers and a sore throat. She has a history of well-controlled epilepsy and has recently come back from holiday in Magaluf celebrating her GCSEs. On examination, her temperature is 39.0 °C. The tonsils and oropharynx are erythematous. There is bilateral tender posterior cervical lymphadenopathy.

Which is the rapid point-of-care test used to identify the underlying pathogen?

A. Serum HIV rapid test
B. Monospot test
C. Anti-CMV antibodies
D. Anti-streptolysin O titre
E. Urinary Legionella antigen
A

B. Monospot test

The monospot test is a type of heterophile antibody test (for the purposes of 3rd year SBAs they are the same thing) which is used to diagnose mononucleosis (glandular fever). The Paul-Bunnel test was previosuly used to diagnose mononucleosis, and may be mentioned for that use in SBAs.

This is a standard history of mononucleosis, with lymphadenopathy, swollen tonsils, and fever. The trip to Magaluf presumably involved some ‘tonsil tennis’ which is the classic route of transmission for the virus.

237
Q

A 40 year-old man is brought in to A&E by his friends because he is unresponsive. His friends tell you that he has just had a seizure before coming to A&E. They reluctantly tell you that he has now been abstinent from alcohol for 1 week.

What is the best immediate management for this patient?

A. I.V. Lorazepam
B. Send to ITU
C. Watch and wait
D. Start 0.9% saline infusion
E. Give oxygen 100%
A

A. I.V. Lorazepam

Benzodiazepines are used to make alcohol withdrawal safe, otherwise patients are vulnerable to delerium tremens. Delerum tremens is characterised by shaking, sweating, palpitations, seizures, and hallucinations (one specific example is formication - the sensation of ants crawling over skin).

238
Q

A 14 year-old is found disorientated and semi-conscious on a high street bench in the middle of the day. The front of his shirt is stained with vomit, his face is flushed and swollen, and he has an audible wheeze. He is rushed to A&E.

What is the first step in his management?

A. I.V. chloramphenamine
B. I.V. insulin
C. I.V. adrenaline
D. I.M. adrenaline
E. Gastric lavage
A

D. I.M. adrenaline

This is a history of anaphylaxis, the first step in the management of which is I.M. adrenaline with an epipen. Afterwards the patient will be given antihistamines, fluids, high flow oxygen, and steroids, and their obs will be carefully monitored. Nebulised salbutamol may also be given. It is very important the patient not be discharged within 4 hours of admission, as the anaphylactic reaction may be biphasic - there may be a secondary reaction hours after the initial one.

239
Q

A 53 year-old man presents to A&E with blood in his stool, which he said appeared fresh, and coated the stool. He is feverish, and reports abdominal tenderness in the left iliac region. During the history, the doctor notes the patient’s diet is low in fibre.

What is the most likely diagnosis?

A. Anal fissure
B. Diverticulitis
C. Ulcerative colitis
D. Diverticulosis
E. Appendicitis
A

B. Diverticulitis

Diverticulosis is the presence of small pockets within the colon; it is a common incidental finding and is not inherently problematic. Diverticulitis is infection taking hold within these pockets and leads to fever and blood that coats the stool. Low dietary fibre increases the chance of diverticulosis.

This question is largely based on excluding other options: an anal fissure would cause agony on defecation, ulcerative colitis would cause diarrhoea and the history would probably present a chronic picture, diverticulosis is asymptomatic, and appendicitis would give RIF pain with no bowel symptoms (bar a loss in appetite which is universally present in appendicitis patients).

240
Q

An 8 year-old boy is brought to A&E by his frantic father, who says he ate a pack of something from the medicine packet - in his panic the father didn’t check what it was. The child is tachypnoeic, dizzy, and feverish, and keeps complaining about a high pitched sound nobody else can hear.

What is the treatment given the most likely cause of these symptoms?
A. It will self-resolve
B. N-acetyl cysteine 
C. Naloxone
D. Sodium bicarbonate
E. Flumazenil
A

D. Sodium bicarbonate

Tachypnoea/ hyperventilation, dizziness, and fever are all signs of salicylate (aspirin) poisoning. The ringing noise described is tinnitus, which is fairly unique to aspirin poisoning and so can be useful for spotting it. Without treatment the patient may develop confusion, metabolic acidosis, and coma.

Treatment involves giving activated charcoal if the patient is within 1 hour of ingestion; if not, sodium bicarbonate is given. Sodium bicarbonate increases the pH of the blood, which causes aspirin to dissociate into its ions, preventing it from crossing the blood-brain barrier into the CNS, and increasing urinary clearance. Fluid support is given if and when needed, and in severe cases, haemodialysis is used

241
Q

A 45 year-old woman presents with rapid-onset severe sharp epigastric pan which is accompanied by vomiting and is forcing her to curl into a ball. Though taking a history is difficult, you learn the pain radiates through to her back and is constant, and has never happened before. Obs show she is tachypnoeic, feverish, and tachycardic. Serum amylase is high.

Given the most likely diagnosis, which option is probably the underlying cause?

A. Gallstones
B. Ethanol abuse  
C. Trauma
D. Steroid use
E. Malignancy
A

A. Gallstones

There is a common misconception that alcohol abuse is the most common cause of acute pancreatitis, but it is gallstones (50%), although alcohol abuse is the second biggest cause (25%). The classic acronym for remembering the causes of pancreatitis is GET SMASHED (which has the advantage of listing the most common and second most common causes in order):

Gallstones
Ethanol abuse
Trauma

Steroid use
Mumps/ measles/ malignancy
Autoimmune
Scorpion sting
Hyperlipidaemia/ hypercalcaemia
ERCP
Drugs (e.g. thiazide diuretics, sodium valproate, erythromycin)

P.S. There was one made which spells ITS COMING HOME; it’s not great but here it is if you want:

Ischaemia
Trauma
Scorpion sting

Corticosteroids
Oddi sphincter surgery
Mumps/ measles
Iatrogenic (ERCP)
No cause identified (idiopathic)
Gall stones

Hyperlipidaemia/ hypercalcaemia
Other (drugs)
Malignancy
Ethanol

If you want a bit more info on acute pancreatitis, here is a useful BMJ article: https://www.bmj.com/bmj/section-pdf/762669?path=/bmj/349/7971/Clinical_Review.full.pdf

242
Q

A 24 year-old woman presents with right iliac fossa pain and dizziness. She is pale and tachycardic. An abdominal exam reveals RIF tenderness but no rebound tenderness or guarding, and the patient states the pain has not moved. When asked if she could be pregnant, the patient states she uses an IUD for contraception.

Which is the best test to confirm the diagnosis?

A. Exploratory surgery
B. CT KUB
C. Abdominal CT scan
D. Beta-hCG
E. Trans-vaginal ultrasound
A

E. Trans-vaginal ultrasound

This is a history of an ectopic pregnancy, which commonly features abdominal pain and vaginal bleeding with a recent history of amenorrhea. Other less common symptoms which may occur include: syncope, dizziness, urinary symptoms, rectal pressure/ pain on defecation. The diagnostic tool of choice according to NICE guidelines is trans-vaginal ultrasound, which can confirm the location of the pregnancy. Beta-hCG measurements would also be taken, but they are not the best test for confirming the diagnosis, as well as the position of the implanted embryo.

N.B. The use of an intra-uterine device (IUD) increases the risk of ectopic pregnancy, as does tubal/ uterine surgery, and previous pelvic inflammatory disease.